You are on page 1of 86

PIPE PROBLEMS

A 29.53” x 39.37” pressure vessel contains ammonia with f = 0.041. Compute the minimum
Find the work posses for a Helium gas at 200C required discharge capacity of the relief device in kg/hr.
A. 106.71 kg/hr B. 108.71 kg/hr *C. 110.71 kg/hr D. 112.71 kg/hr
*A. 609 KJ/kg B. 168 KJ/kg C. 229 KJ/kg D. 339 KJ/kg Solution:
C = f D L, kg/s
Solution: C = 0.041(29.53/39.37)(39.37/39.37) = 0.03075 kg/s (3600) = 110.71 kg/hr
W = m R T = m (8.314 / M) T
For helium, M = 4 Compute the maximum length of the discharge pipe installed on the outlet of a pressure-relief
W/m = (8.314/4)(20 + 273) = 609 KJ/kg device in feet for internal pipe diameter of 0.5 inch and rated discharge capacity is 8 lb/min of
air. The rated pressure of relief valve is 16 psig.
Two kilogram of gas is confined in a 1 m3 tank at 200 kpa and 880C. What type of gas is in *A. 0.286 ft B. 0.386 ft C. 0.486 ft D. 0.586 ft
the tank? Solution:
A. Helium *B. Ethane C. Methane D. Ethene P = Pg + Patm = 16 x 1.1 + 14.7 = 32.3 psia
Solution: L = 9P2d5/16Cr2 = 9(32.3)2(0.5)5/16(8)2 = 0.286 ft
PV=mRT
200 (1) = 2 (8.314/M)(88+273) A thermal power plant has a heat rate of 11,363 Btu/kw-hr. Find the thermal efficiency of the
M = 30 plant.
Therefore: the gas is Ethane (C2 H6) A. 28% *B. 30% C. 34% D. 40%
Solution:
Find the enthalpy of Helium if its internal energy is 200 KJ/kg eth = 3412/Heat rate = 3412/11,363 = 30%
A. 144 KJ/kg B. 223.42 KJ/kg *C. 333.42 KJ/kg D. 168 KJ/kg
Solution: What is the hydraulic gradient of a 1 mile, 17 inches inside diameter pipe when 3300 gal/min
R = 8.314/4 = 2.0785 of water flow with f = 0.03.
K = 1.667 for helium *A. 0.00714 B. 0.00614 C. 0.00234 D. 0.0018
Cp = k R/(k - 1) = 1.667(2.0785)/(1.667 – 1) = 5.195 KJ/kg-K Solution:
Cv = R/(k – 1) = 2.0785/(1.667 – 1) = 3.116 KJ/kg – K v = (3300/7.481)/(π/4)(17/12)2(60) = 4.66 ft/s
∆h/∆U = Cp/Cv L = 1 mile = 5280 ft
∆h/200 = 5.195/3.116 hL = fLv2/2_D = 0.03(5280)(4.66)2/2(32.2)(17/12) = 37.7 ft
∆h = 333.42 KJ/kg Hydraulic gradient = 37.7/5280 = 0.007.14

Compute the mass of a 2 m3 propane at 280 kpa and 40˚C. Find the loss of head in the pipe entrance if speed of flow is 10 m/s.
A. 6.47 kg B. 5.1 kg C. 10.20 kg *D. 9.47 kg A. 5.10 m B. 10.2 m C. 17.4 m *D. 2.55 m
Solution: Solution:
Propane is C3 H3--------------M = 12(3) + 8(1) = 44 Loss at entrance = 0.5 (v2/2g) = 0.5 [102 / 2(9.81)] = 2.55 m
PV = m R T
280(2) = m (8.314/44)(40 + 273) Wet material, containing 220% moisture (dry basis) is to be dried at the rate of 1.5 kg/s in a
m = 9.47 kg continuous dryer to give a product containing 10% (dry basis) . Find the moisture removed,
kg/hr
Compute the air flow in ft3/min of mechanical ventilation required to exhaust an accumulation *A. 3543.75 kg/hr B. 3513.75 kg/hr C. 3563.75 kg/hr D. 3593.75 kg/hr
of refrigerant due to leaks of the system capable of revolving air from the machinery room for Solution:
a mass of 4 lbs refrigerant. Solid in wet feed = solid in dried product
*A. 200 B. 210 C. 220 D. 230 [1/(1 + 2.2)](1.5) = [1/(1 + 0.1)](x)
Solution: x = 0.5156 kg/s (total dried product)
Q = 100 x G0.5 ft3/min Moisture removed = 1.5 – 0.5156 = 0.984 kg/s = 3543.75 kg/hr
Q = 100 x (4)0.5 = 200 ft3/min
Copra enters a dryer containing 70% moisture and leaves at 7% moisture. Find the moisture
Compute the free-aperture cross section in m2 for the ventilation of a machinery room if the removed on each pound on solid in final product.
mass of refrigerant is 9 kg. A. 6.258 lb B. 1.258 lb C. 4.258 lb *D. 2.258 lb
A. 0.314 *B. 0.414 C. 0.514 D. 0.614 Solution:
Solution: Solid in wet feed = solid in dried product
F = 0.138 G0.5 m2 0.3x = 1
F = 0.138 (9)0.5 = 0.414 m2 x = 3.333 lbs
1 = 0.93y h2 = h1 + x htg = 418.7 + 0.60(2257) = 1,772.9 KJ/kg
y = 1.07527 lb Q = 1(4.187)(100 – 30) + 1(1772.9 – 418.7) = 1,647.29 KJ/kg
Moisture removed = x – y = 3.333 – 1.07527 = 2.258 lb
Find the enthalpy of water at 212˚F and 14.7 psi if the dryness factor is 30%. Use the
approximate enthalpy formula of liquid.
A 1 m x 1.5 m cylindrical tank is full of oil with SG = 0.92. Find the force acting at the bottom A. 461 Btu/lb *B. 471 Btu/lb C. 481 Btu/lb D. 491 Btu/lb
of the tank in dynes. Solution:
A. 106.33 x 103 dynes B. 106.33 x 104 dynes C. 106.33 x 105 dynes *D. 106.33 x ht = (˚F – 32) = (212 – 32) = 180 Btu/lb
106 dynes htg = 970 Btu/lb
Solution: h = ht + x htg
P = w h = (0.92 x 9.81) (1.5) = 13.5378 kpa h = 180 + 0.3(970) = 471 Btu/lb
F = PA = 13.5378(π/4 x 12) = 10.632 KN = 10,632.56 N x 10,000 dynes/N
F = 106.33 x 106 dynes An air compressor consumed 1200 kw-hr per day of energy. The electric motor driving the
compressor has an efficiency of 80%. If indicated power of the compressor is 34 kw, find the
Find the pressure at the 100 fathom depth of water in kpag. mechanical efficiency of the compressor.
*A. 1,793.96 kpag B. 1,893.96 kpag C. 1,993.96 kpag D. 1,693.96 A. 117.65 % B. 75 % *C. 85 % D. 90 %
kpag Solution:
Solution: P/m = 1200kw-hr/24 hrs = 50 kw
H = 100 fathom x 6 = 600 ft BP = 50(0.80) = 40 kw
P = w h = (600/3.281)(9.81) = 1,793.96 kpag em = 34/40 = 85 %

Find the depth in furlong of the ocean (SG = 1.03) if the pressure at the sea bed is 2,032.56 A refrigeration system consumed 28,000 kw-hr per month of energy. There are 20 % of
kpag. energy is lost due to cooling system of compressor and motor efficiency is 90 %. If COP of
*A. 1 B. 2 C. 3 D. 4 the system is 6, find the tons of refrigeration of the system.
Solution: A. 43.15 TR B. 46.15 TR *C. 49.15 TR D. 41.15 TR
P=wh Solution:
2,032.56 = (1.03 x 9.81) h P/m = 28,800/(24 x 30) = 40 kw
h = 201.158 m x 3.281 ft/m x 1 yd/3ft x 1 furlong/220yd = 1 furlong BP = 40(0.90) = 36 kw
Wc = 36(1 – 0.20) = 28.80 kw
Find the mass of 10 quartz of water. COP = RE/Wc
A. 10.46 kg *B. 9.46 kg C. 11.46 kg D. 8.46 kg 6 = RE/28.80
Solution: RE = 172.8/3.516 = 49.15 TR
V = 10 quartz x 1gal/4quartz x 3.785li/1gal x 1m3/1000li
V = 0.0094625 x 10-3m3 A 23 tons refrigeration system has a heat rejected of 100 kw. Find the energy efficiency ratio
w = m/V of the system.
1000 = m/0.0094625 x 10-3 A. 13.42 *B. 14.42 C. 15.42 D. 16.42
m = 9.46 kg Solution:
QR = RE + Wc
Find the mass of carbon dioxide having a pressure of 20 psia at 200˚F with 10 ft3 volume. 100 = 23(3.516) + Wc
A. 1.04 lbs B. 1.14 lbs *C. 1.24 lbs D. 1.34 lbs Wc = 19.132 kw
Solution: COP = RE/Wc = (23 x 3.516) / 19.132 = 4.32
PV = m R T EER = 3.412 COP = 3.412(4.23) = 14.42
(20 x 144)(10) = m (1545/44)(200 + 460)
m = 1.24 lbs A 200 mm x 250 mm, 8-cylinder, 4-stroke diesel engine has a brake power of 150 kw. The
mechanical efficiency is 80%. If two of the cylinders were accidentally cut off, what will be
Find the heat needed to raise the temperature of water from 30˚C to 100˚C with 60% quality. the new friction power?
Consider an atmospheric pressure of 101.325 kpa. Use the approximate enthalpy formula of A. 31.50 kw B. 33.50 kw C. 35.50 kw *D. 37.50 kw
liquid. Solution:
A. 293.09 KJ/kg B. 1,772.90 KJ/kg C. 1,547.90 KJ/kg *D. 1,647.29 KJ/kg em = BP/IP
Solution: 0.8 = 150/IP
At 100˚C IP = 187.5 kw
ht = cp t = 4.187 (100) = 418.7 KJ/kg FP1 = IP – BP = 187.5 – 150 = 37.50 kw
htg = 2257 KJ/kg FP1 = FP2 = 37.50 kw
PV = m R T
If the energy efficiency ratio of the refrigeration system is 12.6, what is the COP of the 280(2) = m (8.314/44)(40 + 273)
system? m = 9.47 kg
*A. 3.69 B. 4.23 C. 5.92 D. 6.83
Solution: Compute the air flow in ft3/min of mechanical ventilation required to exhaust an accumulation
EER = 3.412 COP of refrigerant due to leaks of the system capable of revolving air from the machinery room for
12.6 = 3.412 COP a mass of 4 lbs refrigerant.
COP = 3.69 *A. 200 B. 210 C. 220 D. 230
Solution:
An air compressor has a power of 40 kw at 4 % clearance. If clearance will increase to 70 %, Q = 100 x G0.5 ft3/min
what is the new power? Q = 100 x (4)0.5 = 200 ft3/min
A. 70 kw *B. 40 kw C. 53kw D. 60 kw
Solution: Compute the free-aperture cross section in m2 for the ventilation of a machinery room if the
The power of compressor will not be affected with the changes in clearance. mass of refrigerant is 9 kg.
Therefore the power will still be 40 kw. A. 0.314 *B. 0.414 C. 0.514 D. 0.613
Solution:
What is the approximate value of temperature of water having enthalpy of 208 Btu/lb? F = 0.138 G0.5 , m2
A. 138.67 ˚C *B. 115.55 ˚C C. 258.67 ˚C D. 68.67 ˚C F = 0.138(9)0.5 = 0.414 m2
Solution:
h = ˚F – 32 A 29.53” x 39.37” pressure vessel contains ammonia with f = 0.041. Compute the minimum
required discharge capacity of the relief device in kg/hr.
Find the work posses for a Helium gas at 20°C. A. 106.71 kg/hr B. 108.71 kg/hr *C. 110.71 kg/hr D. 112.71
*A. 609 KJ/Kg B. 168 KJ/Kg C. 229 KJ/Kg D. 339 KJ/Kg kg/hr
Solution: Solution:
W = m R T = m (8.314/M) T C = f D L, kg/s
For Helium, M = 4 C = 0.041(29.53/39.37)(39.37/39.37) = 0.03075 kg/s (3600) = 110.71 kg/hr
W/m = (8.314/4)(20 + 273) = 609 KJ/Kg
Compute the maximum length of the discharge pipe installed on the outlet of a pressure-relief
Two kilogram of gas is confined in a 1 m3 tank at 200 kpa and 88˚C. What type of gas is in device in feet for internal pipe diameter of 0.5 inch and rated discharge capacity is 8lb/min of
the tank? air. The rated pressure of relief valve is 16 psig.
A. Helium *B. Ethane C. Methane D. Ethene *A. 0.286 ft B. 0.386 ft C. 0.486 ft D. 0.586 ft
Solution: Solution:
PV=mRT P = Pg + Patm = 16 x 1.1 + 14.7 = 32.3 psia
200 (1) = 2 (8.314/m)(88 + 273) L = 9P2d5/16Cr2 = 9(32.3)2(0.5)5/16(8)2 = 0.286 ft
M = 30
Therefore: the gas is Ethane (C2H6) A thermal power plant has a heat rate of 11,363 Btu/kw-hr. Find the thermal efficiency of the
plant.
Find the enthalpy of Helium if its internal energy is 200 KJ/kg. A. 28 % *B. 30 % C. 34 % D. 40 %
A. 144 KJ/kg B. 223.42 KJ/kg *C. 333.42 KJ/kg Solution:
D. 168 KJ/kg eth = 3412/heat rate = 3412/11,363 = 30%
Solution:
R = 8.314/4 = 2.0785 What is the hydraulic gradient of a 1 mile, 17 inches inside diameter pipe when 3300 gal/min
K = 1.667 for helium of water flow with f…. 0.03
cp = k R/(k – 1) = 1.667(2.0785)/(1.667 – 1) = 5.195 KJ/kg-K *A. 0.00714 B. 0.00614 C. 0.00234 D. 0.00187
cv = R/(k – 1) = 2.0785/(1.667 – 1) = 3.116 KJ/kg-K Solution:
∆h/∆U = cp/cv v = (3300/7.481)/(π/4)(17/12)2(60) = 4.66 ft/s
∆h/200 = 5.195/3.116 L = 1 mile = 5280 ft
∆h = 333.42 KJ/kg hL =fLv2/2gD = 0.03(5280)(4.66)2/2(32.2)(17/12) = 37.7 ft
Hydraulic gradient = 37.7/5280 = 0.00714
Compute the mass of a 2 m3 propane at 280 kpa and 40˚C.
A. 6.47 kg B. 5.1 kg C. 10.20 kg *D. 9.47 kg Find the loss of head in the pipe entrance if speed of flow is 10 m/s.
Solution: A. 5.10 m B. 10.2 m C. 17.4 m *D. 2.55 m
Propane is C3H6------------------------M = 12(3) + 8(1) = 44 Solution:
Loss at entrance = 0.5 (v2/2g) = 0.5[102/ 2(9.81)] = 2.55 m w = m/V
1000 = m/0.0094625 x 10-3
Wet material, containing 220 % moisture (dry basis) is to be dried at the rate of 1.5 kg/s in a m = 9.46 kg
continuous dryer to give a product containing 10% (dry basis). Find the moisture removed,
kg/hr. Find the mass of carbon dioxide having a pressure of 20 psia at 200°F with 10 ft3 volume.
*A. 3543.75 kg/hr B. 3513.75 kg/hr C. 3563.75 kg/hr D. 3593.75 kg/hr A. 1.04 lbs B. 1.14 lbs *C. 1.24 lbs D. 1.34 lbs
Solution: Solution:
Solid in wet feed = solid in dried product PV = m R T
[1/(1 + 2.2)](1.5) = [1/(1 + 0.1)](x) (20 x 144)(10) = m (1545/44)(200 + 460)
x = 0.5156 kg/s (total dried product) m = 1.24 lbs
Moisture removed = 1.5 – 0.5156 = 0.984 kg/s = 3543.75 kg/hr

Copra enters a dryer containing 70% moisture and leaves at 7% moisture. Find the moisture Find the heat needed to raise the temperature of water from 30°C to 100°C with 60% quality.
removed on each pound of solid in final product. Consider and atmospheric pressure of 101.325 kpa. Use the approximate enthalpy formula of
A. 6.258 lb B. 1.258 lb C. 4.258 lb *D. 2.258 lb liquid.
Solution: A. 293.09 KJ/kg B. 1,772.90 KJ/kg C. 1,547.90 KJ/kg
Solid in wet feed = solid in dried product *D. 1,647.29 KJ/kg
0.3x = 1 Solution:
x = 3.333 lbs At 100°C
1 = 0.93y hf = Cp t = 4.187(100) 418.7 KJ/kg
y = 1.07527 lb hfg = 2257 KJ.kg
Moisture removed = x – y = 3.333 – 1.07527 = 2.258 lbs h2 = hf + xhfg = 418.7 + 0.60(2257) = 1,772.9 KJ/kg
Q = 1(4.187)(100-30) + 1(1772.9 – 418.7) = 1,647.20 KJ/kg
A 1 m x 1.5 m cylindrical tank is full of oil with SG = 0.92. Find the force acting at the
bottom of the tank in dynes. Find the enthalpy of water at 212˚F and 14.7 psi if the dryness factor is 30%. Use the
A. 106. 33 x 103 dynes B. 106.33 x 104 dynes C. 106.33 x 105 dynes *D. approximate enthalpy formula of liquid.
106.33 x 106 dynes A. 461 Btu/lb *B. 471 Btu/lb C. 481 Btu/lb D. 491
Solution: Btu/lb
P = w h = (0.92 x 9.81)(1.5) = 13.5378 kpa Solution:
F = PA = 13.5378(π/4 x 12) = 10.632 KN = 10,632.56 N x 10,000 dynes/N hf = (˚F – 32) = (212 – 32) = 180 Btu/lb
F = 106.33 x 106 dynes hfg = 970 Btu/lb
h = hf + x hfg
Find the pressure at the 100 fathom depth of water in kpag. h = 180 + 0.3(970) = 471 Btu/lb
*A. 1,793.96 kpag B. 1,893.96 kpag C. 1,993.96 kpag
D. 1,693.96 kpag An air compressor consumed 1200 kw-hr per day of energy. The electric motor driving the
Solution: compressor has an efficiency of 80 %. If indicated power of the compressor is 34 kw, find the
H = 100 fathom x 6 = 600 ft mechanical efficiency of the compressor.
P = w h = (600/3.281)(9.81) = 1,793.96 kpag A. 117.65 % B. 75% *C. 85% D. 90%
Solution:
Find the depth in furlong of the ocean (SG = 1.03) if the pressure at the sea bed is 2,032.56 Pim = 1200kw-hr/24 hrs = 50 kw
kpag. BP = 50(0.80) = 40 kw
*A. 1 B. 2 C. 3 D. 4 em = 34/40 = 85%
Solution:
P=wh A refrigeration system consumed 28,800 kw-hr per month of energy. There are 20 % of
2,032.56 = (1.03 x 9.81) h energy is lost due to cooling system of compressor and motor efficiency is 90%. If COP of the
h = 201.158 m x 3.281 ft/m x 1 yd/3ft x 1 furlong/220yd = 1 furlong system is 6, find the tons of refrigeration of the system.
A. 43.15 TR B. 46.15 TR *C. 49.15 TR D. 41.15 TR
Find the mass of 10 quartz of water. Solution:
A. 10.46 kg *B. 9.46 kg C. 11.46 kg Pim = 28,800/(24 x 30) = 40 kw
D. 8.46 kg BP = 40 (0.90) = 36 kw
Solution: Wc = 36(1 – 0.20) = 28.80 kw
V = 10 quartz x 1gal/4quartz x 3,785 li/1gal x 1m3/1000 li COP = RE/Wc
V = 0.0094625 x 10-3m3 6 = RE/28.80
RE = 172.8/3.516 = 49.15 TR *A. 18.38 B. 16.38 C. 14.38 D. 12.38
Solution:
A 23 tons refrigeration system has a heat rejected of 100 kw. Find the energy efficiency ratio P1V1k = P2V22
of the system. (V1/V2)k = (P2/P1)
A. 13.42 *B. 14.42 C. 15.42 D.16.42 rkk = rp
Solution: rp = (8)1.4 = 18.38
QR = RE + Wc
100 = 23(3.516) + Wc A diesel cycle has a cut off ratio of 2.5 and expansion ratio of 4. Find the clearance of the
Wc = 19.132 kw cycle.
COP = RE/Wc = (23 x 3.516) / 19.132 = 4.23 A. 9.11 % B. 5.55 % *C. 11.11 % D. 15.15 %
EER = 3.412 COP = 3.412(4.23) = 14.42 Solution:
rk = rc re
A 200 mm x 250 mm, 8-cylinder, 4-stroke diesel engine has a brake power of 150 kw. The rk = 2.5(4) = 10
mechanical efficiency is 80 %. If two of the cylinders were accidentally cut off, what will be rk = (1 + c)/c
the new friction power? 10 = (1 + c)/c
A. 31.50 kw B. 33.50 kw C. 35.50 kw *D. 37.50 kw c = 11.11 %
Solution:
em = BP/IP A dual cycle has an initial temperature of 30 ˚C. The compression ratio is 6 and the heat
0.8 = 150/IP addition at constant volume process is 600 KJ/kg. If cut-off ratio is 2.5, find the maximum
IP = 187.5 kw temperature of the cycle.
FP1 = IP – BP = 187.5 – 150 = 37.50 kw A. 3638.50 ˚C *B. 3365.50 ˚C C. 3565.50 ˚C D. 3965.50 ˚C
FP1 = FP2 = 37.50 kw Solution:
T2 = T1 rkk-1 = (30 +273)(6)1.4-1 = 620.44 ˚K
If the energy efficiency ratio of the refrigeration system is 12.6, what is the COP of the QAV = m cv (T3 – T2)
system? 600 = 1(0.7186)(T3 – 620.44)
*A. 3.69 B. 4.23 C. 5.92 D. 60 kw T3 = 1455.396 ˚K
Solution: rc = T4/T3
EER = 3.412 COP 2.5 = T4/1455.396
12.6 = 3.412 COP T4 = 3638.49 ˚K = 3365.50 ˚C
COP = 3.69
A three stages air compressor compresses air from 100 kpa to 1000 kpa. Find the intercooler
An air compressor has a power of 40 kw at 4% clearance. If clearance will increase to 7%, pressure between the first and second stage.
what is the new power? A. 505.44 kpa B. 108.44 kpa C. 316.23 kpa *D. 215.44 kpa
A. 70 kw *B. 40 kw C. 53 kw D. 60 kw Solution:
Solution: Px = (P12P2)1/3
The power of compressor will not be affected with the changes in clearance. Px = [(100)2(1000)]1/3 = 215.44 kpa
Therefore power will still be 40 kw.
A 10-stages air compressor compresses air from 100 kpa to 800 kpa. Find the intercooler
What is the approximate value of temperature of water having enthalpy of 208 Btu/lb? pressure between 1st and 2nd stage.
A. 138.67˚C *B. 115.67˚C C. 258.67˚C D. 68.67˚C A. 282.84 kpa B. 113.21 kpa *C. 123.11 kpa D. 333.51 kpa
Solution: Solution:
h = ˚F – 32 Px = (P1s-1P2)1/s
208 = F – 32 Px = [(100)10-1(1000)]1/10 = 123.11 kpa
˚F = 240 ˚F = 115.55 ˚C
A 3-stages air compressor compresses air from 100 kpa to 700 kpa. Find the intercooler
Convert 750˚R to ˚K pressure between the 2nd and 3rd stage.
A. 390.33 ˚K B. 395.33 ˚K C. 410.33 ˚K *D. 416.33 ˚K *A. 365.88 kpa B. 375.88 kpa C. 385.88 kpa D.
Solution: 395.88 kpa
˚R = 1.8 ˚K Solution:
750 = 1.8 ˚K Px = (P12P2)1/3
˚K = 416.667 Px = [(100)2(700)]1/3 = 191.28 kpa
Px/P1 = Py/Px
An otto cycle has a compression ratio of 8. Find the pressure ratio during compression. Py = Px2/P1 = (191.28)2/100 = 365.88 kpa
At 20 Kpa (0.020 Mpa): sf = 0.8320 hf = 251.4 sfg = 7.0766 hfg =
Carnot cycle A, B and C are connected in series so that the heat rejected from A will be the 2358.3
heat added to B and heat rejected from B will be added to C, each cycle operates between 30 A. 1,117 KJ/kg B. 1,132 KJ/ kg C. 1,123.34 KJ/ kg *D. 1,054.95
˚C and 400 ˚C. If heat added to A is 1000 kw, find the work output of C. KJ/kg
*A. 111.44 kw B. 549.78 kw C. 247.53 kw D. SOLUTION:
141.89 kw s1 = s2 = sf + xsfg
Solution: 6.9486 = 0.8320 + x(7.0766)
e1 = e2 = e3 = (400 – 30)/(400 + 273) = 54.98 % x = 0.8643
e1 = W1/QA1 = (QA1 – QR1)/QA1 h2 = 251.40 + 0.8643(2358.3) = 2289.78 KJ/kg
0.5498 = (1000 – QR1)/1000 h1 - h2a
QR1 = 450.22 = QA2 eST = h1 – h2
0.5498 = (450.22 – QR2)/450.22 0.85 =3530.9 – h2a
By heat balance: 3530.9 – 2289.78
Qgain = Qloss h2a = 2475.95 KJ/kg
mw cp (tb- ta) = mg cpg (t2-t1) WT = h1 = h2a = 3530.9 – 2475.95 = 1,054.95 KJ/kg
(0.30)(4.187)( tb – 15) = (0.5)(1.0717)(150 – 80)
tb = 44.86 oC A steam turbine with 80% stage efficiency receives steam at 7 Mpa and 550oC and exhaust as
20 Kpa. Determine the quality at exhaust.
A 350 mm X 450 mm steam engine running at 280 rpm has an entrance steam condition of 2 At 7 Mpa and 550oC: h1 = 3530.9 Kj/kg s1 = 6.9486
Mpa and 230 oC and exit at 0.1 Mpa. The steam consumption is 2,000 kg/hr and mechanical At 20 Kpa (0.020 Mpa): sf = 0.8320 hf = 251.4
efficiency is 85%. If indicated mean effective pressure is 600 Kpa, determine brake thermal *A. 96.96% B. 76.34% C. 82.34% D. 91.69%
efficiency. SOLUTION:
At 2 Mpa and 230 oC (Table 3): h1 = 2849.6 s1 = 6. 4423 sfg = 7.0766 hfg = 2358.3
At 0.1 Mpa: sf = 1.3026 hf = 417.46 sfg = 6.0568 hfg = 2258 hf2 = 417.46 s1 = s2 = sf + sfg
KJ/kg 6.9486 = 0.8320 + x(7.0766)
A. 23.34% *B. 15.25% C. 14.16% D. 27.34% x = 0.8643
h2 = 251.40 + 0.8643(2358.3) = 2289.78 KJ/kg
SOLUTION: nST = h1 – h2a
VD = 2[3.1416/4 (0.35) 2 (0.45)(280/60)]= 0.4041 m3/sec h1 – h2
Indicated Power = Pmi x VD = 600 x 0.4041 =242.45 KW 0.80 = 3530.9 – h2a
Brake Power = IP (em) = 242.45 (0.85) = 206.08KW 3530.9 – 2289.78
Brake Power 206.08 h2a = 2538.004 KJ/kg
etb = BP/ ms (h1-hf2) =206.08/ (2000/3600)(2849.6 – 417.46)=15.25% h2a = hf + x hfg
2538.004 = 251.40 + x (2358.3)
A steam turbine receives 5,000 kg/hr of steam at 5 Mpa and 4000oc and velocity of 30 m/sec. x = 96.96%
It leaves the turbine at 0.06 Mpa and 85% quality and velocity of 15 m/sec. Radiation loss is
10,000 KJ/hr. Find he KW developed. A 16,000KW geothermal plant has a generator efficiency and turbine efficiency of 90% and
At 5 Mpa and 400oC: h1 = 3195.7 KJ/kg s1 =6.6459 80%., respectively if the quality after throttling is 20% and each well discharges 400, 000
At 0.006 Mpa: hf = 151.53 hfg = 2415.9 kg/hr, determine the number of wells are required to produce if the charge of enthalpy if the
A. 1273.29 B. 2173.29 *C. 1373.60 D. 7231.29 change of enthalpy at entrance and exit of turbine is 500KJ/kg.
SOLUTION: A. 4 wells *B. 2 wells C. 6 wells D. 8 wells
h2 = hf + xhfg = 151.53 + 0.85(2415.9) = 2205.045 KJ/ kg SOLUTION:
KE1 = ½ m v2 = ½ (5,000/3600)(30)2 = 625 W = 0.625 KW WT = ms(h3 – h4)
KE2 = ½ m v2 = ½ (5,000/3600)(15)2 = 156.25 W = 0.15625 KW 16,000 = ms (500)
By energy balance: 0.9(0.8)
KE1 + mh1 = KE2 + mh2 + Q + W ms = 44.44 kg/sec
W = (KE1 – KE2) + m(h1-h2) – Q ms = 160,000 kg/hr
5000 10,000 160,000 = 0.20 mg
W = (0.625 – 0.156) + (3600)(3195.7 – 2205.045) – 3600 = 1373.60 KW mg = 800,000 kg/hr
No. of wells = 800,000/400,000 = 2 wells
A steam turbine with 85% stage efficiency receives steam at 7 Mpa and 550oC and exhausts
as 20 Kpa. Determine the turbine work. A liquid dominated geothermal plant with a single flash separator receives water at 204oC.
At 7 Mpa and 550oC: h1 = 3530.9 Kj/kg s1 = 6.9486 The separator pressure is 1.04 Mpa. A direct contact condenser operates at 0.034 Mpa. The
turbine has a polytropic efficiency of 0.75. For a cycle output of 60 MW, what is the mass
flow rate of the well-water in kg/s? Brake thermal efficiency =
At 204oC: hf = 870.51 KJ/kg
At 1.04 Mpa: hf = 770.38 hfg = 2009.2 hg = 2779.6 sg = 6.5729 A waste heat recovery boiler produces 4.8 Mpa(dry saturated) steam from 104°C feedwater.
At 0.034 MPa: hf = 301.40 hfg = 2328.8 sf = 0.9793 sfg = 6.7463 The boiler receives energy from 7 kg/sec of 954°C dry air. After passing through a waste heat
*A. 2,933 B. 2,100 C. 1,860 D. 2,444 boiler, the temperature of the air is has been reduce to 343°C. How much steam in kg is
SOLUTION: produced per second? Note: At 4.80 Mpa dry saturated, h = 2796.
h3 = hg at 1.04 MPa = 2779.6 KJ/kg A. 1.30 B. 0.92 *C. 1.81 D. 3.43
Solving for h4: SOLUTION:
s3 = s4 = sf + xsfg hf = approximate enthalpy of feedwater
6.5729 = 0.9793 + x4(6.7463) hf = Cpt
x4 = 0.829 hf = 4.187(104)
h4 = 301.4 + 0.829(2328.8) = 2232.3 KJ/kg hf = 435.45 KJ/kg
WT = ms (h3 – h4) Heat loss = Heat gain
60,000 = ms (2779.6 – 2232.3) 0.75 m gc p(t 1 - t 2) = m s(h - h f)
ms = 146.17 kg/sec 7(1.0)(954 – 343) = ms(2796.0 – 436.45)
Solving for x2: (h1 = h2) m s = 1.81 kg/sec
h1 = h2 = hf + xhfg
870.51 = 770.38 + x2(2009.2) A diesel electric plant supplies energy for Meralco. During a 24-hour period, the plant
x2 = 0..049836 consumed 240 gallons of fuel at 28°C and produced 3930 KW-hr. Industrial fuel used is
ms = x mg 28°API and was purchased at P30 per liter at 15.6°C. What is the cost of the fuel be to produce
146.17 = 0.049836 mg one KW-hr?
mg = 2,933.06 kg/sec *A. P6.87 B. P1.10 C. P41.07 D. P5.00
SOLUTION:
An engine-generator rated 9000 KVA at 80% power factor, 3 phase, 4160 V has an efficiency SG 15.6C = 141.5/(131.5 + 28) = 0.887
of 90%. If overall plant efficiency is 28%, what is the heat generated by the fuel. Density at 15.6°C = 0.887(1kg/li) = 0.887 kg/li
A. 18,800 KW B. 28,800 KW C. 7500 KW *D. 25,714 SG 28C = 0.887[1-.0007(1 – 15.6)] = .879
KW Density at 28°C = 0.879(1 kg/li) = 0.879 kg/li
SOLUTION: V28C / V15.6C = SG15.6C / SG28C
Gen. Output = pf x KVA = 0.8 x 9000 = 7200 KW 240 / V15.6C = 0.887 / 0.879
eoverall= Gen. Output V15.6C = 237.835 gallons x 3.785 li/gal = 900.21 li
Qg Cost = [(30)(900.21)] / 3930 = P6.87/KW-hr

0.28 = 7200/Qg In a gas turbine unit, air enters the combustion chamber at 550 kpa, 277°C and 43 m/s. The
Qg = 25,714.28 KW products of combustion leave the combustor at 511 kpa, 1004°C and 180 m/s. Liquid fuel
enters with a heating value of 43,000 KJ/kg. For fuel-air ratio of 0.0229, what is the combustor
The indicated thermal efficiency of a two stroke diesel engine is 60%. If friction power is 15% efficiency of the unit in percent?
of heat generated, determine the brake thermal efficiency of the engine. A. 70.38% B. 79.385% *C. 75.38% D. 82.38%
A. 43% *B. 45 % C. 36% D. 37% SOLUTION:
SOLUTION: Heat supplied by fuel = mfQh = 0.0229(43,000) = 984.7 KJ/kg air
ne = IP/ Qg Q = heat absorbed by fuel
0.60 = IP/Qg Q/m = Cp(T2 – T1) + ½(V22 – V12)
IP = 0.60 Qg Q/m = (1.0)(1004 – 277) + ½[(180) 2 –(43) 2]/1000 =742.28 KJ/kg air
BP = IP- FP = 0.60Qg – 0.15Qg = 0.45Qg
etb = BP/Qg = 0.45Qg/Qg = 45%
Combustor Efficiency = = 75.38%
A 305 mm x 457 mm four stroke single acting diesel engine is rated at 150 KW at 260 rpm.
The specific speed of turbine is 85 rpm and running at 450 rpm. If the head is 20 m and
Fuel consumption at rated load is 0.56 kg/KW-hr with a heating value of 43,912 KJ/kg.
generator efficiency is 90%, what is the maximum power delivered by the generator.
Calculate brake thermal efficiency
A. 450.51 KW B. 354.52 KW C. 650.53 KW *D. 835.57 KW
A. 10.53% B. 27.45% *C. 14.64% D. 18.23%
SOLUTION:
SOLUTION:
NS = (N√HP)/h5/4
mf = 0.56 kg/KW-hr x 150 KW = 84 kg/hr = 0.0233 kg/sec
85 = (450√HP)/(20 x 3.281) 5/4
Hp = 1244.52 3.6 = (2 x .00093 x 100 x 22.759) / (9.81D)
Generator Output = (1244.52 x 0.746)(0.9) = 835.57 KW D = 2.728 m

In Francis turbine, the pressure gage leading to the turbine casing reads 380 Kpa. The velocity Q=Axv=[ 2](22.759) = 133.03 m3/sec
of water entering the turbine is 8 m/sec, if net head of the turbine is 45 m, find the distance Power = w Q h = 9.81(133.03)(26.4) = 34,452 KW
from center of spiral casing to the tailrace.
*A. 3.0 m B. 3.5 m C. 4.0 m D. 4.5m Water flows steadily with a velocity of 3.05 m/s in a horizontal pipe having a diameter of 25.4
SOLUTION : cm. At one section of the pipe, the temperature and pressure of the water are 21C and 689.3
Kpa, respectively. At a distance of 304.8 m downstream

h= V2/2g A hydro electric plant having 30 sq. km reservoir area and 100 m head is used to generate
45 = (380/9.81) + z + [82/(2 x 9.81)] power. The energy utilized by the consumers whose load is connected to the power plant
z=3m during a five-hour period is 13.5 x 106 kwh. The overall generation efficiency is 75%. Find
the fall in the height of water in the reservoir after the 5-hour period.
A turbine has a mechanical efficiency of 93%, volumetric efficiency of 95% and total A. 5.13 m B. 1.32 m C. 3.21 *D. 2.20 m
efficiency of 82%. If effective head is 40 m, find the total head. SOLUTION
A. 48.72 m B. 40.72 m *C. 36.22 m D. 34.72 m Energy Output = Power x time = (w Q h) x time
SOLUTION: 13.5 x 106 = 9.81(Q)(100)(0.75)(5)
eT = emehev Q = 3669.725 m3/s
0.8 = 0.93(eh)(.95) Volume after 5 hrs = 3669.725(5 x 3600) = 66,055,050 m3
ηh = 0.9055 Volume = A x height
Total head = h eh = (40)(0.9055) = 36.22 m 66,055,050 = (30 x 106) h
H =2.202 m
A Pelton type turbine has 25 m head friction loss of 4.5 m. The coefficient of friction head loss
(from Moorse) is 0.00093 and penstock length of 80 m. What is the penstock diameter? The gas density of chimney is 0.75 kg/m3 and air density of 1.15 kg/m3. Find the driving
*A. 1,355.73 mm B. 3,476.12 mm C. 6771.23 mm D. pressure if the height of chimney is 63.71 m.
1686.73 mm A. 0.15 kpa *B. 0.25 kpa C. 0.35 kpa D. 0.45 kpa
SOLUTION: SOLUTION:
h =25- 4.5 = 20.5 hw = H(da – dg) = 63.71(1.15 – 0.75) (0.00981) = 0.25 kpa
v = √(2gh) = [(2 x 9.81 x 20.5)1/2] = 20.55 m/sec The actual velocity of gas entering in a chimney is 8 m/sec. The gas temperature is 25C with a
hL = (2fLv2)/gD gas constant of 0.287 KJ/kg-K. Determine the gas pressure for a mass of gas is 50,000 kg/hr
4.5 = (2)(0.00093)(80)(20.055)2 / 9.81D and chimney diameter of 1.39m.
D = 1,355,730 m = 1,355.73 mm A. 95 kpa *B. 98 kpa C. 101 kpa D. 92 kpa
SOLUTION:
In an 9,000 KW hydro-electric plant the over-all efficiency is 88% and the actual power
Vg = A x v = / 4 (1.39)2(8) = 12.139 m3/s
received by the customer is 110,000 KW-hrs for that day. What is the secondary power could
PgVg = mgRgTg
this plant deliver during the entire day?
P(12.139) = (50,000/3600)(.287)(25 +273)
A. 58,960 KW-hrs *B. 80,080 KW-hrs C. 65,960 KW-hrs D. 70,960
P = 97.85 kpa
KW-hrs A steam generator with economizer and air heater has an overall draft loss of 25.78 cm of
SOLUTION: water. If the stack gases are at 177C and if the atmosphere is at 101.3 Kpa and 26C, what
Plant Capacity = 9,000(0.88)(24) = 190,080 KW-hrs theoretical height of stack in meters is needed when no draft fan are used? Assume that the gas
Secondary Power = 190,080 – 110,000 = 80,080 KW-hrs constant for the flue gases is the same as that for air.
A 611.10 B. 631.10 *C.651.10 D.671.10
A Pelton type turbine was installed 30 m below the gate of the penstock. The head loss due to SOLUTION:
friction is 12 percent of the given elevation. The length of penstock is 100 m and coefficient of w = P/RT
friction is 0.00093. Determine the power output in KW. ( Use Moorse equation) da = (101.325)/[(.287)(26 + 273)] = 1.180 kg/m3
A. 22,273 B. 23,234 C. 32,345 *D. 34,452 dg = (101.3)/[(0.287)(177 +273)] = 0.784 kg/m3
SOLUTION: Draft = (0.2578)(1000) = 257.80 kg/m3
hL = 0.12(30) = 3.6 m Draft = H(da – dw)
h = 30 – 3.6 = 26.40 m 257.80 = H(1.18 – 0.784)
v = (2gh)1/2 = [(2)(9.81)(26.4)]1/2 = 22.759 m/sec H = 651.10 m
hL= (2fLv2)/gD
A foundation measures 12 ft x 14 ft x16 ft. Find the number of sacks of cement needed for
1:2:4 mixture. b = 135 – 40 = 95C
A. 302 B. 404 C. 356 *D. 598 mean = [ a - b] / [ln( a b)] = [95-15] / ln(95/15) = 43.34C
SOLUTION: Water is flowing in a pipe with radius of 30 cm at a velocity of 5 m/s at the temperature in
V = 12 X 14 X 16 = 2,688 ft3 (1 yd3 / 33 ft3) = 99.55 yd3 of concrete pipe. The density and viscosity of the water are as follows: density 997.9 kg/sec viscosity =
For every 1 yd3 concrete, it needs 6 sacks of cement 1.131 Pa-s. What is the Reynolds Number for this situation?
Therefore: *A. 2647 B. 96.2 C. 3100 D. 1140
No. of sacks = 6(99.55) = 597.33 sacks or 598 sacks SOLUTION:
A rectangular foundation cross-section has a bed plate dimension of 8 ft x 10 ft. The uniform n = Dvg / v
clearance on each side is 1 ft. The height of foundation is 4.5 ft. If the weight of the steel bar Where:
reinforcements needed is 1/2% of weight of foundation, find the weight of steel bars. Use D = 2(0.30) = 0.60 m
concrete density of 2400 kg/m3 . vg = 5 M/SEC
A. 173.47 kg *B. 183.47 kg C. 163.47 kg D. 153.47 kg v = 1.131/997.9 = 0.0011334 m2 / sec
SOLUTION: Nm = 0.60(5)/0.0011334 = 2,647
A = (8 + 2) (10 + 2) = 120 m2 Compute the amount of condensate form during 10 minutes warm-up of 180 meter pipe
V = Ah = 120(4.5) = 540 ft3 = 15.29 m3 conveys the saturated steam with enthalpy vaporization hfg = 1,947.8 LJ/kg. The minimum
W = wV = (2400)(15.29) = 36,693.25 kg external temperature of pipe is 2C. The final temperature of pipe is 195C. The specific heat of
Weight of steel bars = (1/2%) Wf = 0.005(36,693.25) = 183.47 kg pipe material is 0.6 KJ/kg-C. The specific weight is 28 kg/m.
A. 249.69 kg B. 982.45 kg *C. 299.64 kg D.
A steam pipe having a surface temperature of 250C passes through a room where the 423.45 kg
temperature is 27 C. The outside diameter of pipe is 100 mm and emissivity factor is 0.8. SOLUTION:
Calculate the radial heat loss for 3 m pipe length. mp = mass of pipe = 28(180) = 5,040 kg
A. 1434.7 W B. 37.46 W *C. 2651.82 W D. 3545.45 W Heat Loss by steam = Heat loss from pipe
SOLUTION: m(hg - hf) = mpcp (t2 – t1)
m(1947.8) = (5040)(0.6)(195-2)
A = DL = = 0.425m2 m = 299.64 kg
Solving for heat due to radiation:
Tg = 250 +273 = 523K
T2 = 27 +273 = 300K The discharge pressure of an air compressor is 5 times the suction pressure. If volume flow at
Qa = 20,408.4 x 104 AF(T14 – T24), J/hr = 20,408.4 x 104(0.8)(0.7539)[(523)4 – suction is 0.1 m3/sec, what is the suction pressure if compressor work is 19.57 kw? (use
(300)4] n=1.35)
Qr = 10,266,539.06 j/hr x 1hr/3600sec = 2851.82 W A. 97 kpa *B.98 kpa C. 99 kpa D.100 kpa
Brine enters a circulating brine cooler at the rate of 60 m3/hr at -*C and leaves at -18C. SOLUTION:
Specific heat of brine is 1.072 KJ/kg-K and specific gravity of 1.12. Determine the tons of
refrigeration. W= [(P2/P1)n-1/n – 1]
A. 53.5 TR B. 65.3 TR C.33.5 TR *D. 56.9 TR 19.57 = 1.35(P1)(0.1)/(1.35-1)[(5)1.35-1/1.35 – 1]
P1 = 98 KPa
SOLUTION:
Density of brine = 1.12(1000 kg/m3) = 1120 kg/m3
The initial condition of air in an air compressor is 98 KPa and 27C and discharge air at 450
m = (1120)(60)/3600 = 18.67 kg/sec
KPa. The bpre and stroke are 355 mm and 381 mm, respectively with percent cleared of 8%
Q = mcp = 18.67(1.072)(-8 + 18) = 200.11 KW running at 300 rpm. Find the volume of air at suction.
TR = 200.11/3.516 = 56.91 Tons of refrigeration A. 541.62 m3/hr B. 551.62 m3/hr C. 561.62 m3/hr *D.
A turbo-charged, 16 cylinder, Vee-type diesel engine has an air consumption of 3,000 kg/hr 571.62 m3/hr
per cylinder at rate load and speed. This air is drawn in through a filter by a centrifugal SOLUTION:
compressor directly connected to the exhaust gas turine. The temperature of the air from the ev = 1 + c – c(P2/P1)1/n = 1 + 0.08 - 0.08(450/98)1/1.4 = 0.842
compressor is 135C and a counter flow air cooler reduces the air temperature to 45C before it
goes to the engine suction heater. Cooling water enters air cooler at 30C and leaves at 40C. VD = D2 LN = (0.355)2 (0.381)(300/60) = 0.1885 m3/sec
Calculate the log mean temperature difference. V1 = 0.1885(0.842) = 0.15878 m3/sec = 571.62 m3/hr
A. 47.23C B. 87.82C *C. 43.34C D. 65.24C An air compressor has a suction volume of 0.35 m3/sec t 97 KPa and discharges to 650 KPa.
SOLUTION: How much power saved by the compressor of there are two stages?
A. 18.27 KW B. 16.54 KW C. 13.86 KW *D.
a = 45-30 = 15C 11.58 KW
SOLUTION: W = (1.249 x 96.5 x 0.343) / (1.249-1) [(480 / 96.5)1.249-1/1.249 – 1]
W = 62.57 KW
W= [(P2/P1)n-1/n – 1] = (1.4 x 97 x 0.35)/(1.4 -1) [(650/97)1.4-1/1.4 – 1] = Q = heat loss = mcp(t2 – t1) = (10.9/3600)(4.187)(21 – 15) 0.075 KW
85.79 KW Brake power = W + Q = 62.57 + 0.076 = 62.65 KW
For two stages :
Px = (P1P2)1/2 = (97 x 650)1/2 = 251.097 KPa A double suction centrifugal pumps delivers 20 ft3/sec of water at a head of 12 m and running
at 650 rpm. What is the specific speed of the pump?
W= [(Px/P1)n-1/n – 1] = 2(1.4)(97)(0.35)/(1.4 – 1) [(251.0.97/97)1.4-1/1.4 – A. 5014.12 rpm B. 6453.12 rpm *C. 2770.73 rpm D. 9966.73 rpm
1] = 74.208 KW SOLUTION:
POWER SAVED = 85.79 – 74.208 = 11.582 KW N = N(Q)1/2 / h3/4
Q = 20/2 ft3/sec x 7.481 gal/ft3 x 60 sec/1min = 4,488.6 gal/min
A twop stage air compressor has an intercooler pressure of 4 kg/cm2. What is the discharge h = 12 x 3.281 = 39.37 ft
pressure if suction pressure is 1 kg/cm2? N = (650 x (4,488.6)1/2)/(39.37)3/4
A. 3 kg/cm2 B. 9 kg/cm2 C. 12 kg/cm2 *D. 16 kg/cm2 N = 2,770.73 rpm
SOLUTION: Determine the number of stages needed for a centrifugal pump if it is used to deliver 400
Px = (P1P2)1/2 gal/min of water and pump power of 15 Hp. Each impeller develops a head of 30 ft.
Px2 = P1(P2) A. 6 B. 4 *C. 5 D. 7
42 = 16 kg/cm2 SOLUTION:
Wp = w Q h
A two stage air compressor compresses air at 100 KPa and 22C discharges to 750 KPa. If 15 x 0.746 = 9.81(400 gal/min x 0.00785m3/gal x 1/60)h
intercooler intake is 105C. Determine the value of n. h = 45.20 m x 3.281 ft/m = 148.317 ft
A. 1.400 *B. 1.325 C. 1.345 D. 1.288 Number of stages = 148.317/40 = 4.94 stages = 5 stages
SOLUTION: The suction pressure of a pump reads 3 in. of mercury vacuum and discharge pressure reads
Px = (100 x 750)1/2 =273.86 KPa 140 psi is use to deliver 120 gpm of water with specific volume of 0.0163 ft3/lb. Determine
Tx/T1 = (Px/P1)n-1/n the pump work.
(105 + 273)/(22 + 273) = (273.86/100)n-1/n A. 4.6 KW B. 5.7 KW *C. 7.4 KW D. 8.4 KW
1.281 = (2.6268)n-1/n SOLUTION:
n = 1.326 P1 = -3 in Hg x 101.325/29.92 = -10.16 KPa
A single acting compressor has a volumetric efficiency of 89%, operates at 500 rpm. It takes P2 = 140 psi x 101.325/14.7 = 965 KPa
in air at 900 KPa and 30C and discharges it at 600 KPa. The air handled is 8 m3/min measured w = 1/v = 1/0.163 = 61.35 lb/ft3 x 9.81/62.3 = 9.645 KN/m3
at discharge condition. If compression is isentropic, find mean effective pressure in KPa. h = (P2 – P1)/w = (965 +10.16)/9.645 = 101.105 m
*A. 233.34 B. 973.17 C. 198.34 D. 204.82 Q = 120 gal/min x 3.785/1gal x 1m3/1000li x 1/60 = 0.00757 m3/sec
SOLUTION: P = w Q h = 9.645(0.00757)(101.105) = 7.38 KW
P1V1K = P2V2K A submersible pump delivers 350 gpm of water to a height of 5 ft from the ground. The pump
100(V11.4) = 600(6)1.4 were installed 150 ft below the ground level and draw down of 8 ft during the operation. If
V1 = 28.768 m3/min water level is 25 ft above the pump, determine the pump power.
VD = 28.768/0.89 = 32.32 m3/min A. 7.13 KW B. 4.86 KW C. 7.24 KW *D.
W = n P1V1/n-1 x [(P2 / P1)n-1/n – 1] = [(1.4 x 100 x 32.32)/(1.4 – 1)] x 9.27 KW
[(600/100)1.4-1/1.4 – 1] SOLUTION:
W = 7562.19 KJ/min h = 5 + 150 – (25 – 8) = 138/3.281 = 42.06 m
W = Pm x Vd Q = 350 gal/min x 0.003785 m3/gal x 1 min/60sec = 0.02246 m3/sec
7562.19 = Pm x 32.32 Wp = w Q h = 9.81(0.02246)(42.06) = 9.27 KW
Pm = 233.34 KPa
A water-jacketed air compressed handles 0.343 m3/s of air entering at 96.5 KPa and 21C and A vacuum pump is used to drain a flooded mine shaft of 20 water. The pump pressure of
leaving at 460 KPa and 132C; 10.9 kg/h of cooling water enters the jacket at 15C and leaves at water at this temperature is 2.34 KPa. The pump is incapable of lifting the water higher than
21C. Determine the compressor brake power. 16 m. What is the atmospheric pressure?
A. 26.163 KW *B. 62.650 KW C. 34.44 KW D. *A. 159.30 B. 32.33 C. 196.22 D. 171.9
19.33 KW SOLUTION:
Using Bernoulli’s Theorem:
SOLUTION: P1/w + V12/2g + z1 = P2/w + V2/2g + z2
T2/T1 = (P2/P1) n-1/n P1/w = P2/w + (V22 - V12)/2g + (z2 - z1)
(132+273) / (21+273) = (480/96.5)n-1/n P1/9.81 = 2.34/9.81 + 0 + 16
n = 1.249
P1 = 159.30 KPa mt + ma + mf = 409
A submersible, multi-stage, centrifugal deep well pump 260 gpm capacity is installed in a well ma = 409 – 2(18.34) = 372.32 grams
27 feet below the static water level and running at 3000 rpm. Drawdown when pumping at QA = macp(t3 – t2)
rated capacity is 10 feet. The pump delivers the water into a 25,000 gallons capacity overhead 780.752 = 0.37232(1.11)(T3 – 956.964)
storage tank. Total discharge head developed by pump, including friction in piping is 243 feet.
Calculate the diameter of the impeller of this pump in inches if each impeller diameter T3 = 2846,146
developed a head of 38 ft. rC = T3/T2 = 2846.146/956.964 = 2.97
A. 3.28 B. 5.33 *C. 3.71 D. 6.34
SOLUTION:
V= DN The gain of entropy during isothermal nonflow process of 5 lb of air at 60 is 0.462 Btu/R.
V= Find the V1/V2.
A. 3.85 *B. 0.259 C. 1.0 D. 0.296
D (3000/60) = (2(32.2)(38))1/2 SOLUTION:
D = 0.315 ft = 3.708 inches
A fan pressure of 2.54 cm of water t 1.42 m3 per second of air at static pressure of 2.54 cm of s = m R T ln(V2/V1)
water through a duct 300 mm diameter and discharges it through a duct 275 mm diameter. 0.462 = 5 (53.33/778) ln (V2/V1)
V2/V1 = 3.85
Determine the static fan efficiency if total fan mechanical is 75% and air measured at 25 V1/V2 = 1/3.85 = 0.259
and 60 mm Hg.
A. 50.11% *B. 53.69% C. 65.67% D. 45.34%
SOLUTION: An auditorium seating 1500 people is to be maintained at 80 dry bulb and 85 wet bulb
wA = P/RT = 101.325/(0.287)(25 + 273) = 1.18 kg/m3 temperature when outdoor air is at 91 dry bulb and 75 wet bulb. Solar heat load is
hA = hwww/wA = (0.0254)(1000)/1.18 = 21.52 m
110,000 Btu/hr and supply air at 60 determine the amount of supply air.
vA = 1.42/( /4)(0.3)2 = 20.09 m/s *A. 93,229.17 lb/hr B. 83,229.17 lb/hr C. 73,229.17 D. 63,229.17 lb/hr
Vd = 1.42/( /4)(0.275)2 = 23.9 m/s SOLUTION:
hv = (23.9)2 – (20.09)2 / 2(9.81) = 8.54 m Sensible heat per person = 225 Btu/hr
h = ha + hv = 21.52 + 8.54 = 30.06 m Qa = 225(1500) + 110,000 = 447,500 Btu/hr
eT = wa Q h/BP Qa = m cp(t1 – t2)
0.75 = (1.18 x 0.00981)(1.42)(30.06) / BP 447,500 = ma(0.24)(80 – 60)
BP = 0.6588 KW ma = 93,229.17 lb/hr
ep = wa Q hs/BP = (1.18 x 0.00981)(1.42)(21.52) / 0.6588 = 53.69%
In a Brayton cycle that operates between temperature limits of 300K and 1773K wit k = 1.4,
A water cooler uses 50 lb/hr of melting ice to cool running water from 80 to 42 . Based determine the temperature at the end of the compression (isentropic) for maximum work of the
on te inside coil area, U1 = 110 Btu/hr-ft2- . Find the gpm of water cooled. cycle.
A. 0.10 GPM B. 0.21 GPM *C. 0.38 GPM D. 0.45 GPM A. 700K B. 590.5K *C. 730K D. 350K
SOLUTION: SOLUTION:
Q = mf L = mwcpw(t1 – t2) For maximum work: T2 = (T1T3)1/2
50 (144) = mW(1)(80-42) T2 = (300 x 1773)1/2 = 730K
mw = 189.474 lb/hr
V = (189.474/62.4) (7.48/60) = 0.38 GPM At 35% solution leaves the absorber and 30% solution enters the absorber. The heat removed
The charge in a Diesel engine consists of 18.34 grams of fuel, with lower heating value of from the absorber by cooling water is 547.6 Btu and ammonia is superheated by 10 . Find the
42,571 KJ/kg, and 409 grams of fuel and products of combustion. At the beginning of pound per pound of ammonia gas from the evaporating coils.
compression, t1 = 60 . Let rk = 14. For constant cP = 1.11 KJ/kg-C, what should be the cut- A. 11 B. 12 *C. 13 D. 14
off ratio in the corresponding ideal cycle? SOLUTION:
A. 2.05 B. 2.34 C. 5.34 *D. 2.97 n = lb/lb of ammonia gas from the coils
SOLUTION: n = (1 - x2) / (x1 - x2) – 1
QA = mfQh = 0.01283(42,571) = 780,752 KJ n = (1- 0.3) / (0.35 – 0.3) – 1 = 13
T2/T1 = rkk-1 A Carnot refrigeration system operates at Tmax / Tmin = 1.5. Find the KW per tons of
T2 = (60 + 273)1.4-1 = 956.964K refrigeration.
mt + mg = 409 A. 1.91 B. 2.15 *C. 1.76 D. 1.55
SOLUTION:
Wo / TR = 3.516 / COP = 3.516 / (Tmin / (Tmax – Tmin)
Wo / TR = [3.516(Tmax – Tmin)] / Tmin Determine the size of pipe which will deliver 8 liters of medium oil (v= 6.10 x 10-6 m2/s)
Wo / TR = 3.516[(Tmax / Tmin) – 1] = 3.516(1.5-1) = 1.758 KW/TR assuming laminar flow conditions:
A. 622 mm B. 754 mm C. 950 mm *D. 835 mm
Assume 8 ft3 of air at 100 psi, 100 are compressed isothermally to a volume of 2 ft3. For SOLUTION:
each of end states of the process, find the bulk modulus.
*A. 400 and 100 psi B. 400 and 110 psi C. 400 and 120 psi D. 400 and
130
V=
An empty open can is 30 cm high with a 15 cm diameter. The can, with the open end and
down, is pushed under water with a density of 1000kg/m3. Find the water level in the can Re =
when the top of the can is 50 cm below the surface. For laminar flow, Re = 2000
A. 17.20 cm *B. 2.12 cm C. 4.20 cm D. 5.87cm
SOLUTION:
Consider the water pressure
Pw = w h + 1010.325 + (0.8-x)(9.81) + 101.325 = 109.173 – 9.81x 2000 =
Consider the air pressure d = 0.835 m = 835 mm
P1V1 = P2V2
101.325(Ax0.3) = P2[A(0.3-x)] The type of flow occupying in a 1 cm diameter pipe which water flows at a velocity of 2.50
m/s. Use v = 1.13 x 10-6 m2/s for water.
*A. turbulent B. constant C. laminar D. none of the above
P2 =
Pw = P2 SOLUTION:

109.173 – 9.81x = Re =
9.81x2 – 112.116x + 2.3705 = 0
By quadratic formula: Re =
X = 0.02118 m = 2.12 cm
Re = 22,124
A cylindrical pope with water flowing downward at 0.03 m3/s having top diameter of 0.08, Since it is greater than 2000 then it is turbulent flow
bottom diameter of 0.04 m and a height of 1.5m. Find the pressure between the pipe
A. 154.63 Kpa B. 197.93 Kpa *C. 252.44 Kpa D. 243.92 Kpa What is the force is exerted by water jet 60 mm diameter if it strikes a wall at the rate of 15
SOLUTION: m/s?
*A. 636.17 N B.442.62 N C. 764.23 N D. 563.34 N
+ Z1 = + Z2 SOLUTION:
F=wQv

Q=Av= = 0.0424 m3/s


F = (1000)(0.0424)(15) = 636.17 N
Z1 – Z2 = 1.5 m
Z2 – Z1 = -1.5 m A 300 mm diameter pipe discharges water at the rate of 200 li/s. Point 1 on the pipe has a
pressure of 260 kpa and 3.4 m below point 1 is point 2 with a pressure of 300 kpa. Compute
the head loss between points 1 and 2.
V1 = = 5.968 m/s A. 4.29 m B. 2.59 m C. 6.32 m *D. 1.87 m
SOLUTION:

V2 = = 23.87 m/s hL
hL =

P1 – P2 = 252.44 Kpa
Water flowing at the rate of 10 m/s from an orifice at the bottom of a reservoir. Find the To = T1 + v2/2000Cp
pressure at the bottom of the reservoir. 468 = 420 + v2/2000
A. 30 kpag B. 40 kpag *C. 50 kpag D. 60 kpag v = 309.838 m/s
SOLUTION:
h = V2/ 2g = 102/ 2(9.81) = 5.0968 m C=
P = w h = 9.81(5.0968) = 50 kpag M = v/C = 309.838/410.8 = 0.754

Steam flows through a nozzle at 400oC and 1 Mpa (h = 3263.9 KJ/kg) with velocity of 300 Air at 300oK and 200 kpa is heated at constant pressure to 600oK. Determine the change of
m/s. Find the stagnation enthalpy. internal energy.
A. 3300 KJ/kg B. 3290 KJ/kg *C. 3320 KJ/kg *D. 3309 KJ/kg A. 245.58 KJ/kg B. 235.58 KJ/kg C. 225.58 KJ/kg *D. 215.58
SOLUTION: KJ/kg
ho = h + v2/2000 = 3263.9 + 3002/2000 = 3309 KJ/kg SOLUTION:
ΔU = mCv (T2 – T1) = 1(0.7186)(600 -300) = 215.58 KJ/kg
Air flows through a nozzle at a speed of 350 m/s. Find the stagnation temperature if entrance
temperature is 200oC. An insulated rigid tank initially contains 1.5 lb of helium at 80oF and 50 psia. A paddle wheel
A. 241.25oC B. 251.25oC *C. 261.25oC D. 271.25oC with power rating of 0.02 hp is operated within the tank for 30 min. Determine the final
SOLUTION: temperature.
To = T1 + v2/2000Cp = (2000 + 273) + 3502/2000(1) A. 159.22oF B. 169.22oF *C. 179.22oF D. 189.22 oF
To = 534.25oK = 261.25 SOLUTION:
W = ΔU = m Cv (T2 – T1)
Carbon dioxide flows through a nozzle with a speed of 400 m/s. Compute the dynamic 0.02 hp (0.50hr)(2545Btu/hr/hp) = 1.5(0.171)(t2 – 80)
temperature. t2 = 179.22oF
A. 92.56oK *B. 94.56oK C. 96.56oK D. 98.56oK
SOLUTION: A 4m2 asphalt pavement with emissivity of 0.85 has a surface temperature of 50oC. Find the
For CO2: Cp = 0.846 KJ/kg-K maximum rate of radiation that can be emitted from the surface.
Dynamic temperature = v2/2000Cp = 4002/2000(0.846) = 94.56oK A. 2,068.32 watts B. 2,078.32 watts C. 2,088.32 watts *D. 2.098.32
watts
Carbon dioxide flows through a nozzle with a speed of 380 m/s. The entrance condition of SOLUTION:
nozzle is 250oC and 1200 kpa. Find the stagnation pressure. Qr = e kev A Ts4
*A. 2,136.34 kpa B. 2,146.34 kpa C. 2,156.34 kpa D. 2,166.34 Kev = 5.67 x 10-8 ( Stefan Boltzman constant)
kpa Qr = 0.85(5.67 z 10-8)(4)*50 +273)4 = 2,098.32 watts
SOLUTION:
T1 = 250 + 273 = 523oK Air at 10oC and 90 kpa enters a diffuser of a jet engine steadily with a velocity of 200 m/s.
To = T1 + v2/2000 = 523 = 3802/2000 = 595.2oK The inlet area diffuser is 0.40 m2. Determine the mass flow rate of air.
P1 = 1200 kpa A. 72.79 kg/s B. 74.79 kg/s C. 76.79 kg/s *D. 78.79 kg/s
T1/To = (P1/Po)k-1/k SOLUTION:
For CO2: k = 1.289 W = P/RT = 80/0.287(10 + 273) = 0.985 kg/m3
523/595.2 = (1200/Po)1.289-1/1.289 m = w v A = 0.985(200)(0.40) = 78.79 kg/s
P0 = 2,136.34 kpa
Consider a refrigeration whose 40 watts light bulb remains on continuously as a result of a
Air enters a diffuser with a velocity of 200 m/s. Determine the velocity of sound if air malfunction of the switch. If the refrigerator has a COP of 1.3 and the cost of electricity is 8
temperature is 30oC. cents per kw-hr., determine the increase in the energy consumption of the refrigerator and its
*A. 349 m/s B. 359 m/s C. 369 m/s D. 379 m/s cost per year if the switch is not fixed.
SOLUTION: *A. P49.59 B. P47.59 C. P45.59 D. P43.59
C= SOLUTION:
COP = RE/Wref
1.3 = 40/Wref
Wref = 30.769 watts
W = Wb + Wref = 40 + 30.769 = 70.77 watts
Air flows through a nozzle with temperature of entrance of 420oK stagnation temperature of W = 0.07077 Kw
468oK. Find the mach number. Cost = 0.07077(8760)(P0.08) = P49.59
A. 0.744 *B. 0.754 C. 0.764 D. 0.774
SOLUTION:
A 75 hp motor that has an efficiency of 91% is worn out and is replaced by a high-efficiency A. 382 m/s B. 540 m/s C. 458 m/s *D. 568 m/s
motor that has an efficiency of 95.4%. Determine the reduction in heat gain of the room due to SOLUTION:
higher efficiency under full-load conditions. e = (v2/v3)2
A. 2.24 KW *B. 2.44 KW C. 2.64 KW D. 2.84 KW 0.9 = (v2/600)2
SOLUTION: v2 = 568.21 m/s
P01 = (75 x 0.746)(0.91) = 50.91 KW
P02 = (75 x 0.746)(0.954) = 53.376 KW A 50 kg block of iron casting at 500K is thrown into a large lake that is at a temperature of
Qreduced = 53.376 – 50.91 = 2.44 KW 258oK. The iron block eventually reaches thermal equilibrium with the lake water. Assuming
average specific hear of 0.45 KJ/kg-K for the iron, determine the entropy generated during this
A household refrigerator that has a power input of 450 watts and a COP of 2.5 is to cool five process.
large watermelons, 10 kg each, to 8oC. If the watermelons are initially at 20oC, determine *A. -12.65 KJ/k B. 16.97KJ/K C. 4.32 KJ/K D. 6.32 KJ/K
how long will take for the refrigerator cool them. The watermelons can be treated as a water SOLUTION:
whose specific heat is 4.2 KJ/kg-oK. ΔSiron = m c ln (T2/T1) = 50(0.45)ln(285/500) = -12.65 KJ/K
A. 2220 seconds B. 2230 seconds *C.2240 seconds D. 2250 ΔSlake = Q/T = [50(0.45)(500-285)]/285 = 16.97 KJ/K
seconds ΔSgen. = -12.65 + 16.97 = 4.32 KJ/K
SOLUTION:
COP = RE/Wc A windmill with a 12 m diameter rotor is to be installed at a location where the wind is
2.5 = RE/450 blowing at an average velocity of 10 /s. Using standard conditions of air (1 atm, 25oC),
RE = 1,125 watts determine the maximum that can be generated by the windmill.
RE = m cp (t2 – t1) A. 68 KW *B. 70 KW C. 72 KW D. 74 KW
450 t = (10 x 5)(4.2)(20-8) SOLUTION:
t = 2240 seconds w = P/RT = 101.325/(0.28)(25+ 273) = 1.1847 kg/m3
m = w A v = 1.1847(π/4 x 122)(10) = 1,1339.895 kg/s
When a man returns to his wall-sealed house on a summer day, he finds that the house is at KE = v2/2000 = 102/2000 = 0.05 KJ/kg
32oC. He returns on the air conditioner which cools the entire house to 20oC in 15 minutes, if Power = m KE = 1,1339.895(0.05) = 70 KW
COP is 2.5, determine the power drawn by the airconditioner. Assume the entire mass within
the house is 800 kg of air for which cv = 0.72 KJ/kg-K, cp = 1.0KJ/kg-K. Consider a large furnace that can supply heat at a temperature of 2000oR at a steady rate of
A. 1.072 KW B. 2.072 KW *C. 3.072 KW D. 4.072 KW 3000Btu/s. Determine the energy of this energy. Assume an environment temperature of 77oF.
SOLUTION: A. 2305.19 KW *B. 2315.19 KW
RE = m cv (T2 –T1) = (800/15x60)(0.72)(32-20) C. 2325.19 KW D. 2335.19 KW
RE = 7.66 KW SOLUTION:
Wc = 7.68/2.5 = 3.072 KW
e= = 0.7315
A heat source at 8000K losses 2000 KJ of heat to a sink at 500oK. Determine the entropy W = e Q = 0.7315(3000) = 2194.5 Btu/s = 2315.19 KW
generated during this process.
*A. 1.5 KJ/K B. 2.5 KJ/K C. -2.5 KJ/K D. 4
KJ/K
SOLUTION: A heat engine receives hat from a source at 1200oK at a rate of 5000KJ/s and rejects the waste
ΔSsource = -2000/800 = -2.5 heat to a medium at 3000oK. The power output of the heat engine is 180 KW. Determine the
ΔSsink = 2000/500 = 4 irreversible rate for this process.
ΔSgen. = -2.5 + 4 = 1.5 KJ/K A. 190 KW *B. 195 KW C. 200 KW D. 205 KW
SOLUTION:
Helium gas is compressed in an adiabatic compressor from an initial state of 14 psia and 50oF e = (1200 – 300) / 1200 = 0.75
to a final temperature of 320oF in a reversible manner. Determine the exit pressure of Helium. W = 0.75(500) = 375 KW
A. 38.5 psia *B. 40.5 psia C. 42.5 psia D. Irreversibilities = 375 – 180 195 KW
44.5 psia
SOLUTION: A dealer advertises that he has just received a shipment of electric resistance heaters for
T2/T1 = (P2/P1)n-1/n residential buildings that have an efficiency of 100 percent. Assuming an indoor temperature
(320 + 460)/(50 +460) = (P2/14)1.587-1/1.587 of 21oC and outdoor temperature of 10oC, determine the second law efficiency of these
P2 = 40.5 psia heaters.
A. 8.74% B. 6.74% *C. 3.74% D. 4.74%
Air pass thru a nozzle with efficiency of 90%. The velocity of air at the exit is 600 m/s. Find SOLUTION:
the actual velocity at the exit. COP1 = 100% efficient = 1
COP2 = (21 + 273) / (21 – 10) = 26.72 A rigid tank contains 5 kg of an ideal gas at 4 atm and 40oC. Now a valve is opened, and half
e = COP1 /COP2 = 1/ 26.72 = 3.74 % of mass of the gas is allowed to escape. If the final pressure in the tank is 1,5 atm, the final
temperature in the tank is:
A thermal power plant has a heat rate of 11,363 Btu/KW-hr. Find the thermal efficiency of the *A. -38oC B. -30oC C. 40oC D. 53oC
plant. SOLUTION:
A. 34% B. 24% C. 26% *D. 30% PV = m R T
SOLUTION: (4 x 9.81)(V) = 5(0.287)(40 + 273)
e = 3412 / Heat rate = 3412 / 11363 = 30 % V = 11.446 m3
PV = mRT
A rigid tank contains 2 kmol of N2 and 6 kmol of CO2 gasses at 300oK and 115 Mpa. Find (1.5 x 9.81)(11.446) = (5/2)(0.287)(T)
the tank volume us ideal gas equation. T = 234.74oK = -38.26oC
A. 7.33 m3 B. 5.33 m3 C. 3.33 m3 *D. 1.33 m3
SOLUTION: The pressure of an automobile tire is measured to be 200 kpa(gage) before the trip and 220
PmVm = Nm R Tm kpa(gage) after the trip at a location where the atmospheric pressure is 90 kpa. If the
15,000 Vm = (6 + 2)(8.314)(300) temperature of the air in the tire before the trip is 25oC, the air temperature after the trip is:
Vm = 1.33 m3 *A. 45.6oC B. 54.6oC C. 27.5oC D. 26.7oC
SOLUTION:
A spherical balloon with a diameter of 6 m is filled with helium at 20oC and 200 kpa. T2 / T1 = P2 / P1
Determine the mole number. T2 / (25+ 273) = (220 +90) / (200 + 90)
*A. 9.28 Kmol B. 10.28 Kmol C. 11.28 Kmol D. 13.28 Kmol T2 = 318.55 K
SOLUTION: t2 = 45.55oC
PV=NRT
(200)[(4/3)(π)(6/2)3] = N (8.314)(20 + 273) Water is boiling at 1 atm pressure in a stainless steel pan on an electric range. It is observed
N = 9.28 Kmol that 2 kg of liquid ater evaporates in 30 mins. The rate of heat transfer to the water is:
A. 2.97 KW B. 0.47 KW *C. 2.51 KW D. 3.12 KW
The air in an automobile tire with a volume of 0.53 ft3 is at 90oF and 20 psig. Determine the SOLUTION:
amount of air that must be added to raise the pressure to the recommended value of 30 psig.
Assume the atmospheric to be 14.7 psia and the temperature and the volume to remain Q = mL = (2257) = 2.51 KW
constant. Consider a person standing in a breezy room at 20oC. Determine the total rate of heat transfer
*A. 0.026 lb B. 0.046 lb C. 0.066 lb D. 0.086 lb from this person if the exposed surface area and the average outer surface temperature of the
SOLUTION: person are 1.6 m2 and 29oC, respectively, and the convection heat transfer coefficient is 6
PV=mRT W/m2 with emissivity factor of 0.95.
(20 + 14.7)(144)(0.53) = m1 (53.3)(90 + 460) A. 86.40 watts B. 61.70 watts C. 198.1 watts *D. 168.1 watts
m1 = 0.09034 lb SOLUTION:
(30 + 14.7)(144)(0.53) = m2(53.3)(90 + 460) Qc = h A (t2 – t1) = (6)(1.6)(29.20) = 86.40 watts
m2 = 0.11634 lb Qr = (0.95)(5.67 x 10-6)[(1.6)(29 + 273)4 – (20 + 273)4] = 81.7 watts
madded = m2 – m1 = 0.11634 – 0.09034 = 0.026 lb Q = Qc + Qr = 86.40 + 81.7 = 168.1 watts

A rigid tank contains 20 lbm of air at 20 psia and 70oF. More air is added to the tank until the Water is boiler in a pan on a stove at sea level. During 10 minutes of boiling, it is observed
pressure and temperature rise to 35 psia and 90 oF, respectively. Determine the amount of air that 200 grams of water has evaporated. Then the rate of heat transfer to the water is:
added to the tank. A. 0.84 KJ/min *B. 45.1 KJ/min C. 41.8 KJ/min D. 53.5 KJ/min
A. 11.73 lb *B. 13.73 lb C. 15.73 lb D. 17.73 lb SOLUTION:
SOLUTION: Q = m L = (0.2 / 10) (2257) = 45.1 KJ/min
P1V1 = m1 R1T1
(20 x 144)(V1) = 20 (53.3)(70 + 460) An aluminum pan whose thermal conductivity is 237 W/m-C has a flat bottom whose diameter
V = 196.17 ft3 is 20 cm and thickness 0.4 cm. Heat is transferred steadily to boiling water in the pan through
P2V2 = m2R2T2 its bottom at a rate of 500 watts. If the inner surface of the bottom of the pan is 105oC,
(35 x 144)(196.17) = m2 (53.3)(90 + 460) determine the temperature of the surface of the bottom of the pan.
m2 = 33.73 lbs A. 95.27 oC *B. 105.27oC C. 115.27oC D. 125.27oC
madded = m2 –m1 = 33.73 – 20 = 13.73 lb SOLUTION:
A = π / 4 ( 0.20)2 = 0.0314 m2
Q=
A classroom that normally contains 40 people is to be air- conditioned with window air-
conditioning units of 5 KW cooling capacity. A person at rest may be assumed to dissipate
500 = heat at a rate of about 360 KJ/hr. There are 10 light bulbs in the room, each with a rating of
100 watts. The rate of heat transfer to the classroom through the walls and the windows is
T2 = 105.27oC estimated to be 15,00 KJ/hr. If the room air is to be maintained at a constant temperature of
21oC, determine the number of window air- conditioning units required.
For heat transfer purposes, a standing man can be modeled as a 30 cm diameter, 170 cm long A. 1 unit *B. 2 units C. 3 units D. 4 units
vertical cylinder with both the top and bottom surfaces insulated and with the side surface at SOLUTION:
an average temperature of 34oC. For a convection heat transfer coefficient of 15 W/m2- oC, Q = total head load = 40(360/3600) + 10(0.100) + 15,000/3600 = 9.167 KW
determine the rate of heat loss from this man by convection in an environment at 20oC. No. of air-conditioning = 9.167/5 = 1.833 = 2 units
A. 316.46 watts B. 326.46 watts *C. 336.46 watts D. 346.46 watts
SOLUTION: A 4m x 5m x 6m room is to be heated by a baseboard resistance heater. It is desired that the
Qc = k A (t2 – t1) = 15 (π x 0.30 x 1.7) (34 – 20) = 336.46 watts resistance heater be able to raise the air temperature in the room from 7 to 23oC within 15
minutes. Assuming no heat losses from the room and an atmospheric pressure of 100 kpa,
A 5cm diameter spherical ball whose surface is maintained at a temperature of 70oC is determine the required power of the resistance heater. Assume constant specific heats at room
suspended in the middle of a room at 20oC. If the convection heat transfer coefficient is 15 temperature.
W/m2 – C and the emissivity of the surface is 0.8, determine the total heat transfer from the A. 2.34 KW *B. 1.91 KW C. 4.56 KW D.
ball. 6.34 KW
A. 23.56 watts *B. 32.77 watts C. 9.22 watts D. 43.45 watts SOLUTION:
SOLUTION: w = P / R T = 100 / (0.287)(7 +273) = 1.244 kg / m3
A = 4 π r2 = 4 π (0.05)2 = 0.0314 m2 m = 1.244 (4 x 5 x 6) = 149.28 kg
Qc = h A (t2 – t1) = 15 (0.0314)(70 – 20) = 23.56 watts Q = m cv (t2 – t1 ) = 149.28 (0.7186)(23 – 7) = 1,716.36 KJ
Qr = (0.80)(5.67 x 10-6)(0.0314)[(70 + 273)4 – (50 + 273)4] = 9.22 watts Power = 1,716.36 / (15 x 60) = 1.91 KW
Q = Qr + Qc = 23.56 + 9.22 = 32.77 watts
A student in a 4m x 6m x 6m dormitory room turns on her 150 watts fan before she leaves the
A frictionless piston-cylinder device and rigid tank contain 1.2 kmol of ideal gas at the same room on a summer day, hoping that the room will be cooler when she comes back in the
temperature, pressure, and volume. Now heat is transferred, and the temperature of both evening. Assuming all the doors and windows are tightly closed and disregarding any heat
system is raised by 15oC. The amount of extra heat that must be supplied to the gas in the transfer through the walls and the windows, determine the temperature in the room when she
cylinder that is maintained at constant pressure. comes back 10 hours later. Use specific heat values at room temperature, and assume the room
to be at 100 kpa and 15oC in the morning when she leaves
SOLUTION: A. 28.13oC B. 38.13oC C. 48.13oC *D. 58.13oC
A. 0 B. 50 KJ C. 100 KJ *D. 150 KJ SOLUTION:
Q = m cp (t2 – t1) = (1.2 x 8.314)(1)(15) = 150 KJ w = P / R T = 100 / (0.287)(15 + 273) = 1.2098 kg / m3
m = 1.2098(4 x 6 x 6) = 174.216 kg
A supply of 50 kg of chicken needs at 6oC contained in a box is to be frozen to -18oC in a Q = m cv (t2 – t1)
freezer. Determine the amount of heat that needs to be removed. The latent heat of chicken is 0.15(10 x 3600) = 174.216 (0.7186)(t2 – 15)
247 KJ/kg, and its specific heat is 3.32 KJ/kg-oC above freezing and 1.77 KJ/kg-oC below t2 = 58.13oC
freezing. The container box is 1.5 kg, and the specific heat of the box material is 1.4 Kj/kg-oC.
Also the freezing temperature of chicken is -2.8oC. A piston cylinder device whose piston is resting on top of a set stops initially contains 0.50 kg
*A. 15,206.4 KJ B. 50.4 KJ C. 15,156 KJ D. 1,863 KJ of helium gas at 100 kpa and 25oC. The mass of the piston is such that 500 kpa of pressure is
SOLUTION: required to raise it. How much heat must be transferred to the helium before the piston starts
Qchicken = 50 [3.32(6 + 2.8) = 247 1.77(-2.8 + 18)] = 15,156 KJ rising?
Qbox = 1.5(1.4)(6 + 8) = 50.4 KJ A. 1557.13 KJ B. 1657.13 KJ C. 1757.13 KJ *D. 1857.13
Q = 15,156 + 50.4 = 15, 206.4 KJ KJ
SOLUTION:
Water is being heated in a closed pan on top of a range while being stirred by a paddle wheel. For helium: cv = R / (k-1) = (8.314 / 4) (1.667 – 1) = 3,116 KJ/ kg-K
During the process, 30 KJ of heat is transferred to the water, and 5 KJ of heat is lost to the T2 = (25 + 273)(500 / 100) = 1,490oK
surrounding air. The paddle wheel work amounts to 500 N-m. Determine the final energy of T1 = 25 + 273 = 298oK
the system if its initial energy is 10 KJ. Q = m cv (T2 – T1) = 0.50(3.116)(1490 – 298) = 1857.13 KJ
*A. 35.5 KJ B. 45.5 KJ C. 25.5 KJ D. 14.5 KJ
SOLUTION: In order to cool 1 ton (100kg) of water at 20oC in an insulated tank, a person pours 80 kg of
Final energy = Qa + ΔU – Qloss + W = 30 + 10 – 5 + 0.50 = 35.5 KJ ice at -5oC into the water. Determine the final equilibrium temperature in the tank. The
melting temperature and the hat of fusion of ice at atmospheric pressure are 0oC and 333.7 If the ambient outside the vacuum shell is 15oC, calculate the magnitude of the conductive
kJ/kg, respectively. heat flow in watts along the support rods.
*A. 12.43oC B. 14.43oC C. 16.43oC D. 18.43oC *A. 0.168 B. 0.0587 C. 0.182 D. 0.176
SOLUTION: SOLUTION:
Qwater =Qice Q = h A (Δt) = 16.3 (π/4 x 0.0082)(15 – (-190)) = 0.168 watts
1000(4.187)(20 –te) = 80(2.09)(0 + 5) + 80(333.7) + 80(4.187)(te – 0) An elastic sphere containing gas at 120 kpa has a diameter of 1.0 m. Heating the sphere causes
te = 14.43oC it to expand to a diameter of 1.3 m. During the process the pressure is proportional to the
sphere diameter. Calculate the work done by the gas in KJ.
A fan is powered by a 0.5 hp motor and delivers air at a rate of 85 m3/min. Determine the A. 41.8 B. 50.6 *C. 87.5 D. 35.4
highest value for the average velocity of air mobilized by the fan. The density of air to 1.18 SOLUTION:
kg/m3. PαD
P=kD
A. 18.23 m/s *B. 21.12 m/s C. 25.34 m/s D. 120 = k(1)
32.23 m/s K = 120
SOLUTION: P = 120 D
P=wQh V = 4/3 π(D/d)2 = 4/24 π D3
0.50(0.746) = ( 1.18 x 0.00981)( 85 / 60) (h) dV = (12/24) π D2 dD
h = 22.74 m
W= D3dD
v= = 21.12 m/s W = 87.47 KJ
An Ocean – Thermal Energy Conversion power plant generates 10,000 KW using a warm An ideal gas with a molecular weight of 7.1 kg/kg mol is compressed from 600 kpa and
surface water inlet temperature of 26oC and a cold deep- water temperature of 15oC. ON the 280oK to a final specific volume of 0.5 m3/kg. During the process the pressure varies
basis of a 3oC drop in the temperature of the warm water and a 3oC rise in the temperature of according to p = 620 + 150v + 95v2 where p is in kpa and v in m3/kg. Calculate the work of
the cold water due to removal and addition of heat, calculate the power required in KW to compression in KJ/kg?
pump the cold- deep water to the surface and through the system heat.Assume a Carnot cycle A. 32.8 B. 28.7 C. 35.6 *D. 33.3
efficiency and density of cold water to be 1000 kg/m3. SOLUTION:
A. 108 *B. 250 C. 146 D. 160 V1 = R T / P = (8.314 / 7.1)(280) / (600) = 0.546 m3/kg
SOLUTION:
e = (Th – TL)/ TH = [(26 + 273) – (15 + 273)] / (26 + 273) = 0.0679
e =W / Qa W=
0.03676 = 10, 000 / QA
QA = 271, 612. 99 KW
QR = Qa – W = 271,812.99 – 10, 000 = 261,813 KW
QR = m cp (Δt)
261, 813 = m (4.187)(3) One cubic meter container contains a mixture of gases composed of 0.02 kg-mol of oxygen
M = 20, 643.32 kg/s and 0.04 kg-mol of helium at a pressure of 220 kpa. What is the temperature of this ideal gas
Q = 20,843.32 kg/s or 20,843.32 li/s = 20.843 m3/s mixture in degrees Kelvin?
H = P / w = 12 / 9.81 = 1.223 m *A. 441 B. 350 C. 400 D. 450
Wp = w Q h = 9.81(20.843)(1.223) = 250.12 KW SOLUTION:
V = V1 + V2
A plate – type solar energy collector with an absorbing surface covered by a glass plate is to
receive an incident radiation of 800 W/m2. The glass plate has a reflective of 0.12 and a VT =
transmissivity of 0.85. The absorbing surface has an absorptivity of 0.90. The area of the
collector is 5m2. How much solar energy in watts is absorbing by the collector? 1=
A. 2500 B. 2880 C. 3510 *D. 3060
SOLUTION: T = 441 oK
Q = heat absorbed from sun
Q = (800 W/m2)(5 m2)(0.85)(0.9) = 3,060 watts Methyl alcohol (CH3OH) is burned with 25% excess air. How much unburned oxygen in kg-
mol-oxygen / kg-mol fuel will there be in the products if the combustion is complete?
A tank contains liquid nitrogen at -190oC is suspended in a vacuum shell by three stainless A. 0.35 B. 0.45 *C. 0.37 D. 0.65
steel rods 0.80 cm in diameter and 3 meters long with a thermal conductivity of 16.3 W/m2-C. SOLUTION:
CH3OH + O2 + (3.76)N2 = CO2 + H2O + (3.76)N2
CH3OH + 1.5O2 + 1.5(3.76)N2 = 1 CO2 + 2H2O + 1.5(3.76)N2 SOLUTION:
Consider 25% excess air: Cv of helium = 3118.9 j/kg-C
CH3OH + 1.25(1.5)O2 + 1.25(1.5)(3.76)N2 = 1CO2 + 2H2O + 1.25(1.5)(3.76)N2 + m g h = m cv Δt
0.25(1.5)O2 m (9.81)(4500) = m (3118.9) Δt
Unburned O2 = 0.25(1.5) = 0.375 Δt = 14.15 oC

A 12 DC electrical motor draws a current of 15 amps. How much work in KJ does this motor Consider two Carnot heat engines operating in series. The first engine receives heat from the
produce over a 10 minute period of operation? reservoir at 2500oK and rejects the waste heat to another reservoir at temperature T. The
*A. 108.0 B. 129.6 C. 216.0 D. 318.2 second engine receives heat by the first one, convert some of it to work, and rejects the rest to
SOLUTION: a reservoir at 300oK. If thermal efficiencies of both engines are the same, determine the
W=E=QV temperature T.
W = (15 x 10 x 60)(12) = 108,000 J = 108 KJ *A. 849oK B. 578 oK C. 763 oK D. 976 oK
SOLUTION:
A 4 liter (2-liter per revolution at standard pressure and temperature) spark ignition engine has et = e2
a compression ratio of 8 and 2200 KJ/kg heat addition by the fluid combustion. Considering a
cold air-standard Otto cycle model, how much power will the engine produce when operating
at 2500 rpm? T = 849oK
*A. 166.53 hp B. 73.12 hp C. 97.4 hp D. 148 hp
SOLUTION: An ideal gas mixture consists of 2 kmol of N2 and 6 kmol of CO2. The mass fraction of CO2
W = 1.2 kg/m3 (standard density of air) is:
M = 2 li/rev x 2500 rev/min x 1.2 kg/m3 x 1 m3/ 1000 li x 1 min/ 60 sec = 0.10 kg/s A. 0.175 B. 0.250 *C. 0.825 D. 0.750
e = W / QA SOLUTION:
e = 1 -1 / 8 1.4 -1 = 0.5647
0.5647 = W / 2200 6(44)
W = 1,242.34 KJ/kg (0.10 kg/s) = 124.23 KW = 166. 53 hp Mass fraction of CO2 = = 0.825
6(44) + 2(28)
A simple Rankine cycle produces 40 MW of power, 50 MW of process heated and rejects 50 An ideal gas mixture consists of 2 kmol of N2 and 6 kmol of CO2. The apparent gas constant
MW of heat to the surroundings. What is the utilization factor of this cogeneration cycle of mixture is:
neglecting the pump work? *A. 0.208 B. 0.231 C. 0.531 D. 0.825
A. 50% B. 60% *C. 64% D. 80% Solution:
SOLUTION: M= (2/8)(28) + (6/8)(44) = 40
QA = WT + WprocessWp = 40 + 50 + 50 = 140 KW
R= 8.314/M = 8.314/40 = 0.208 KJ/kg-K
A Carnot cycle operates between the temperature limits of 300OK and 1500OK, and
UF = = 64 % produces 600 KW of net power.
The rate of entropy changes of the working fluid during the heat addition process is:
The rate of heat transfer to the surroundings’ from a person at a rest is about 400 KJ/hr. A. 0 B. 0.4KW/K *C. 0.5KW/K D.2.0KW/K
Suppose that the ventilation system fails in an automobile in an auditorium containing 120
Solution:
people and assuming that the energy goes into the air of volume 1500 m3 initially at 300oK
W= ( s) (TH – TL)
and 101 kpa, calculate the rate in oC/min of air temperature change.
600 = ( s) (1500 – 300)
A. 0.81 B. 0.53 *C. 0.63 D. 1.0
SOLUTION: ( s)= 0.50 KW/K
Q = m cv Δt Air in an ideal Diesel cycle is compressed from 3L to 0.15L and then it expands during the
PV = m RT constant pressure heat addition process to 0.3L. Under cold air standard conditions, the
101(1500) = m (0.287)(300) thermal efficiency of this cycle is:
m = 1,759.58 kg rk = 3/0.15= 20
Q = m cv Δt rc= 0.3/0.15= 2
120(400 / 60) = 1759.58 (0.7186)(Δt)
Δt = 0.633 oC / min 1 rck- 1 1 21.2 - 1
e=1- = 1- = 0.6467
An insulated box containing helium gas falls from a balloon 4.5 km above the earth’s surface. rkk-1 k(rc – 1) 201.4-1 1.4 (2-1)
Calculate the temperature rise in oC of the helium when box hits the ground. =64.67%
A. 15.2 B. 12.6 C. 25.3 *D. 14.1
Helium gas is an ideal Otto cycle is compressed from 20oC and 2L to 0.25L and its A water reservoir contains 100,000 kg of water at an average elevation of 60 m. The
temperature increases by an additional 800oC during the addition process. The temperature maximum amount of electric power that can be generated from this water is:
of helium before the expansion process is: A.8KWh *B.16KWh C.1630KWh D.58, 800KWh
*A. 1700oC B. 1440oC C. 1240oC D.880oC Solution:
Solution: P= m h = (100,000 x 0.00981)(60)= 58,860 KJ
rk =2/0 25 = 8 P= 58,860 KJ x KWh/3600 KJ = 16.35KWh
T2 = (20+273) (8)1.667-1 =1,172K A house is maintained at 22oC in winter by electric resistance heaters. If the outdoor
T3 = T2 + 800 = 1172 + 800 = 1972oK temperature is 5oC, the second law of efficiency of the resistance heaters is:
t3 = 1699oC = 1700oK A.0% *B.5.8% C.34% D.77%
In an ideal Brayton cycle has a net work output of 150KJ/kg and backwork ratio of 0.4. If Solution:
both the turbine and the compressor had an isentropic efficiency of 80%, the net work ea= 100% of resistance heaters
output of the cycle would be. et= (22 – 5)/(22 + 273) = 5.8%
A. 50KJ/kg *B. 75KJ/kg C. 98KJ/kg D.120KJ/kg es= 5.8/100 = 5.8%
Solution: A thermoelectric refrigerator that resembles a small ice chest is powered by a car battery,
Backwork ratio = WO/WT and has a COP of 0.10. If the refrigerator cools at 0.350L canned drink from 20OC to 4OC in
0.40 = WO/WT 30 min. determine the average electric power consumed by the thermoelectric refrigerator.
WO = 0.40 WT *A.130 watts B.110 watts C.120 watts D.140 watts
Wnet = WT - WO Solution:
150 = WT – 0.4 WT (1 x 0.35)
WT =250 KJ/kg Q= m cp(t2 – t1) = (4.187)(20 – 4) = 13 watts
WT ‘=250(0.8) = 200KJ/kg 30 x 60
WP = 0.40(200) =100KJ/kg COP= RE/Wc
WP’ = 100/0.80 =125 KJ/kg 0.10= 13/0.10= 130 watts
Wnet= WT’ – WC’= 200 – 125 = 75 KJ/kg A Carnot refrigerator operates in a room in which the temperature is 25OC and consumes 2
Air entered a turbojet engine at 200 m/s at a rate of 20 kg/s, and exists at 800 m/s relative to kW of power when operating. If the food compartment of the refrigerator is to be
the aircraft. the thrust developed by the engine is: maintained at 3OC, determine the rate of heat removal from the food compartment.
A. 6KN *B. 12KN C.16KN D. 20KN *A.1504.8 kJ/min B.12.86 kJ/min C.1625 kJ/min D.9.57
Solution: kJ/min
Thrust developed = m(v2 – v1) = 20(800 – 200) = 12,000N = 12KN Solution:
A thermal power has a net power 10MW. The backwork ratio of the plant is 0.005. COP= TL /TH– TL = (3 + 273)/ (25 + 273) – (3+273) =12.54
Determine the compressor wor. QL= COP x W = 12.54 x 2(60) = 1504.8 kJ/min
A. 50.15KW B. 50.35KW *C.50.25KW D. 50.45KW A household refrigerator with EER 8.0 removes heat from the refrigerated space at a rate of
Solution: 90 kJ/min. Determine the rate of heat transfer to the kitchen air.
Wnet= WT + WP A.101.25 kJ/min B.63.05 kJ/min *C.128.46 kJ/min
BW= WP / WT D.80 kJ/min
0.005 =WP / WT Solution:
WP= 0.005WT COP= EER /3.412 = 8/ 3.412 = 2.34
Wnet= WT - WP COP= QL /QH – QL = 2.34 = 90 / QH – 90
10,000 = WT – 0.005WT QH=128.46Kj/min
WT= 10,050.25 KW An air-conditioning system is used to maintain a house at 75OF when the temperature
WC= 0.005(10,050.25) = 50.25KW outside is 95OF. The house is gaining heat through the walls and windows at a rate of 1250
A heat engine receives heat from a source at 1200oK at a rate of 500KJ/s and rejects the Btu/min, and the heat generation rate within the house from people, lights and appliances
waste heat to a sink at 300oK. If the power output of the engine is 200KW, the second law amounts to 350 Btu/min. Determine the minimum power input required for this air-
efficiency of the heat engine is: conditioning system.
A.35% B.40% *C.53% D.75% A.10.06 hp B.1.36 hp *C.1.41 hp D.7.94 hp
Solution:
ea= 200/500 = 0.40
et= (TH – TL)/TH = (1200 – 300)/1200 = 0.75 Solution:
es= 0.40/0.75= 53.33% QL= 1250 + 350 = 1600 Btu/min
COP= TL / TH – TL= (75 + 460)/(95 + 460) - (75+460) = 26.75 determine the refrigerating capacity if the refrigerating effect is 164 kJ/kg. From the table for
W= QL /COP = (1600 / 26.75) / 42.4 = 1.41 hp R22 the specific volume at the compressor entrance is 40.36L/kg.
A refrigeration system is to cool bread loaves with an average mass of 450 g from 22OC to - A.393.3 TR B.79.3 TR C.96.4 TR *D.27.4 TR
10OC at a rate of 500 loaves per hour by refrigerated air. Taking the average specific and Solution:
latent heats of bread to be 2.93 kJ/kg, OC and 109.3 kJ/kg, respectively, determine the m= V1/v1= 23.72 / 40.36 = 0.5877 kg/s
product load. Qe= m (qe) = 0.5877 (164)/ 3.52 = 27.4 TR
A.541.7 kJ/min B.351.6 Kj/min *C.761.5 kJ/min
D.409.9 kJ/min
Solution:
Mbread= (500 breads/h) (0.45 kg / bread) = 225 kg/h
Qtotal= Qbread + Qfreezing The refrigerant volume flow rate at the entrance of compressor were obtained from a test on
= (mcp∆T)breadl (mhlatent)bread = (225)(2.93)[22-(-10)] l (225)(109.3) a twin cylinder single acting 15 cm x 20 cm, 320 rpm compressor ammonia refrigerating plant
Qtotal= 45,688.5 kJ/h = 761.5 kJ/min 33 L/s. Determine the volumetric efficiency of the compressor.
A house that was heated by electric resistance heaters consumed 1200 kWh of electric A.77.65% *B.87.6% C.97.6 TR D.65.65%
energy in a winter month. If this house were heated instead by a heat pump that has an Solution:
average performance factor, PF of 2.4, determine how much money the homeowner would VD = (π D2 L/4) N= (π /4) (0.15)2(0.2) (320) (2) = 2.26 m3/min
be saved that month. Assume a price of 0.085$/kWh for electricity. nv =V1/VD = 33/2.26 (1000/60) = 0.876 or 87.6 %
A. $42.5 *B. $59.50 C.$109 D.$97.75
Solution: A twin cylinder ammonia compressor with volume displacement of 14,726 cm3 operates at
W= QH/PF = 1200kWh / 2.4 = 500 kWh 300 rpm. Condenser and evaporator pressure are 1200 kPa and 227 kPa respectively. Specific
$ Savings per month = (1200 – 500) (0.085) = $59.50 volume of refrigerant at the entrance of compressor is 528.26 L/kg. Compression process is
An ammonia simple saturation cycle operates with a suction pressure of 291.6 kPa and a polytrophic with a 1.20 and clearance factor of compressor is 2%. Determine horsepower
condenser pressure of 1204 kPa develops 15 tons of refrigeration. Determine the theoretical required.
horsepower of the compressor. A.60hp B.70hp C.80hp D.90hp
The following enthalpies have been found: condenser entrance = 1653 kJ/kg, exit Solution:
=346.6 kJ/kg, compressor entrance = 1450.2 kJ/kg, exit= 1653kJ/kg. nv= 1+c-c(p2/p1)1/n= 1 + 0.02 – 0.02(1200/227)1.20= 0.8725
A.7.23 hp *B.13 hp C. 15 hp D.8.23 VD= (π D2 L/4) N= (0.014726) (320) (2) = 9.424 m3/min
hp V1= (nv) VD- (0.8725) (9.424) - 8.2224 m3/min
Solution: W= (n p1 V1 /1-n) [(p2/p1) n-1/n-1]
m= Qe/ (h1 – h4) = (15 x 3.52) / (1450.2 – 346.6) = 0.0478 kg/s = [(1.20 x 2.27 x 8.2224)/ (1-1.20)] [(1200/2.27)1.2-1/1.2 -1] = 3582 kJ/min
W= m (h2 – h1) = (0.0478)(1653 – 1450.2) / 0.746 = 13 hp W= 3582/ (60) (0.746) = 80 hp
An ammonia ice plant operates between a condenser temperature of 35OC and evaporator
of -15OC. It produces 10 metric tons of ice per day from water at 30OC to ice at -5OC. A reversed Carnot cycle has a refrigerating COP of 2.5. Determine the ratio TH/TL?
Assuming simple saturation cycle, determine the horsepower of the motor if the adiabatic A.1.4 B.1.5 C.1.25 D.1.2
efficiency of the compressor ηc=0.85 and mechanical efficiency ηm=0.95. The specific heat of Solution:
ice is 2,094kJ/kg. OC and the latent heat is 335kJ/kg. COP= TH/ (TH-TL)
From the table for ammonia the following enthalpies are: condenser entrance = 1703 kJ/kg, 1/COP = (TH-TL)/TL= TH/TL-1
exit= 366.1 kJ/kg; compressor entrance= 1443.9kJ/kg, exit = 1703kJ/kg TH/TL =1 + 1/COP =1 +1/2.5 =1.4
A.17.68 hp B.18.61 hp C.15.5 hp *D.21.9 hp
Solution: Three thousand cubic feet per minute of air are circulated over an air-cooled condenser. If
qe=cpa (te – tf) lhlatentlcph(tf – ts) the load on the condenser is 64,800 Btu/hr, compute the temperature rise of the air passing
= (4.187) (30 – 0) + 335 + (2.094) [(0- (-5)] = 471.08 kJ/kg over the condenser. Specific volume of standard air (13.34ft3/lb)
Qe= (10 x 1000) (471.08) / 24 = 196,283.33 kJ/hr= 54.523kJ/s A.10OF B.15OF C.20OF D.25OF
m= Qe / (h2 – h4) = (54.523)/ (1443.9 – 366.1) = 0.05059kg/s Solution:
W= m (h2 – h1) = (0.0509) (1703 – 1443.9)/ 0.746 = 17.68 hp Qc= m c ∆t
Wmotor= 17.68 / (0.85) (0.95) =21.9 hp ∆t = Qc/ m c = 64,800/ [3000 (60) / (13.34)] (0.24) = 20OF
A Freon 22 air conditioning under standard operating conditions of 35OC is condensing and
5OC evaporating temperatures. The volume flow rate entering the compressor is 23.72 L/s. Saturated vapor ammonia at -16OC (h1 = 1442.60 kJ/kg) leaves the evaporator and enters
the compressor at -6OC (h1 = 1465kJ/kg. The refrigerant leaves the condenser as saturated
liquid at 40OC (h4= 390.6 kJ/kg) and enter the expansion valve at 35OC (h5 =366.1kJ/kg). A.2.027 B.5.22TR C.3.314TR D.9.1TR
Heat rejected from the compressor amount to 50kW. The work to compressor is 208kJ/kg, Solution:
while the heat loss from compressor is 33kJ/kg. If 95kJ/kg of heat are lost in the piping Wtotal= 10.12 + 15.93 = 26.05 kW
between the compressor discharge and condenser inlet, determine the refrigerating capacity COP = 15 (3.52) /26.05 = 2.027
of the system.
A.49.5 TR B.46.61 TR C.12.88 TR D.13.24 TR When a man returns to his well-sealed house on a summer day, he finds that the house is at
Solution: 32OC. He turns on the air conditioner, which cools the entire house to 20OC in 15min. If the
Solving for the enthalpy at the exit of compressor using energy balance about the COP of the air-conditioner system is 2.5, determine the power drawn by the air conditioners.
compressor Assume the entire mass within the house is equivalent to 800 kg of air for which c= 0.72kJ/kg
h2 = h1 + w - qwc = 1465 + 208 – 33= 1640 kJ/kg OC
Solving for the enthalpy at the entrance of condenser using energy balance about A.7.68Kw B.3.07kW C.19.2kW D.12.03kW
piping from compressor exit to condenser entrance Solution:
h3= h2 – q2-3= 1640 – 95 = 1545kJ/kg Qe = m c (t1 – t2) / ∆time = (800) (0.72) (32 – 20) / (15 x 60) = 7.68kW
Solving for heat rejected in condenser using energy balance about the condenser W= Qe / COP = 7.68 / 2.5 = 3.07kW
qc = h3 – h4 = 1545 – 390.6 = 1154.4kJ/kg
m = Qc/qc = 50/1154.4 = 0.0433kg/s It is desired to double the COP of a reversed Carnot engine for cooling from 5.0 by raising the
Solving for refrigerating effect using energy balance about the evaporator h5 – h4 = temperature of the heat addition while keeping the temperature of the heat rejection
366.1kJ/kg constant. By what percentage must the temperature of heat addition be raised?
A.10.1% B.9.1% C.8.1% D.7.1%
qe= h1 – h5 = 1442.6 – 366.1 = 1076.5kJ/kg Solution:
Therefore, the refrigerating capacity Percent increase of the temperature = (TL – TL) TL = TL / TL – 1 of heat addition
Qe= m qe = [(0.0433) (1076.5)] / 3.52 = 13.24 TR Original COP: 5 = TL / (TH – TL); (TH – TL) TL = 1/5; TH / TL -1 = 1/5 = 02; TH / TL = 1.2
eq.1
An actual refrigerating cycle using R12 as working fluid, the refrigerant flow rate is 0.05 kg/s. Doubling the COP:
Vapor enters the expansion valve at 1.15 MPa, 40OC (h = 238.5kJ/kg) and leaves the 10 = TL / (TH – TL); (TH – TL) TL = 1/10; TH / TL – 1 =1/10 = 0.1; TH/TL=1.1
evaporator at 175 kPa, -15OC (h = 345kJ/kg). The electric input to motor driving the Dividing eq.1 and eq.2:
compressor is measured and found 3.0 kW. Motor efficiency at this load is 92% and (TH/TL) (TH/TL) = 1.2 / 1.1 = 1.091
mechanical efficiency 82%. Determine the actual coefficient of performance for this cycle. Percent Increase of heat addition = 1.091 – 1 = 0.091 or 9.1%
A.1.58 B.2.36 C.1.78 D.1.34 An ammonia water-cooled compressor receives the refrigerant at specific volume 62 L/kg. It
Solution: has a piston displacement rate of 3m3/min. If a squirrel cage motor is running at 1200 rpm
Qe= (0.05) (345 - 238.5) = 5.325kW drives the compressor and average piston speed is 490m/min, calculate size of cylinder bore.
W = (3) (0.92) (0.82) = 2.26kW A.20.4 cm B.26.0 cm C.16.13 cm D.13.6 cm
COPactual= 5.325 / 2.26 = 2.36 Solution:
Piston speed = 2LN = 490 = 2(L) (1200); L= 0.204 m = 20.4 cm
An ammonia refrigeration system the temperature in the evaporator is -12OC and the Vp = (π D2 L/4) N = (π/4) D2 (0.204) (1200) =5
ammonia at the evaporator entry is 0.1511 dry while at exit is 0.95 dry. If the rate of D = 0.1613m = 16.13cm
ammonia circulation is 5.64 kg/min, determine the refrigerating capacity of the system. If the initial volume of an ideal gas is compressed to one-half its original volume and to twice
Enthalpy of saturated liquid and vapor at -12OC is 144.929kJ/kg and 1447.74kJ/kg its temperature, the pressure:
respectively. A. Doubles B. Quadruples C. Remains constant D. Halves
A.17.82 B.34.82TR C.27.82TR D.4.82TR Solution:
Solution: P1V1/T1 = P2V2/T2
h4= hf4 + x4 (hg4 – hf4) = 144.929 + (0.1511) (1447.74 – 144.929) = 341.78 P1V1/T1= P2 (1/2 V1) / (2T1)
h1= hf1 + x1 (hg1 – hf1) = 144.929 + (0.95) (1447.74 – 144.929) = 1382.6kJ/kg P2 = 4 P1
Qe = m (h1 – h4) = (5.64) (1382.6 – 341.78) / 211 = 27.82 TR If the gage pressure of a medium is 30kPa (vacuum) and the atmospheric pressure is
101.3kPa, the absolute pressure will be:
A two stage cascade vapor compression refrigeration system uses ammonia in the low- A.131.3kPa B.-71.3kPa C.71.3kPa D.-131.3kPa
temperature loop and R-12 in the high-temperature loop. The ammonia provides 15 tons Solution:
cooling. If the high-loop temperature requires 10.12 kW compressor power and low-loop Pabs = Patm - Pvac
15.93 kW, determine the COP of the system. = 101.3kPa - 30kPa = 71.3kPa
If a particle has a velocity of 4 meters per second and a kinetic energy of 144 Joules, then the P= 750 mm Hg (101.325 kPa/ 760 mm Hg)
mass, in kilograms of this particle must be: = 99.99 kPa
A.44 B.16 C.18 D.24 A double purpose tank 18 ft wide, 24 ft long and 4 ft depth is filled with water. What is the
Solution: weight of water in the tank in long tons?
KE = (1/2) (m) (V2) / 2k A.49tons B.48tons C.54tons D.50tons
144 – (1/2) (m) (4)2 / [(2) (1)] Solution:
m = 18kg W = (62.4 lb/ft2) [(18) (24)(4)] ft3 = 107,827.2 lbf
A condenser vacuum gauge reads 715 mm Hg when the barometer stands at 757 mm Hg. = 107,827.2 lb (1ton/ 2200 lb) = 49tons
State the absolute pressure in the condenser in kN/m or kPa.
A.5.6kPa B.5.9kPa C.6.5kPa D.5.2kPa Oil flow through a 16 tubes on a single cooler with a velocity of 2 m/s, the internal diameter
Solution: of the tube is 30mm and oil density is 0.85gm/ml. Find the volume flow in liters per sec.
Pabs = Patm – Pvac = 757 – 715 = 42 mm Hg (101.325kPa/760 mm Hg) = 5.60kPa A.22.62 B.32.22 C. 62.22 D.42.62
Determine the force in Newton in a piston of 465 mm2 area with a pressure of 0.172MPa. Solution:
A.65N B.72N C.80N D.111 Volume flow rate = (3.1416) (0.015)2 (2) (16) = 0.02262 m3/s or 22.62 liters/s
Solution:
F= PA – (0.172MPa) (105 Pa/MPa) (465 mm2) (m2 / 105 mm2) A substance temperature was 620OR. What is the temperature in OC?
F = 79.98N A.50.7 B.45.54 C.71.11 D.94.44
One piston of a hydraulic press has an area of 1cm2. The other piston has an area of 25cm2. Solution:
If a force of 130N is applied on the smaller piston, what will be the total force on the larger T, OC = [(620 – 460) – 32] (5/9) = 71.11
piston is both piston surfaces are the same level?
A.6N B.175N C.3750N D.4250N Unknown volume of container gas of gas of 1 atmosphere is allowed to expand to another
Solution: container of 10 m3 volume at 500 mm Hg at constant temperature. Find the unknown
F1/A1 = F2/A2 volume.
(150/1) = (F2/ 25) A.6.58 m3 B.6.75 m3 C.5.67 m3 D.7.65 m3
F2 = 3750N Solution:
If the pressure of a confined gas at a constant temperature is tripled, what will happen to the P1V1 – P2V2
volume? (760) V1 = (500) (10)
A. The volume will be tripled B. The volume will V1= 6.58 m3
remain
C. The volume will be reduced to one-third of its original value D. The volume is An iron block weighs 5 Newton and has volume of 200 cm3. What is the density of the block?
constant A. 2458kg/m3 B. 2485 kg/m3 C. 2584 kg/m3 D. 2549 kg/m3
Solution: Solution:
P1V1 = P2V2 Density = specific weight [at sea level or near the surface of the earth]
P1V1 = (3P1) V2 = (5N/200 cm) (10 cm3/m3) (1kg/9.8066N) = 2549.30 kg/m3
V2 = (1/3) V1
If air is at a pressure of 22.22 psia and at temperature of 800 OR, what is the specific volume?
The work done on air is 10.86kJ/kg, determine the compressor power if it is receiving 272 A.11.3 ft3/lbm B.33.1 ft3/lbm C.13.3 ft3/lbm D.31.3 ft3/lbm
kg/min if air. Solution:
A.36.72 hp B.49.23 hp C. 29.54 hp D.66 hp Pv = RT
Solution: v = (53.34) (800) / [(22.22) (144)] = 13.33 ft3/lbm
W= (10.86) (272.60) = 49.232 kJ/s or kW
= 49.232 kW (1 hp/ 0.746kW) = 65.99 hp The specific gravity of mercury is 13.55. What is the specific weight of mercury?
A water tank of 18 ft long and 4 ft high, calculate the pressure at the bottom of the tank. A.123.9 kN/m3 B.139.2 kN/m3 C.139.9 kN/m3
A.1.733psi B.1.999psi C.2.337psi D.3.773psi D.193.2 kN/m3
Solution: Solution:
P= (62.4 lbf/ft3) (4ft) (1ft2 / 144 in2) = 1.733psi ? = (13.55) (9.8066) = 132.88 kN/m3
The pressure of 750 mm Hg in kN/m2 The equivalent weight of mass 10 kg at a location where the acceleration of gravity is 9.77
A.90 B.100 C.103 D110 m/sec2
Solution:
A. 97.7N B.79.7N C.77.9N Solution:
D.977N m = rv/RT = [(551.43) (600)/ (102)] / [(0.18896) (20+273)] = 0.00598 kg or 5.98 g
Solution:
Weight = mg/k = [(10) (9.77)] /1 =97.7N Is the most common dryer used which consist of rotating cylinder inside which the materials
A transportation company specializes in the shipment of pressurized gaseous materials. An flow while getting in contact with hot gas.
order is received from 100 liters of a particular gas at STP (32OF and 1 atm). What minimum A. Tower dryer B. Centrifugal dryer C. Tray dryer D.
volume tank is necessary to transport the gas at 80OF and maximum pressure of 8 atm? Rotary dryer
A.16 liters B.14 liters C.10 liters D.12 liters ANSWER: D
Solution: Is the ratio of the mass water-vapor in air and the mass of air if it is saturated is called:
P1V1/ T1 = P2V2/ T2 A. Humidity ratio B. Mass ratio C. Vapor ratio D. Relative humidity
[(1) (100)/ (32 + 460)] / [(8) (V2)/ (80 + 460) ANSWER: D
V2 = 13.72 liters The hands feel painfully cold when the skin temperature reaches:
100 g of water are mixed with 150 g of alcohol (density = 790 kg/m3). What is the specific A.8OC B.10OC C.12OC D.14OC
volume of the resulting mixtures, assuming that the fluids mixed completely? ANSWER: B
A.0.82x10-3 cu.m/kg B.0.88x10-3 cu.m/kg C.0.63x10-3 cu.m/kg D.1.16x10-3 cu.m/kg The refrigerant used in steam jet cooling is:
Solution: A. Steam B.R-11 C. Ammonia D. Water
Mass of mixtures = 100 + 150 =250 g ANSWER: D
Volume of mixture = [(0.100) / (1000)] + [(0.150)/ (790)] = 0.00029 m3 The total heat of the air is a function of
Specific volume of mixture = (0.00029)/ (0.250) = 1.16x10-3 cu.m/kg A. WB temperature B.DP temperature C.DB temperature D.WB
How much does 30 lbm weigh on the moon? (gmoon = 5.47 ft/s2 depression
A.20 lbr B.3.2 lbr C.3.4 lbr ANSWER: A
D.5.096 lbr Boiling point of Freon-12 at atmospheric pressure is:
Solution: A. 21OF B.15OF C.5OF D.28OF
Weight = mg/k = {[(30) (5.42)]/32.174} = 5.1 lbf ANSWER: A
A 10 kg block is raised vertically 3 meters. What is the change in potential energy? Which of the following is NOT a type of water cooled condenser in refrigeration?
A.320 J B.350 kg-m2/s2 C.294 J D.350 A. Double pipe B. Double shell C. Shell and coil D. Shell and tube
N-m ANSWER: B
Solution: Component of absorption refrigeration system in which the solution is cooled by cooling
PE = mgs/k = {[(10) (9.8066) (3)]/1} = 294.2 J water:
How many cubic meters is 100 gallons of liquid? A. Rectifier B. Generator C. Evaporator D. Absorber
A.3.7850 cu.m B.0.1638 cu.m C.0.3785 cu.m D.1.638 cu.m ANSWER: D
Solution: Cascade refrigeration cycle is often used in industrial process where objects must be cooled
100gal (785 liters/gal) (1m3/1000 liters) = 0.3785 m3 to temperature below:
Steam turbine is receiving 1014 lbm/hr of steam, determine the horsepower output of the A. -46OC B. -56OC C. -66OC D. -76OC
turbine if the work done by steam is 251 Btu/lbm ANSWER: A
A.100 hp B.462.7 hp C.200 hp D.6002.7 hp Type of refrigerant control designed to maintain a pressure difference while the compressor
Solution: is operating.
W= (251 Btu/lbm) (1014 lbm/hr) 1hp/2545 Btu/hr) = 100 hp A. Thermostatic expansion valve B. Automatic expansion valve C. Using low side float
What is the resulting pressure when one pound of air at 15 psia and 200OF is heated at flooded system D. Capillary tube
constant volume to 800OF? ANSWER: D
A.52.1 psia B.15 psia C.28.6 psia D.36.4 psia As a rule of thumb, for a specified amount of compressed air, the power consumption of the
Solution: compressor decreased by ______ for each 3OC drop in the temperature inlet air to the
P1/T1 = P2/T2 compressor.
P2 = [(800+460) / (200+460)] (15) = 28.64 psia A. 1% B. 1.5% C. 2% D. 2.5%
A bicycle tire has a volume of 600 cm3. It is inflated with carbon dioxide to pressure of ANSWER: A
551.43kPa at 20OC. How many grams of CO2 are contained in the tire? Rcoz = 0.18896 Modern way of detecting air compressor leak is by using
kJ/kg.K A. Soup and water B. Air leak detector C. Acoustic leak detector D. Ammonia leak
A.5.98 g B.6.43 g C.4.63 g detector
D.3.83 g ANSWER: C
For foundation of stacks, the maximum pressure on the soil is equal to the pressure due to A type of geothermal plant used when there is a presence of brine extracted from underground
the weight and the . A. Dry geothermal plant B. Double-flesh geothermal plant C.Single-flash geothermal
A. Soil movement B. Wind movement C. Ground movement D. Engine plant
movement D. Binary geothermal plant
ANSWER: B ANSWER D
Foundation bolts of specified size should be used and surrounded by a pipe sleeve with an Is the most important safety device on the power boiler
inside diameter of at least A. Check valve
A. 3 times the diameter of engine bolt B. 3 times the diameter of anchor bolt B. Gate valve
C. 2 times the diameter of engine bolt D. 2 times the diameter of anchor bolt C. Safety valve
ANSWER: B D. Globe valve
For multi stage compression of an ideal Brayton cycle, the backwork ratio will ANSWER C
A. Increase B. Decrease C. Remain the same D. none of During hydrostatic test, the safety valves should be
these A. Removed
ANSWER: B B. Open
Type of turbine that has a specific speed below 5
C. Closed
A. Impulse turbine B. Propeller turbine C. Francis turbine D. Deriaz
D. Partially closed
turbine
ANSWER A
ANSWER: A
Where deaerating heaters are not employed, it is recommended that the temperature of the
A high discharge type of turbine
feed less than
A. Impulse turbine B. Francis turbine C. Propeller turbine D.
A. 197°C
Deriaz turbine
B. 102°C
ANSWER: C
C. 104°C
Use to minimize the speed rise due to a sudden load rejection
A. needle valve B. wicket gate C. Shut-off valve D. jet D. 106°C
deflector ANSWER A
ANSWER: D Is a reaction during which chemical energy is released in the form of heat?
Is the speed of a turbine when the head on the turbine is one meter? A. Cosmic reaction. B.Ethnic reaction C.Endothermic reaction D.Exothermic
A. specific speed B. rated speed C. utilized speed D. unit reaction
speed ANSWER D
ANSWER: D By reheating the steam in an ideal Rankine cycle the heat rejected will:
A. Increase B. Decrease C. Remains the same D. None of these above
Is a fluid property which refers to the intermolecular attraction by which the separate ANSWER A
particles of the fluid arc held together? By increasing the boiler pressure in Rankine cycle the moisture content at boiler exit will:
A. Cohesion B. Adhesion C. Surface tension D. A. Increase
hypertension B. Decrease
ANSWER: A C. Remains the same
D. None of these above
Which of the following is NOT the cause of black smoke in diesel engine? ANSWER A
A. fuel valve open too long B. high compression pressure C. carbon in exhaust pipe Presently the highest steam temperature allowed at the turbine inlet is about
D. overload on engine A. 40°C
B.5 20°C
Which of the following is not a method of starting a diesel engine? C.6 20°C
A. Manual rope, crank and kick B.Electric (battery) C. Compressed air D. Using D. 020°C
another generator ANSWER C
ANSWER D The most common gases employed in Stirling and Ericsson cycles are:
Two-stroke engine performs to complete one cycle A. Air and helium B. Oxygen and helium C. Hydrogen and helium D. Nitrogen
A. Suction and discharge stroke B. Power and exhaust stroke C. Compression and power
and helium
stroke
D. Suction and exhaust stroke ANSWER C
ANSWER C In most common designs of Gas turbine, the pressure ratio ranges from
A. 10 to 12 Refrigerants consisting of mixtures of two or more different chemical compounds, often used
B. 11 to 16 individually as refrigerants for other applications
C. 12 to 18 A. Suspension
D. 15 to 20 B. Compound reaction
ANSWER B C. Blends
In Brayton cycle, the heat is transformed during what process? D. Mixing of refrigerants
ANSWER C
A. constant temperature Pairs of mating stop valves that allow sections of a system to be joined before opening these
B. sentropic process valves or separated after closing them
A. Check valve
C .isobaric process
B. Gate valve
D.isochoric process
C. Safety valve
ANSWER C
The fuel injection process in diesel engine starts when piston D. Companion valve
A. Is at the TDC ANSWER D
B. Leaving TDC An enclosed passageway that limits travel to a single path
A. Corridor
C. Approaches TDC
B. Hallway
D. Halfway of the stroke
C. Lobby
ANSWER C
If the cut-off ratio of diesel cycle increases, the cycle efficiency will D. Tunnel
A. Decrease ANSWER A
B. Increase For Immediate Dangerous to Life or Health (IDHL) the maximum concentration from which
unprotected persons have time to escape within without escape-impairing symptoms or
C. Remains the same
irreversible health
D. None of these A. 16 minutes
ANSWER A B. 1 minute
The fuel used in a power plant that is used during peak periods
C. 20 minutes
A. Gas
D. 30 minutes
B. Solid
ANSWER D
C. Liquid
The volume as determined from internal dimensions of the container with no allowance for the
D. None of these volume of internal parts
ANSWER C A. Internal allowance volume
Typical compression ratio of Otto cycle is B. Internal gross volume
A. 6
C. Internal interference volume
B. 8
D. Internal fots volume
C. 10
ANSWER B
D. 12 A waiting room or large hallway serving as a waiting room
ANSWER B A. Terrace
If Joule Thompson coefficient is equal to zero, then the process will become B. Test room
A. Isentropic
C. Compound room
B. Isenthalpic
D. Lobby
C. Isobaric
ANSWER D
D. Isothermal A continuous and unobstructed path of travel from any in a building or structure to a public
ANSWER D way
If the fluid passed through a nozzle its entropy will: A. Average of aggress
A. Increase B. Mean o aggress
B. Decrease C. Hallway of aggress
C. Remains the same D. Pathway of aggress
D. None of these ANSWER B
ANSWER C
Any device or portion of the equipment used to increase refrigerant pressure B. Indirect closed system
A. Pressure relief device C. Indirect open spray system
B. Pressure imposing element D. Indirect vented closed system
C. Pressure lift device ANSWER B
D. Pressure limiting device Refrigerant number R-744 is:
ANSWER B A. Butane
The quantity of refrigerant stored at some point is the refrigeration system for operational, B. Carbon monoxide
service, or standby purposes C. Propane
A. Pressure vessel D. Carbon dioxide
B. Pumpdown charge ANSWER D
C. Liquid receiver Refrigerant number R-1150 is:
D. Accumulator A. Propylene
ANSWER B B. Ethane
Secondary refrigerant is a liquid used for the transmission of heat, without a change of state, C. Ethane
and having no flash point or a flash point above as determined from ASTM D. Methyl formale
A. 150°F B. 160°F C. 180°F D. 200°F
ANSWER B
ANSWER A
Refrigerant number R-40 is:
A service valve for dual pressure-relief devices that allows using one device while isolating
A. Chlorodifluoromethane
the other from the system maintaining one valve in operation at all times
A. Three-way valve B. Difluoromethane
B. Two-way valve C. Ammonia
C. One-way valve D. Chloromethane
D. Four-way valve ANSWER D
When the air duct system serves several enclosed spaces, the permissible quantity of
ANSWER A
refrigerant in the system shall not exceed the amount determined by using the total volume of
Tubing that is unenclosed and therefore exposed to crhsing, abrasion, puncture, or similar
those spaces in which the airflow cannot be reduced to less than of its maximum
damage after installation
when the fan is operating
A. Protected tubing
A. One-quarter
B. Bare tubing
B. One half-quarter
C. Open tubing
C. Three-quarter
D. Unprotected tubing
D. One fourth-quarter
ANSWER D
ANSWER A
Refers to blends compromising multiple components of different volatile that, when used in
The space above a suspended ceiling shall not be include 1 in calculating the permissible
refrigeration cycles, change volumetric composition and saturation temperature as they
quantity of refrigerant in the system unless such space is continuous and is part of the air
evaporate (boil) or condense at constant pressure
return system
A. Zeoline
A. Partition
B. Blending
B. Plenum
C. Composition
C. Separator
D. Zertropic
D. Plate divider
ANSWER D
ANSWER B
Is a premises or that portion of a premise from which, because they are disabled, debilitated,
Which of the following is NOT a possible location of service valve?
or confined, cooperate cannot readily leave without the assistance of others
A. Suction of compressor
A. Institutional occupancy
B. Discharge of compressor
B. Public assembly occupancy
C. Outlet of liquid receiver
C. Residential occupancy
D. Outlet of condenser
D. Commercial occupancy
ANSWER D
ANSWER A
A cool to serve with evaporator that is use to prevent the liquid refrigerant entering the
Is one in which a secondary coolant is in direct contact with the air or other substance to be
compressore
cooled or heated
A. Accumulator
A. Double indirect open spray system
B. Liquid superheater Superheating the refrigerant in refrigeration system without useful cooling, the refrigeration
C. Drier loop effect per unit mass will
D. Liquid suction heat exchanger A. Increase
ANSWER C B. Decrease
A type of valve connected from discharge of compressore directly to suction that is normally C. Remains the same
closed and will open automatically only if there is high discharge pressure D. None of these
A. Check valve ANSWER C
B. Solenoid valve Which of the following is NOT a type of air-cooled condenser?
C. King valve A. Shell and tube
D. Relief valve B. Natural draft
ANSWER B C. Forced draft
Use to increase the capacity of condenser D. Induced draft
A. Water regulating valve ANSWER A
B. Desuperheating coils A type of refrigerant control typically used in household refrigeration
C. Liquid-suction heat exchanger A. Thermostatic expansion valve
D. Condenser heating coils B. Automatic expansion valve
ANSWER B C. Capillary tube
Is use to subcooled the refrigerant from the condenser D. High side float
A. Liquid subcooler ANSWER C
B. Condenser subcooler Type of condenser that operates like cooling tower
C. Desuperheating coils A. Air-cooled condenser
D. Liquid receiver B. Evaporative condenser
ANSWER A C. Shell and tube condenser
Which of the following is NOT a part of low pressure side in refrigeration system? D. Water-cooled condenser
A. Liquid line ANSWER B
B. Refrigerant flow control The major problem of heat pump is
C. Evaporator A. Refrigerant used
D. Suction line B. Outside air
ANSWER A C. Supply air
Which of the following is NOT a part of high pressure side in refrigeration system? D. Frosting
A. Compressor ANSWER D
B. Condenser Dominant refrigerant used in commercial refrigeration system
C. Liquid line A. R-11
D. Suction line B. R-22
ANSWER D C. R-12
Which of the following is NOT a part of condensing unit? D. R-502
A. Compressor ANSWER D
B. Discharge line Cascade refrigeration system are connected in
C. Condenser A. Series
D. Liquid line B. Parallel
ANSWER D C. Series-parallel
By subcooling the refrigerant in refrigerant system, the compressor power per unit mass will D. Parallel-series
A. Increase ANSWER A
B. Decrease Is use to heat up the solution partially before entering the generator in absorption refrigeration
C. Remains the same system
D. None of these A. Rectifier
ANSWER C B. Absorber
C. Regenerator
D. Pump ANSWER C
ANSWER C Are suitable for use in ammonia system
The COP of actual absorption refrigeration system is usually A. Copper
A. Less than 1 B. Aluminum and its alloy
B. Less than 2 C. Plastic
C. Less than 3 D. Cast iron
D. Less than 4 ANSWER B
ANSWER A If a pressure-relief device is used to protect a pressure vessel having an inside dimension of 6
Sight glass is often located at: in or less, the ultimate strength of the pressure vessel so protected shall be sufficient to
A. Discharge line withstand a pressure at least the design pressure
B. Liquid line A. 2 times
C. Between condenser and liquid receiver B. 3 times
D. Suction line C. 4 times
ANSWER B D. 5 times
Use to detects a vibration in current caused by the ionization of decomposed refrigerant ANSWER B
between two opposite-charged platinum electrodes Seats and discs shall be limited in distortion, by pressure or other cause, to a set pressure
A. Electronic detector change of not more than in a span of five years
B. Halide torch A. 1%
C. Bubble method B. 5%
D. Pressurizing C. 10%
ANSWER B D. 50%
Joints and all refrigerants containing parts of a refrigerating system located in an air duct ANSWER B
carrying conditioned air to and from an occupied space shall be constructed to withstand a Liquid receivers, if used, or parts of a system designed to receive the refrigerant charge during
temperature of without leakage into the airstream pumpdown charge. The liquid shall not occupy more than of the volume when
A. 550°F temperature of the refrigerant is 90°F
B. 600°F A. 80%
C. 650°F B. 85%
D. 700°F C. 90%
ANSWER D
Refrigerant piping crossing an open space that affords passageway in any building shall be D. 95%
less than_________ above the floor unless the piping is located against the ceiling of such ANSWER C
space and is permitted by the authority having jurisdiction
A. 2.2 m The discharge line (B4) shall be vented to the atmosphere through a fitted to its upper
B. 3.2 m extremity
C. 4.2 m
A. Nozzle
D. 5.2 m
ANSWER A B. Convergent-divergent nozzle
Methyl chloride shall not be in contact with C. Pipe
A. Aluminum
D. Diffuser
B. Zinc
C. Magnesium ANSWER D
D. All of these Convert fossil fuels into shaft work
ANSWER D
A. Nuclear power plant
Shall not be in contact with any halogenated refrigerants
A. Aluminum B. Gas turbine power plant
B. Zinc C. Dendrothermal power plant
C. Magnesium
D. All of these D. Thermal power plant
ANSWER D D. Regenerative heater
Ultimate strength drops by 30% as steam temperature raises from for unalloyed steel ANSWER A
A. 300 to 400°C Dissolve gases like makes water corrosive react with metal to form iron oxide
B. 400 to 500°C A. O2 and N2
C. 600 to 700°C B. O2 and CO
D. 700 to 800 °C C. O2 and CO2
ANSWER B D. N2 and SO2
Recent practice limits steam temperature to ANSWER C
A. 438°C A cycle typically used in paper mills, textile mills, chemical factories, sugar factories and rice
B. 538°C mills
C. 638°C A. Cogeneration cycle
D. 738°C B. Combined cycle
ANSWER A C. By-product cycle
In a closed feed water heater, the feed water pass through D. Cascading cycle
A. Inside the tube ANSWER A
B. Outside the tube When process steam is basic need and power is byproduct, this cycle is known as
C. Inside the shell A. Cogeneration cycle
D. Outside the shell B. Combined cycle
ANSWER A C. By-product cycle
In use if extracted steam upon condensation gets subcooled D. Cascading cycle
A. Trap ANSWER C
B. Deaerator A type of turbine employed where steam continuously extracted for process heating
C. Filter A. Back pressure turbine
D. Drain cooler B. Gas turbine
ANSWER D C. Steam turbine
Needs only single pump regardless of number of heaters D. Passout turbine
A. Open heater ANSWER D
B. Closed heater Which of the following is used for Binary cycle power generation for high temperature
C. Mono heater application
D. Regenerative heater A. Mercury
ANSWER B B. Sodium
Is also known as deaerator C. Potassium
A. Open heater D. All of these
B. Closed heater ANSWER D
C. Reheat heater critical temperature of mercury is
A. 1160°C Which of the following helps in the ignition of coal?
B. 1260°C A. Moisture
C. 1360°C B. Ash
D. 1460°C C. Fixed carbon
ANSWER D D. Volatile matter
Critical pressure of mercury is ANSWER D
A. 100 Mpa is the ratio of fixed carbon and volatile matter
B. 108 Mpa A. Air-fuel ratio
C. 128 Mpa B. Fuel ratio
D. 158 Mpa C. Combustion ratio
ANSWER B D. Carbon-volatile ratio
Method used in converting heat directly to electricity by magnetism ANSWER B
A. Electronic induction A suspension of a finely divide fluid in another
B. Magnetodynamic A. Filtration
C. Magnetohyrdodynamic B. Floatation
D. Thermoelectric C. Emulsion
ANSWER C D. Separation
Which of the following is NOT a material used for thermoelectric elements ANSWER C
A. Bismuth telluride Contains 90% gasoline and 10% ethanol
B. Lead telluride A. Gasohol
C. Zinc telluride B. Gasonol
D. Germanium C. Gasothanol
ANSWER C D. Gasethanol
A type of coal formed after anthracite ANSWER A
A. Lignite Process used commercially in coal liquefaction
B. Bituminous A. Tropsch process
C. pear B. Fisher process
D. graphite C. Fisher-Tropsch process
ANSWER D D. Mitch-Tropsch process
Which of the following is lowest grade of coal? ANSWER C
A. Peat Is an organic matter produced by plants in both land and water
B. Lignite A. Bio-ethanol
C. Sub-bituminous B. Biomass
D. Bituminous C. Petroleum
ANSWER B D. Biodegradable
ANSWER B D. Magnesium hydroxide
In thermal power plant, induced draft fans are located at the ANSWER D
A. Exit of furnace Presence of excess hydrogen ions makes the water
B. Foot if the stack A. Acidic
C. Above the stock B. Alkalinity
D. Top of the stock C. Base
ANSWER B D. Hydroxicity
In thermal power plant, forced draft fans are installed at the ANSWER A
A. Foot of the stack PH of water varies with
B. Top of the stack A. Pressure
C. Exit of the preheater B. Temperature
D. Inlet of the preheater C. Density
ANSWER D D. Volume
Known as drum less boiler ANSWER B
A. La Mont boilers Ph value of is usually maintained for boiler water to minimized corrosion
B. Fire tube boiler A. 8.5
C. Forced circulation boiler B. 9.5
D. Once-through boiler C. 10.5
ANSWER D D. 11.5
Reduces the steam temperature by spraying low temperature water from boiler drum ANSWER C
A. Reheater
What type of turbine that has a degree of reaction of ?
B. Preheater
A. Impulsive turbine
C. Desuperheater
B. Reaction turbine
D. Superheater
C. Rarsons turbine
ANSWER C
D. Deriaz turbine
Carbon dioxide can be removed by:
ANSWER C
A. Deaeration
The cooling water is made to fall in series of baffles to expose large surface area for steam fed
B. Aeration
from below to come in direct contact
C. Evaporation
A. Spray condenser
D. Vaporization
B. Surface condenser
ANSWER B
C. Jet condenser
Is often used to absorb silica from water
D. Barometric condenser
A. Sorbent
ANSWER D
B. Rectifier
Show the variation of river flow (discharge) with time
C. Silica gel
A. Hydrograph
B. Hyetograph A. Thermosiphon cooling
C. Mass curve B. Thermostat cooling
D. Flow duration curve C. Pressurized water cooling
ANSWER A D. Evaporating cooling
Is an open channel erected on surface above the ground ANSWER A
A. Canal Type of lubrication system in diesel engine in which oil from pump is carried to a separate
B. Tunnel storage tank outside the engine cylinder and used for high capacity engine.
C. Penstock A. Mist lubrication system
D. Flume B. Wet pump lubrication system
ANSWER D C. Splash system
Type of turbine used up to 300 m head D. Dry sump lubrication system
A. Impulsive turbine ANSWER D
B. Francis turbine Produces extreme pressure differentials and violent gas vibration
C. Propeller turbine A. Vibration
D. Deriaz turbine B. Detonation
ANSWER D C. Explosion
A turbine that has a diagonal flow D. Knocking
A. Impulsive turbine ANSWER B
B. Francis turbine In a spark ignition engine, the detonation occurs near the
C. Propeller turbine A. End of combustion
D. Deriaz turbine B. Middle of combustion
C. Beginning of combustion
ANSWER D
D. Beginning of interaction
Oil is optimized either by air blast or pressure jet at about ANSWER A
A. 60 bar In a compression ignition engine, the detonation occurs near the
A. End of combustion
B. 70 bar B. Middle of combustion
C. 80 bar C. Beginning of combustion
D. Beginning of interation
D. 90 bar
ANSWER C
ANSWER B Morse test is use to measure the of multi-cylinder engine
Type of solid injection that use single pump supplies fuel under high pressure to a fuel header A. Brake power
B. Indicated power
A. Common rail injection
C. Friction power
B. Individual pump injection system D. Motor power
C. Distributor system ANSWER B
Ignition delay can be minimized by adding to decrease engine knocking
D. Single rail injection A. Ethel ether
ANSWER A B. Ethyl chloride
Water flow in diesel engine that is caused by density differential C. Ethyl nitrate
D. Ethyl oxide
ANSWER C C. Below the normal depth
For the submerged plane surface, the point on the surface where the resultant force acts is D. Below the critical depth
called the ANSWER B
A. Center of buoyancy Which of the following head loss coefficient among the following types of entrance?
B. Center of gravity A. Bell mouth
C. Center of pressure B. Square edge
D. Center of attraction C. Reentrant
ANSWER C D. It depends
At any point in fluid at rest, the pressure is the same in all direction. This principle is known ANSWER C
as: What waste water treatment method involves of algae from stabilization pond effluents?
A. Bernoulli Principle A. Sedimentation
B. Archimedes Principle B. Floatation
C. Pascal’s Law C. Filtration
D. Torricelli’s Law D. Microscreening
ANSWER C ANSWER D
The hot-wire manometer is used to measure The number of nozzles will depend on the quantity of steam required by the turbine. If nozzle
A. Pressure in gases occupy the entire arc of the ring, the turbine is said to have
B. Pressure in liquids A. Partially full peripheral admission
C. Wind velocities at airports B. One-half full peripheral admission
D. Gas velocities C. Maximum peripheral admission
ANSWER D D. Full peripheral admission
The pitot static tube measures ANSWER D
A. The static pressure Tandem compound units may also have two low pressure castings that produces
B. The gage pressure A. Single flow
C. The total pressure B. Double flow
D. The dynamic pressure C. Triple flow
ANSWER A D. Quadruple flow
The terminal velocity of a small sphere setting in a viscous fluid varies as the ANSWER C
A. First power of its diameter A type of turbine used for driving pumps, fans, and other auxiliaries in power plant commonly
B. Inverse of fluid viscosity operate at exhaust pressure approximating atmospheric
C. Inverse square of the diameter A. Tandem compound turbine
D. Inverse of the diameter B. Passout turbine
ANSWER B C. Cross-compound turbine
Pressure drag results from D. Back pressure turbine
A. Skin friction ANSWER D
B. Deformation drag A governor with 0% regulation is termed as:
C. Breakdown of potential flow near the forward stagnation point A. Isochronous governor
D. Occurrence of wake B. Synchronous governor
ANSWER A C. Isenchronous governor
The pressure coefficient is the ratio of pressure forces to D. Isobarnous governor
A. Viscous forces ANSWER A
B. Inertia forces The speed of regulation for most turbine-generators is adjustable from
C. Gravity forces A. 2 to 6%
D. Surface tension force B. 4 to 8%
ANSWER B C. 6 to 10 %
Tranquil flow must always occur D. 8 to 12%
A. Above the normal depth ANSWER A
B. Above the critical depth Poppet valves of steam turbine are used for extraction pressures of
A. 20 to 120 psig B. Retaining wall
B. 20 to 150 psig C. Coffer dam
C. 20 to 130 psig D. Earth dam
D. 20 to 140 psig ANSWER C
ANSWER B Which is not a physical characteristics of water?
When both bearings of steam engine are on one side of the connecting rod, the engine is A. Total suspended and dissolved solids
referred to as B. Turbidity
A. Center-crank engine C. Color
B. Side crank engine D. Hardness
C. Under crank engine ANSWER D
D. Standard crank engine Which dam is best for weak foundation?
ANSWER B A. Gravity
When the valve in steam engine is in mid-position of its travel, it will cover the steam port by B. Arch
an amount known as C. Buttress
A. Steam lap D. Earth
B. Partial lap ANSWER C
C. Full lap What is the volume of water which will drain freely the aquifer?
D. Angular lap A. Specific yield
ANSWER A B. Reservoir yield
A type of governor in steam engine that do not control the actual admission of steam to the C. Safe yield
cylinder but controls the pressure of the steam D. Secondary yield
A. Flyball governor
ANSWER A
B. Variable cut-off governor What is the line defined by water level in a group of artesian walls?
C. Throttling governor A. Water table
D. Shaft governor B. Peizometric surface
ANSWER C C. Specific yield
By inter-cooling using two stage compressor of brayton cycle, the backward ratio will D. All of the above
A. Increase
ANSWER B
B. Decrease Select the one that is positive indication of pollution of a river
C. Remains the same A. Acidity
D. None of these B. Oxygen content
ANSWER B C. Chloride content
On dynamic similitude, the relation which represents the ratio of inertia force to pressure force D. Nitrite content
is
ANSWER C
A. Froude number
Which instruments is used to measure humidity of the atmosphere continuously?
B. Cauchy number A. Barograph
C. Euler number B. Thermograph
D. Raynolds number C. Hydrograph
ANSWER C D. Thermo-hydrograph
What is the maximum velocity in a sewer flowing full?
ANSWER C
A. 0.6 m/sec
Entrance losses between tanks and pipe or losses through elbows, fittings and valves are
B. 0.9 m/sec generally expresses as a function of
C. 1.2 m/sec A. Kinetic energy
D. 1.8 m/sec B. Pipe diameter
ANSWER A C. Friction factor
A temporary structure constructed to exclude water from the site of the foundation during its D. Volume flow rate
excavation and construction is called:
ANSWER A
A. Caisson
The air that contains no water vapor is called
A. Zero air Passing from the solid phase directly into vapor phase is called
B. Saturated air A. Condensation
C. Dry air B. Fusion
D. Humid air C. Sublimation
ANSWER C D. None of these
In psychometric chart, the constant enthalpy lines coincide with constant temperature lines at ANSWER C
temperature Robert Boyle observed during his experiments with a vacuum chamber that the pressure of
A. Above 50°C gases is inversely proportional to their
B. Below 40°C A. Temperature
C. Below 50°C B. Pressure
D. Above 10°C C. Volume
ANSWER C D. None of these
The amount of moisture in air depends on its ANSWER C
A. Pressure Is energy in transition
B. Volume A. Heat
C. Temperature B. Work
D. Humidity C. Power
ANSWER C D. None of these
The deep body temperature of healthy person is maintained constant at ANSWER A
A. 27°C Is the mode of energy transfer between solid surface and the adjacent liquid or gas which is in
B. 37°C motion, and it involves combine effects of conduction and fluid motion
C. 47°C A. Conduction
D. 48°C B. Convection
ANSWER B C. Radiation
Air motion also plays important role in D. None of these
A. Surroundings ANSWER B
B. Cooling Radiation is usually considered as
C. Human comfort A. Surface phenomenon
D. None of these B. Surface interaction
ANSWER C C. Surface corrosion
During simple heating and cooling process has a humidity ratio D. None of these
A. Increasing ANSWER A
B. Decreasing Work is between the system and the surroundings
C. Constant A. Work interaction
D. None of these B. Energy interaction
ANSWER C C. Heat interaction
The follows a line of constant wet bulb temperature on the psychometric chart D. None of these
A. Evaporative cooling process ANSWER B
B. Condensing process Is a process during which the system remains in equilibrium at all times
C. Direct cooling process A. Quasi-equilibrium
D. None of these B. Static equilibrium
ANSWER A C. Dynamic equilibrium
A vapor which is not about to condense is called a D. None of these
A. Mixture of vapor and liquid ANSWER A
B. Critical vapor In the absence of any work interactions between a system and its surroundings, the amount of
C. Superheated vapor net heat transfer is equal
A. To the change in total energy of a closed system
D. None of these
B. To heat and work
ANSWER C
C. Energy interactions C. C7H8
D. None of these D. C6H12
ANSWER A ANSWER C
The constant volume and constant pressure specific heats are identical for As the air passes through a nozzle, which of the following will increase?
A. Compressible substance A. Temperature
B. Incompressible substance B. Enthalpy
C. Compressible gas C. Internal energy
D. None of these D. Mach number
ANSWER B ANSWER D
The velocity of fluid is zero at the wall and maximum at the center because of the As the air passes through a diffuser, which of the following will decrease?
A. Velocity effect A. Temperature
B. Viscous effect B. Enthalpy
C. Temperature effect C. Internal energy
D. None of these D. Mach number
ANSWER B ANSWER D
For steady flow devices, the volume of the control volume is As the air passes through a nozzle, which of the following will decrease?
A. Increase A. Entropy
B. Decrease B. Velocity
C. Constant C. Internal energy
D. None of these D. Mach number
ANSWER C ANSWER C
The work done in a turbine is since it is done by the fluid As the air passes through a diffuser, which of the following will increase?
A. Positive A. Density
B. Negative B. Entropy
C. Zero C. Mach number
D. None of these D. Velocity
ANSWER A ANSWER A
Reheating process in Brayton cycle, the turbine work will As the air passes through a diffuser, which of the following will NOT be affected?
A. Increase ANSWER B
B. Decrease After passing through a convergent-divergent nozzle, the temperature of air will:
C. Remains the same A. Increase
D. None of these B. Decrease
ANSWER A C. Remains the same
Which of the following is the chemical formula of ethanol D. None of these
A. C7H16 ANSWER B
B. C2H6O After passing though a convergent-divergent nozzle, the density of air will:
C. C7H8 A. Increase
D. C6H12 B. Decrease
ANSWER B C. Remains the same
Which of the following is the chemical formula of Heptane? D. None of these
A. C7H16 ANSWER B
B. C2H6O After passing through a convergent-divergent nozzle, the match number of air will:
C. C7H8 A. Increase
D. C6H12 B. Decrease
ANSWER D C. Remains the same
Which of the following is the chemical formula of Toluene? D. None of these
A. C7H16 ANSWER B
B. C2H6O
By increasing the temperature source of Carnot cycle, which of the following will not be D. Mass flow rate
affected? ANSWER A
A. Efficiency By sub-cooling the refrigerant in vapor compression cycle at condenser exit, which of the
B. Work following will decrease (Use per unit mass analysis)
C. Heat added
D. Heat rejected A. Coefficient of performance B. Heat rejected from condenser
C. Refrigerating effect D. Mass flow rate
ANSWER D
ANSWER: D
By decreasing the temperature source of Carnot cycle, which of the following will not be
By increasing the vaporizing temperature in vapor compression cycle, which the following
affected?
will increase? (Use per unit mass analysis)
A. Efficiency
A. mass flow rate B. COP C. specific volume D. compressor
B. Work work
C. Heat added ANSWER: B
D. Heat rejected By increasing the vaporizing temperature in vapor compression cycle, which the following
ANSWER C will decrease? (Use per unit mass analysis)
By superheating the refrigerant in vapor compression cycle with useful cooling, which of the A. Refrigerating effect B. COP C. evaporator temperature
following will increase? (Use per unit mass analysis)
A. Condenser pressure D. temperature difference between evaporator and compressor
B. Evaporator pressure ANSWER: D
C. Quality after expansion By increasing the condenser pressure in vapor compression cycle, which of the following will
increase? (Use per unit mass analysis)
D. Heat rejected from condenser
A. Refrigerating effect B. COP C. Specific volume at suction
ANSWER D D. Compressor power
By superheating the refrigerant in vapor compression cycle with useful cooling, which of the ANSWER: D
following will decrease? (Use per unit mass analysis) By increasing the condenser pressure in vapor compression cycle, which of the following will
A. Refrigerant effect B. COP C. Compressor power D. Mass flow rate decrease? (Use per unit mass analysis)
ANSWER D A. moisture content after expansion B. compressor power C. heat
By superheating the refrigerant in vapor compression cycle without useful cooling, which of rejected in the condenser D. mass flow rate
the following will decrease? (Use per unit mass analysis) ANSWER: A
A. Heat rejected B. COP C. Compressor power D. Specific volume at If the pressure drop in the condenser increases in a vapor compression cycle, which of the
suction following will increase? (Use per unit mass analysis)
ANSWER B A. mass flow rate B. compressor power C. heat rejected in the condenser
By superheating the refrigerant in vapor compression cycle without useful cooling, which of D. specific volume at suction
the following will increase? (Use per unit mass analysis) ANSWER: C
A. Refrigerant effect If the pressure drop in the condenser increases in a vapor compression cycle, which of the
B. COP following will decrease? (Use per unit mass analysis)
C. Compressor power A. Refrigerating effect B. mass flow rate C. heat rejected in the condenser
D. Compressor power
D. Mass flow rate
ANSWER: B
ANSWER C If the pressure drop in the condenser increases in a vapor compression cycle, which of the
By superheating the refrigerant in vapor compression cycle without useful cooling, which of following will not be affected? (Use per unit mass analysis)
the following will not be affected? (Use per unit mass analysis) A. compressor power B. mass flow rate C. heat rejected in the
A. Refrigerant effect condenser D. COP
B. COP ANSWER: A
C. Compressor power If the pressure drop in the evaporator increases in a vapor compression cycle, which of the
D. Mass flow rate following will increase? (Use per unit mass analysis)
ANSWER C A. Refrigerating effect B. vaporizing temperature
By sub-cooling the refrigerant in vapor compression cycle at condenser exit, which of the C. heat rejected in the condenser D. COP
following will increase? (Use per unit mass analysis) ANSWER: C
A. Refrigerating effect If the pressure drop in the evaporator increases in a vapor compression cycle, which of the
following will decrease? (Use per unit mass analysis)
B. Specific volume at suction
C. Compressor power
A. specific volume at suction B. compressor power C. heat rejected in ANSWER: C
the condenser D. COP Transfers heat directly to electrical energy by utilizing thermionic emissions.
ANSWER: D A. thermionic motor B. thermionic generator C. thermionic converter D.
By lowering the condenser pressure in Rankine cycle, which of the following will decrease? thermionic cell
(Use per unit analysis) ANSWER: B
A. Pump work B. turbine work C. heat rejected D. cycle Is the largest group of coal containing 46-89% of fixed carbon and 20% to 40% volatile
efficiency matter.
ANSWER: C A. anthracite B. sub-anthracite C. bituminous D. sub-
By increasing the boiler pressure in Rankine cycle, which of the following will decrease? (Use bituminous
per unit analysis) ANSWER: C
A. heat rejected B. pump work C. cycle efficiency D. moisture When 1 gram of coal is subjected to a temperature of about 105C for a period of 1 hour, the
content loss in weight of the sample gives the:
ANSWER: A A. Volatile matter B. ash C. fixed carbon D. moisture content
By superheating the steam to a higher temperature in Rankine cycle, which of the following ANSWER: D
will decrease? (Use per unit analysis) When 1 gram of sample of coal is placed in a crucible and heated 950C and maintain at the
A. moisture content at the turbine exhaust B. turbine work C. heat added temperature for 7 minutes there is a loss in weight due to elimination of:
D. heat rejected A. volatile matter and moisture B. ash C. fixed carbon D. moisture
ANSWER: A content
By superheating the steam to a higher temperature in Rankine cycle, which of the following ANSWER: A
will increase? (Use per unit analysis) Consist of hydrogen and certain hydrogen carbon compounds which can be removed from
A. moisture content at the turbine exhaust B. pump work C. condenser coal by heating.
pressure D. cycle efficiency A. moisture content B. product of combustion C. ash D. volatile
ANSWER: D, B matter
By reheating the steam before entering the second stage in Rankine cycle, which of the ANSWER: D
following will decrease? By heating 1 gram of coal in an uncovered crucible until the coal is completely burned, the
A. Turbine work B. moisture content after expansion C. heat ____ will formed.
added D. heat rejected A. volatile matter and moisture B. ash C. fixed carbon D. moisture
ANSWER: B content
What Rankine cycle is modified with regeneration, which of the following will increase? ANSWER: B
A. turbine work B. heat added C. heat rejected D. cycle Caking coal are use to produce coke by heating in a coke oven in the absence of ____ with
efficiency volatile matter driven off.
ANSWER:D A. air B. oil C. oxygen D. nitrogen
Is the combination of base load and peaking load? ANSWER: A
A. rated load B. intermediate load C. combine load D. Gindability of standard coal is:
over-all load A. 80 B. 90 C. 100 D. 110
ANSWER: B ANSWER: C
Sum of the maximum demand over the simultaneous maximum demand?
A. use factor B. capacity factor C. demand factor D. diversity Major constituent of all natural gases is:
factor A. ethane B. methane C. propane D. cethane
ANSWER: D ANSWER: B
Regenerative with feed heating cycle with infinite number of feed water heaters thus Two types of fans are:
efficiency is equal to? A. centrifugal and axial B. reciprocating and axial C. centrifugal and
A. otto cycle B. stirling cycle C. erricson cycle D. carnot rotary D. tangential and rotary
cycle ANSWER: A
ANSWER: D Enthalpy of substance at specified state due to chemical composition.
A type of turbine used in desalination of sea water. A. Enthalpy of reaction B. enthalpy of combustion C. enthalpy of formation
A. back pressure turbine B. passout turbine C. peaking turbine D. D. enthalpy of product
reaction turbine ANSWER: C
ANSWER: A A type of boiler used for super critical pressure operation.
State that when conductor and magnetic field move relatively to each other, an electric voltage A. La Mont boiler B. Once- through-circulation boiler C. Force
is induced in the conductor. circulation boiler D. Natural circulation boiler
A. Maxwell’s law B. Kirchhoff’s law C. Faraday’s law D. ANSWER: B
Newton’s law
Economizer in a water tube boiler is heated by: A ____ is used in aircraft engines and some automotive engine. In this method, a turbine
A. electric furnace B. electric current C. incoming flue gas D. outgoing driven by the exhaust gases is used to provide power to compressor or blower at the inlet.
flue gas A. discharging B. turbo charging C. supercharging D. scavenging
ANSWER: D ANSWER: B
Receives heat partly by convection and partly by radiation. The only devices where the changes in kinetic energy are significant are the
A. radiant superheater B. desuperheater C. convective superheater A. compressor B. pumps C. nozzles and diffusers D.
D. pendant superheater none of these
ANSWER: D ANSWER: C
Regenerative superheater is a storage type of heat exchangers have an energy storage The distance between TDC and BDC in which the piston can travel is the
medium called: A. Right extreme position B. displacement stroke C. stroke of
A. matrix B. regenerator C. boiler D. recuperator the engine D. swept stroke
ANSWER: A ANSWER: C
Stirling cycle uses a ____ as working fluids. In compression-engine, the combustion of air-fuel mixture is self-fuel ignited as a result of
A. incompressible gas B. incompressible fluids C. compressible compressing the mixture above its
refrigerant D. compressible fluids A. self developed temperature B. mixing temperature C. self feed
ANSWER: D temperature D. self ignition temperature
In stirling process the heat added is added during? ANSWER: D
A. isobaric process B. isentropic process C. isothermal process
D. heat process The thermal efficiency of an ideal Otto cycle depends ____ of the working fluid.
ANSWER: C A. the pressure ratio of the engine and the specific heat ratio
Brayton cycle is also known as: B. the temperature ratio of the engine and the specific heat ratio
A. carnot cycle B. joule cycle C. diesel cycle D. rankine C. the moles ratio of the engine and the specific heat ratio
cycle
D. the compression ratio of the engine and the specific heat ratio
ANSWER: B
Is applied to propulsion of vehicles because of certain practical characteristics. ANSWER: D
A. diesel cycle B. otto cycle C. carnot cycle D. brayton As the number of stages is increased, the expansion process becomes
cycle A. isentropic B. isothermal C. isometric D. polytropic
ANSWER: D ANSWER: B

Heat exchangers typically involve Aircraft gas turbines operate at higher pressure ratio typically between
A. no work interactions B. no heat interactions C. no energy A. 6 to 8 B 12 to 24 C. 10 to 18 D. 10 to 25
interactions D. none of these ANSWER: D
ANSWER: A The first commercial high-pass ratio engines has a bypass ratio of
A device that is used to convert the heat to work is called A. 1 B. 3 C. 5 D. 7
A. adiabatier B. regenerator C. heat engines D. none of ANSWER: C
these The single-stage expansion process of an ideal Brayton cycle without regeneration is replaced
ANSWER: C by a multistage expansion process with reheating the samepressure limits. As a result of
The objective of a heat pump is to maintain a heated space at modification, thermal efficiency will:
A. Low temperature B. high temperature C. medium A. increase B. decrease C. remain constant D. none of
temperature D. none of these these
ANSWER: B ANSWER: B
A device that violates the second law of thermodynamics is called Which of the following is/are the application of Brayton cycle.
A. perpetual motion machine of second kind B. perpetual motion machine of third A. propulsion system B. automotive turbine engine C. aircraft turbine
kind C. perpetual motion machine of first kind D. none of these engineD. all of these
ANSWER: A ANSWER: D
A process is called ____ if no irreversibility’s occur outside the system boundaries during the It is used as working fluid in high-temperature applications of vapor cycles.
process. A. helium B. deuterium C. mercury D. water
A. externally reversible B. internally reversible C. reversible D. ANSWER: C
none of these The superheated vapor enters the turbine and expands isentropically and produces work by
ANSWER: A the rotating shaft. The ____ may drop during the process.
An energy interaction which is not accompanied by entropy transfer is A. density B. viscosity of fuel C. temperature and pressure
A. energy B. heat C. work D. none of these D. none of these
ANSWER: C ANSWER: C
Only ____ of the turbine work output is required to operate the pump. A. internal energy B. work C. gravitational potential energy
A. 0.01% B. 0.02% C. 0.03% D. 0.04% D. kinetic energy
ANSWER: D ANSWER: D
Superheating the steam to higher temperatures decreases the moisture content of the steam at A thermodynamic process wherein temperature is constant and the change in internal energy is
the ____. zero.
A. turbine inlet B. compressor inlet C. compressor exit A. isobaric process B. isometric process C. isothermal process
D. turbine exit D. polytropic process
ANSWER: D The science terminology concerned with precisely measuring energy and enthalpy
Regeneration also provides a convenient of dearating the feedwater to prevent? A. thermodynamics B. chemistry C. calorimetry D. none of these
A. boiler explosion B. boiler scale production C. boiler corrosion D. ANSWER: C
compressor damage The rate of doing work per unit time
ANSWER: C A. torque B. power C. force D. moment
Can be apply steam turbine cycle(rankine). Gas turbine cycle(brayton) and combined cycle. ANSWER: B
A. hydroelectric plant B. nuclear power plant C. cogeneration plant A vapor having a temperature higher than the saturation temperature corresponding to the
D. tidal power plant existing pressure.
ANSWER: C A. superheated vapor B. saturated vapor C. wet vapor D. none of the above
In a rankine cycle with fixed turbine inlet conditions. What is the effect of lowering the ANSWER: A
condenser pressure, the heat rejected will: It is the work done in pushing a fluid across a boundary, usually or out of a system.
A. increase B. decrease C. remains the same D. none of A. mechanical work B. nonflow work C. flow work D. electrical work
these ANSWER: C
ANSWER: B A liquid that has a temperature lower that the saturation temperature corresponding to the
In an ideal rankine cycle with fixed boiler and condenser pressure. What id the effect of existing pressure.
superheating the steam to a higher temperature, the pump work input will. A. subcooled liquid B. saturated liquid C. unsaturated liquid D. water
A. increase B. decrease C. remain the same D. none of ANSWER: A
these In this type of boiler, the water passes through the tubes while the flue gases burn outside the
ANSWER: C tubes.
The fact that total energy in any one energy system remains constant is called the principle of A. water-tube boiler B. fire-tube boiler C. steam generator D. electric
____? boiler
A. conversion of energy B. second law of thermodynamics C. conservation of ANSWER: A
mass D. zeroth law of thermodynamics It shows the water level in the boiler drum.
ANSWER: A A. water column B. try cocks C. gauge glass D. all of the above
A process for which the inlet and outlet enthalphies are the same ANSWER: C
A. isntropic B. enthalphy conservation C. throttling D. steady state It prevents damage to the boiler by giving warning of low water.
ANSWER: C A. safety valve B. fusible plug C. relief valve D. try cocks
ANSWER: B
The sum of energies of all the molecules in system, energies that in several complex forms. it is heat exchanger which utilizes the heat of the flue gases to preheat air needed for
A. kinetic energy B. internal energy C. external energy D. flow work combustion.
ANSWER: B A. economizer B. feedwater heater C. reheater D. air preheater
A system that is completely impervious to its surrounding. Neither mass nor energy cross its ANSWER: D
boundaries. It is the subject that deals with the behavior of moist air.
A. Open system B. closed system C. adiabatic system D. A. psychrometer B. psychometry C. refrigeration D. pneumatics
isolated system ANSWER: B
ANSWER: D Air whose condition is such that any decreases in temperature will result in condensation of
A device used to measure small and moderate pressure difference. water vapor into liquid.
A. manometer B. bourdon gage C. barometer D. piezometer A. saturated air B. unsaturated air C. saturated vapor D. moist air
ANSWER: A ANSWER: A
A vapor having a temperature higher that the saturation temperature corresponding to its It is the warm water temperature minus the cold water temperature leaving the cooling tower
pressure. A. approach B. terminal difference C. cooling range D.
A. superheated pressure B. saturated vapor C. super saturated vapor LMTD
D. subcooled vapor ANSWER: C
ANSWER: A The surrounding air ____ temperature is the lowest temperature to which water could possibly
The energy or stored capacity for performing work possessed by a moving body, by virtue of cooled in a cooling tower.
its momentum. A. dry-bulb B. wet-bulb C. dew-point D. saturated temperature
ANSWER: B Ans. Compressor
Which of the following compressors have the compressing element and drive seals in a single
PIPE ELEMENTS housing?
Ans. hermitically sealed compressor
Which of the following compressors are so widely used for today’s refrigeration system? Which of the following consists of two mating helically grooved male and female grooves?
Ans. Centrifugal Ans. Helical rotary-screw
A valve sometimes known as the magnetic valve. What is the another name of helical-rotary screw compressors?
Ans. Solenoid valve Ans. Lysholm type
Which of the following valve use to regulate the flow of refrigerant to the evaporator? At what head is the helically-rotary compressor are designed to operate?
Ans. Expansion valve Ans. High head pressure
Which of the following use control the flow of refrigerant gas from the evaporator coil. This The bulk quantity of oil separated from the refrigerant in the helically-rotary compressor falls
type of control valve is also known as back pressure regulator of an evaporator. by?
Ans. Suction line regulator Ans. Gravity to a sump
Which of the following refrigerant control is used to limit the flow of gas to the compressor to If the temperature in the icebox is too high, the trouble could be:
prevent surge of excessive load from overloading the compressor. Ans. Automatic control not functioning properly
Ans. Hold-back valve If any of the electrically controlled devices in a Freon system malfunction, which of the
Which of the following refrigerant control is used in the liquid suction or discharge to following valves also automatically shut-off?
interrupt the flow on demand from any one of the several types of temperature or pressure Ans. Solenoid valve
sensing devices. Natural ice in an ice box was used in the early
Ans. Solenoid valve Ans. 19th century
What is the pressure drop from the receiver to the expansion valve? The amount of matter present in a quantity of any substance is called:
Ans. 5 psi Ans. Mass
If any of the electricity controlled device in a Freon system malfunction, which following What pressure is indicated by a barometer?
valves will also automatically shut off? Ans. Atmospheric pressure
Ans. Solenoid valve The basic unit measurement for heat is:
The relief valve on a CO2 machine is located: Ans. BTU
Ans. On the discharge pipe between the compressor and the discharge valve. The force that acts upon a unit of area is termed:
When checking zinc plates in a condenser, one should: Ans. Pressure
Ans. Clean the plate and renew worn-out ones. What is the physical state of a refrigerant entering the flow control?
A refrigerant should have a Ans. Sub-cooled
Ans. High latent heat What method of heat transfer does not depend on molecular motion?
The mechanical energy of a device is the ratio of: Ans. Radiation
Ans. Mechanical energy input to the mechanical energy output Where s the filter drier is located?
When the winding or circuit is open, what is the resistance reading? Ans. After the condenser
Ans. Infinity A motor drive device which removes the heat-laden vapor refrigerant from the evaporator is
If the meter scale of the ohmmeter reads 0 ohms and the range adjustment is R 10, what is the the:
resistance reading? Ans. Compressor
Ans. 100 The function of the compressor is to:
What is the resistance reading of an electric motor in good condition? Ans. Squeeze out refrigerant from the evaporator
Ans. With resistance In which part of the compressor is the piston attached to the crankshaft?
The basic unit of electrical pressure is: Ans. Connecting rod
Ans. Volt This is a compressor which has an off center-rotor.
What is the most commonly used conductor? Ans. Rotary
Ans. Copper Which of the following compressor is commonly used in household refrigerators?
Which of the following is the effect of superheating the refrigerant? Ans. Hermetic
Ans. Increase is COP What is the length of the tubing extended from the part to be swaged
Which of the following statement is true in ice making capacity? Ans. ½ inch
Ans. Ice making capacity is always proportional to the refrigerating effect The best method of making leak proof connection is:
Which of the following has the largest heat load in cold storage room? Ans. Silver brazing
Ans. Product heat load The refrigerant that should not be used copper or brass tubing:
Which of the following is a halocarbon refrigerant? Ans. R-717
Ans. Methyl chloride The color of the hose connected to suction line or process be
A refrigerant system in which pressure- imposing element is mechanically operated? Ans. Blue
Where is the compound gauge installed in a refrigerant system? Ans. Metal-edge filter
Ans. Suction line # When four events takes place in one revolution of a crankshaft of an engine, the engine is called
A gauge used to measure pressure both below and above atmospheric pressure Ans. Two stroke engine
Ans. Compound gauge # Which of the following does not belong to the group.
What is the valve position if the valve stem is turned all the way in? Ans. Time injection system
Ans. Front-seated # The function of a super heater is to:
What is the valve position if the valve stem is turned all the way out? Ans. Exchange heat to increase energy to the flow during an adiabatic, internally
Ans. Back-seated reversible process
Ohm’s law states that resistance is inversely proportional to: # What is true about change in entropy?
Ans. Current Ans. it is always zero
To prevent corrosion within the flue gas, what must be prevented from condensing? # How does an adiabatic process compare to an isentropic process?
Ans. Vapor Ans. Both heat transfer = Q, isentropic reversible
Ans. Evaporative cooling # Adiabatic heat transfer within a vapour cycle refers to :
# An adiabatic saturation process. This process can be produced with _____, which is essentially Ans. The transfer of energy from one stream to another in a heat exchanger that the energy of
a regular thermometer with its bulb wrapped in wet cotton or gauze..... the input streams equal to the energy of output.
Ans. Sling psychrometer # All of the following process are irreversible except:
# A high velocity gas is defined as a gas moving with a velocity in excess off approximately Ans. An isentropic compression of a perfect fluid
Ans. 300ft/s/ 100m/s # Refers to the system where the evaporator coils are placed in the ice box
# The theoretical maximum velocity is achieved when..... Ans. Direct system
Ans. All internal and pressure energies are converted to kinetic energies # What is the usual dehydrating agent in Freon system?
# If the gas flow is adiabatic and frictionless (i.e. reversible) Ans. Activated alumina
Ans. Both A and B # Refers to the system where the evaporator coils are indicated in a brine solution and brine is pumped
# A property by which the refrigerant remains at its original chemical form/original condition is thru the ice box
imposed by the operation..... Ans. Indirect system
Ans. Stability of refrigerant # Is the most common dryer which consist of rotating cylinder inside which the materials flow
# The maximum temperature at which a gas is condense into liquid. Above temperature, a vapor while getting in contact with hot gas.
imposed by the operation..... Ans. Rotary dryer
Ans. Stability of refrigerant # Is the ratio of the mass of water-vapor in air and mass of air if it is saturated is called:
# Which of the following is the name given for halogenated hydrocarbon? Ans. Relative humidity
Ans. Halocarbon # The hands feel painfully cold when the skin temperature reaches
# Which of the following refrigerants are produced by carrier corporation? Ans. 10°C
Ans. Carrene # The total heat of air is a function of
# Which of the following refrigerants is produced by E.L.DU FONT? Ans. WB temperature
Ans. Freon # Boiling point of Freon-12 at atmospheric pressure is:
#Which of the following is the other name of R 718? Ans. 21°F
Ans. Water # Which of the following is NOT a type of water cooled condenser in refrigeration?
# Energy per unit volume stored in a deformed material is called: Ans. Double shell
Ans. Strain energy # Component of absorption refrigeration system in which the solution is cooled by cooling water.
# The ratio of ultimate failure strain to yielding strain is known as: Ans. Absorber
Ductility # Cascade refrigeration cycle is often used in industrial process where objects must be cooled to
# One ton of refrigeration is equal to: temperature below:
Ans. 3.5KW Ans. -46°C
# With what cycle does air refrigeration works? # Type of refrigerant control designed to maintain a pressure difference while the compressor is
Ans. Bell-coleman cycle operating.
# What cycle does air refrigeration works? Ans. Capilliary tube
Ans. Coal # As a rule of thumb, for a specified amount of compressed air, the power consumption of the
# What is produced by fission process? compressor decreased by ____________for each 3°C drop in the temperature inlet air to the
Ans. Radiation compressor.
# Loss of power is due to: Ans. 1 percent
Ans. Low injection pressure # Modern way of detecting air compressor leak is by using
# A branch system of pipes to carry waste emissions away from the combustion chamber. Ans. Acoustic leak detector
Ans. Exhaust manifold # For foundation of stacks, the maximum pressure on the soil is equal to the pressure due to the
# The type of filter where the filtering elements is replaceable. weight and the _______.
Ans. Wind movement # Refers to the increase of enthalpy of a substance when it undergoes some phase change at
# Foundation bolts of specified size should be used and surrounded by a pipe sleeve with an inside constant pressure and temperature.
diameter of at least Ans. Heat of vaporization
Ans. 3 times the diameter of anchor bolt # Which of the following keeps moisture from passing thru the system?
# For multi stage compression of an ideal Brayton cycle, the back work ratio will Ans. Dehydrator
Ans. decrease # What is the lowest temperature to which water could possibly be cooled in a cooling tower?
# Type of turbine that has a specific speed below 5. Ans. Temperature of adiabatic saturation
Ans. Impulse turbine #What do you call the intake pipe to a hydraulic turbine from a dam?
# A high discharge type of turbine Ans. Penstock
Ans. Propeller turbine # An ideal fluid is one that
# Use to minimize the speed rise due to a sudden load rejection Ans. is frictionless and incompressible
Ans. jet deflector # What principal was used by McLeod gauge which is used for low pressure measurement/
# Is the speed of a turbine when the head on the turbine is one meter. Ans. Boyles Law
Ans. Unit speed #The difference between the brake and hydraulic power is
# Is a fluid property which refers to the intermolecular attraction by which the separate particles of the Ans. Friction power
fluid arc held together. # Product of motor pump efficiency is known as
Ans. Cohesion Ans. Over-all efficiency
# Which of the following is NOT the cause of black smoke in diesel engine? # The most efficient cross-section of an open channel is the one that
Ans. high compression pressure Ans. All of the above
# Which of the following is not a method of starting a diesel engine? #Refers to the structure across an open channel over which water flows.
Ans. Using another generator Ans. Weir
# Two-stroke engine performs _____ to complete one cycle. # When can we say that a trapezoidal weir is a “cipoletti” weir?
Ans. compression and power stroke Ans. When side slope 4 vertical and 1 horizontal
# A type of geothermal plant used when there is a presence of brine extracted from undergroungd # A weir is broad-crested if:
Ans. Binary geothermal plant Ans. The weir length is greater than half of the head
# Is the most important safety device on the power boiler. # A feature of a dam over where water is discharged is called:
Ans. Safety valve Ans. Spillway
# During hydrostatic test, the safety valves should be # Laminar flow occurs when the Reynolds number is approximately less than
Ans. removed Ans. 2100
# Where deaerating heaters are not employed, it is recommended that the temperature of the # Turbulent flow occurs when the Reynolds number exceeds:
feed water be not less than _______ Ans. 4000
Ans. 197°C # Laminar flow is typical when
# What is the possible cause of too much suction pressure? Ans. All of the above
Ans. Shortage refrigerant # When the Reynolds number is between 2100 to 4000, the flow is said to be in
Where is the oil separator located? Ans. Critical zone
Ans. Between the compressor and condenser # Which of the following characterizes a laminar flow?
# What is the purpose of the low pressure cutout switch? Ans. All of the above
Ans. To cut compressor in and out at present pressure # Without a pump and turbine, which of the following rules is true in a frictionless environment about
# Freon unit will tend to short cycle when operating under energy and hydraulic gradient?
Ans. Light loads Ans. All of the above
# What cause hot suction line? # What is the latent heat of vaporization of water?
Ans. Insufficient refrigerant Ans. 970 Btu
# When changing a Freon system which valve is not used? # A dry bulb temperature at which water starts to condense but when moist appears in a constant
Ans. King valve pressure process
# Where is the scale trap located? Ans. Dew point temperature
Ans. Between the king valve and expansion valve # A mixture of dry air and saturated vapour is known as:
# Where is the solenoid valve located? Ans. Saturated air
Ans. Between the scale trap and thermal expansion valve # What is another term for “humidity ratio”?
# Water tube boilers have how many fusible plugs? Ans. Specific humidity
Ans. 4 # The water vapour mass is often reported in
# What is the main cause of air pollution as a result of burning fuel? Ans. Grains of water
Ans. Nitrogen dioxide # Approximately how many grains of water are there in 1 pound?
Ans. 7000
# The ratio of actual humidity ratio to the saturated humidity ratio at the inlet temperature and p Ans. Rated horsepower
# Which of the following is a possible cause of cavitation
Ressure is known as Ans. All of the above
Ans. Degree of saturation # Is a reaction during which chemical energy is released in the form of heat
# What is another term for “degree of saturation”? # When forebay is not part of the generating plant design, it will be desirable to produce a ________ in
Ans. Percentage of humidity order to relieve the effect of rapid changes in flow rate.
# The partial pressure of the water vapour divided by the saturation pressure? Ans. Surge chamber
Ans. Relative humidity #What is used to keep the turbine in a hydroelectric generating plant up to 15 feet above the tail
# The point where sonic velocity has been achieved (M=1) is known as: water surface?
Ans. Critical point Ans. Draft tube
# The ratio of the thrust in equal mass flow rate is known as... # If a draft tube is not employed, water may be return to the tail water by way of a chamber known as
Ans. Effective exhaust velocity the:
# Is the velocity of gas dropped from supersonic to subsonic, gas will experience Ans. Tail race
Ans. Shock wave. # An impulse turbine consists of a rotating shaft on which buckets of blades are mounted. What is
# An adiabatic flow with friction, which would be approximately as flow through an insulated duct... the term used for rotating shaft?
Ans. Fan no flow Ans. Turbine runner
# What is an inevitable by product of the combustion of hydro carbon fuel? # Which of the following refrigerant that belong to group B refrigerants and harmful or toxic
Ans. Water vapour refrigerants?
# A compound pressure gauge is used to measure: Ans. All of the above
Ans. Positive and negative pressure # Which of the following refrigerants are combustible?
# Which of the following best describes a Curtis turbine? Ans. All of the above
Ans. It is a velocity pressure compound turbine # Which of the following refrigerants is used for frozen foods and ice cream display cases
# What causes low head pressure? Warehouses and food freezing plants, medium temperature display cases truck refrigeration and heat
Ans. Too much cooling water and in sufficient refrigeration gas pumps?
# What causes high head pressure? Ans. R-502
Ans. All of the above # The refrigerant that was formerly the most widely used for air conditioning and refrigeration. It
# A commercial pipe which has the characteristics of being immune to electrolysis and corrosion, light was the principal refrigerant for automotive air conditioning.
in weight weak structurally. Ans. R-12
Ans. Asbestos cement pipe # How do we know that there is shortage of refrigerant considering that we are in the liquid line?
# A commercial pipe which is durable, water tight, low maintenance and smooth interior. Ans. Vapor bubbles in the sight glass appear
Ans. Concrete pipe # The amount of power actually entering a fluid is known as:
# A commercial pipe which has the characteristics of being corrosion, and scour and erosion Ans. All of the above
Resistance. #The input power to the pump will be:
Ans. Vitrified clay pipe Ans. Equal to the output power of the electric motor driving the pump
# A commercial pipe which that is ductile, high strength, shock resistant, very smooth internally. # A bent or curved carries fluid from a container at a high elevation to another container at a lower
Ans. Steel pipe elevation.
a Ans. Siphon
# A commercial pipe that is chemically insert, resistant to corrosion, very smooth, light weigth and # A water path, usually a large diameter pipe used to channel water around or through an obstruction
very cheap feature.
Ans. Plastic (PVC and ABS) Ans. Culvert
# Which commercial pipe that is primarily used for water condensate, and refrigerant lines easier to # Which of the following principles governs the distributions of flow between the two branches in a
bent by hand, good thermal conductivity pipe system.
Ans. Brass and copper pipe Ans. All of the above
# A fluid pathway that expose parts of the fluid to the atmosphere is the: # A method used to determine the network flow in a multi-hoop piping system.
Ans. Open channel Ans. Hardy-cross method
# The ratio of the area in flow to the width of the channel at fluid surface in an open channel is # A device used to measure velocity by determining the cooling effect of fluid flowing over an
known as: electrically heated tungsten.
Ans. Hydraulic depth Ans. Hot-wire anemometer
# The most efficient cross-section for an open channel is known as: # Which of the following is used for R-12?
Ans. Semi-circular section Ans. All of the above
# The most efficient section rectangular section has: # Which of the following tubing (pipes) usually used in ammonia refrigerant?
Ans. A width twice the depth Ans. Ferrous pipe
# What is the maximum power that the motor can provide # Which of the following tubing (pipes) usually used in halo-carbon refrigerant?
Ans. K and L copper tubing # What is used to increased the mean effective temperature at which heat is added without producing
# Properties of superheated vapour can be found in? significant expansion in the liquid-vapor region.
Ans. Superheated table Ans. Superheat
# Properties of non-reacting gas mixture are given by: # If some of the heat energy from these waste product is recovered and used for space heating or
Ans. Volumetric weight for molecular weight and density and geometric weighing for all cooling the process is called
the Ans. Cogeneration
properties except entropy. # In cogeneration, the recovered heat
# The relation between the total volume of a mixture of a non-reacting gases for initial volume is Ans. Is used as heat
given by: # The ratio of useful energy to the energy input is called?
Ans. Amagat’s law Ans. Fuel utilization
# Which of the following is the first definition of enthalpy? # The ratio of the energy of a turbine to the recovered heat is known as:
Ans. The amount of useful energy in a system. Ans. Power to heat ratio
# A consists press thermodynamics process obeys: # If the recovered heat is used to vaporized water in a vapour cycle this is called:
Ans. Charles law Ans. Combined cycle
# The volume of an ideal is halved, while it’s temperature is double, what happens to the pressure? # What is the best coolant for high temperature gas reactor?
Ans. Pressure is multiplied by 4 Ans. Helium (He)
# A liquid boils when its pressure equal. # What is the liquid metal most frequently used as coolant, in liquid metal reactor?
Ans. Ambient pressure Ans. Sodium (Na)
# A system composed of ice and water at zero °C is said to be: # Combustion power cycles differ from vapour power in that:
Ans. All of the above Ans. The combustion product cannot be returned into their initial condition for reuse.
# A heat of fusion for a pure substance is ________. # A closed system using a fixed amount of ideal air as the working fluid?
Ans. The energy required to melt the substance. Ans. Air standard cycle
#The heat of vaporization involves the change of enthalpy due to: # Which of the following is not a specific compound but is a mixture of octane and lighter
Ans. The change in phase from liquid to gas. hydrocarbon?
# The heat of sublimation involves the change of enthalpy due to: Ans. Gasoline
Ans. The change in phase from solid to gas. # A series of process that eventually brings the system back to its original condition is known as?
# The unit for absolute viscosity which is dyne-sec/cm3 is also known as: Ans. Cycle
Ans. Poise # The _____ of a power cycle is defined as the ratio of usual work output to the supplied input
# What is the unit of absolute viscosity? energy?
Ans. Pa-second Ans. Thermal efficiency
# The ratio of absolute viscosity to mass density is also known as: # Is use to heat up the solution partially before entering the generator in absorption refrigeration
Ans. Kinetic viscosity system.
# What is the kinetic viscosity? Ans. regenerator
Ans. m2/s # The COP of actual absorption refrigeration system is usually
# Stroke is a unit of kinetic viscosity which is equivalent to: Ans. less than 1
Ans. cm2/s # Sight glass is often located at:
# What are volatile liquids? Ans. liquid line
Ans. The liquid that vaporizes immediately # Use to detects a vibration in current caused by the ionization of decomposed refrigerant between
# When does boiling occur? two opposite-charged platinum electrodes.
Ans. When the liquid temperature is increased to the point that the vapour present is equal Ans. Electronic detector
to # The ability of oil to mix with refrigerants
the local ambient pressure. Ans. miscibility
# The liquid boiling pressure is dependent on: # For the submerged plane surface, the point on the surface where the resultant force acts is called
Ans. Both A and B the
# Referred as the mass flow the rate divided by the generator output in kilowatts Ans. center of pressure
Ans. Water rate of stream rate # At any point in fluid at rest, the pressure is the same in all directions. This principle is known as:
# The power after the auxiliary loads has been removed is known as: Ans. Pascal’s Law
Ans. Net electrical output # The hot-wire manometer is used to measure
# _________ of a turbine is the ratio of actual to ideal energy extraction? Ans. gas velocities
Ans. Both A and B # The pitot static tube measures
# What is the approximate maximum practical metallurgical limit on superheat? Ans. the static pressure
Ans. 1150°C and 625°C # The terminal velocity of a small sphere setting in a viscous fluid varies as the
Ans. inverse of fluid viscosity
# Pressure drag results from Ans. Wet-bulb temperature
Ans. skin friction # How often should compressor oil be changed?
# The pressure coefficient is the ratio of pressure forces to: Ans. At least semi-annually
Ans. inertia forces # Which of the following is the possible effect of the weak solution of brine in a refrigeration system?
# Tranquil flow must always occur Ans. Corrosion
Ans. above the critical depth # In Freon refrigeration system, where is the scale trap located?
# Which of the following head loss coefficient among the following types of entrance? Ans. On the suction side of the compressor
Ans. re-entrant # At what pressure is the high pressure cut-off in the Freon (R-12)
# What waste water treatment method involves of algae from stabilization pond effluents? Ans. 125-150 psi
Ans. microscreening # What is the other name for brine refrigeration system?
# The number of nozzles will depend on the quantity of steam required by the turbine. If the Ans. Indirect refrigeration system
nozzles occupy the entire arc of the ring, the turbine is said to have: # What is the indication that there is air in the refrigeration system?
Ans. full peripheral admission Ans. Unusual high head pressure
# Tandem compound units may also have two low-pressure casting that produces: # A Freon-12 leaking can be detected by halide torch. What color will it turn to in the presence of a
Ans. triple flow refrigerant if this torch has a normal blue flame?
# By inter-cooling using two stage compressor of Brayton cycle, the backwork ratio will: Ans. Green
Ans. decrease # All of the following process are irreversible except
# On dynamic similitude, the relation which represents the ratio of inertia force to pressure force is: Ans. Inelastic tension and release of steel bar
Ans. Euler number # Which of the following is a point function except
# What is the maximum velocity in a sewer flowing full? Ans. Work
Ans. 0.6 m/sec # All heat transfer process require medium of energy exchange except
# A temporary structures constructed to exclude water from the site of the foundation during its Ans. Radiation
excavation and construction is called: # Thermal conduction is described by:
Ans. coffer dam Ans. Fourier Law
# Which is not a physical characteristics of water? # Convection is described by:
Ans. hardness Ans. Newton’s Law
# Which dam is best for weak foundation? # Radiant heat transfer is described by:
Ans. buttress Ans. The Stefan-Boltzman Law
# What is the volume of water which will drain freely from the aquifer? # The equivalence of ratios of emissive power to absorptivity equilibrium is described by
Ans. specific yield Ans. Kirchoff’s Law
# What is the line defined by the water level in a group of artesian wells? # The temperature potential between temperature difference.
Ans. peizometric surface Ans. The logarithmic temperature difference
# Select the one that is a positive indication of pollution of a river. # A thermodynamic process whose deviation from equilibrium is infinitesimal at all times.
Ans. chloride content Ans. In quasi-equilibrium
# Which instruments is used to measure humidity of the atmosphere continuously? # A thermodynamic property best describes the molecular activity of a substance?
Ans. hydrograph Ans. Internal energy
# Entrance losses between tank and pipe or losses through elbows, fittings and valves are generally # The combination of conditions that best describes thermodynamic process is given:
expresses as a function of: Ans. Has successive states thru which the system passes
Ans. kinetic energy # A substance whose properties are uniform throughout is called a:
# The air that contains no water vapour is called: Ans. Pure substance
Ans. dry air # All of the following are thermodynamic properties except:
Ans. Modulus
# A process that is adiabatic and reversible is also called:
# What effectively states that it is impossible to built a cylindrical engine that will have a thermal Ans. Isentropic
efficiency of 100%? # The first law of thermodynamics for a closed system is Q= ∆U + W, the sign convention is:
Ans. Kelvin-Planc statement of second law Ans. Q positive in, W positive out and ∆U negative for decreased internal energy
# Refers to the maximum possible work that can be obtained from a cycle. # The heat transfer term in the first law of thermodynamics may be due to any of the ff. except:
Ans. Availability Ans. Internal heat generation
# The difference between the maximum and the actual work output is known as? # A system that experiences no mass crossing the system boundaries called:
Ans. Process inversibility Ans. Closed system
# The study of the property of atmospheric air? # A substance in which a substance is allowed to enter and have is most properly called:
Ans. Psychrometry Ans. Open system
# The temperature of the air that has gone through an adiabatic saturation process is known as? # First and second law of thermodynamics are:
Ans. Energy equations # Bodies that radiate at absorptivity of 1 are known as:
# A constant temperature thermodynamic process obeys: Ans. Black bodies or ideal radiators
Ans. Boyles Law # Which of the following does not radiate at the ideal level?
# A series of process that eventually bring the system back to the original condition is called a: Ans. Real body
Ans. Cycle # The ratio of actual to ideal emissive power is known as:
# If the refrigerant leaves the evaporator with a quality of < 1, the cycle is known as: Ans. Emissivity
Ans. Wet vapor compression cycle # What is the refrigerator’s main function?
# For even lower temperature, are required as in commercial freezing equip. What is used as Ans. To cool this low temperature area
refrigerant? # The rate of energy removal from the low temperature area is known as:
Ans. Ammonia Ans. Either of the refrigeration capacity or refrigerating effect
# What is another name for Air-Refrigeration cycle? # The refrigeration capacity is measured in refrigeration ton where one ton is _____ heat
Ans. Brayton cycle remove
# What is the major disadvantage of Air-Refrigeration cycle? Ans. All of the above
Ans. High power consumption # The refrigeration ton is derived from the heat flow required to melt the ton of ICE in:
# What refrigeration is practically used when a large quantity of waste-inexpensive heat is avail? Ans. 24 hours
Ans. Heat-driven refrigeration cycle # In psychrometric chart, the constant- enthalpy lines coincide with constant-temperature
# For an absorption cycle, how many working fluids are required? lines at temperature
Ans. Two (2) Ans. below 50°C
# In absorption cycle, what is needed in NH3 water system to remove any remaining traces of # The amount of moisture in air depends on its
absorbents from the refrigerant and is placed between generator and condenser. Ans. temperature
Ans. Rectifier # The deep body temperature of healthy person is maintained constant at
# Which is a “TEMA” stands for? Ans. 37°C
Ans. All of the above # Air motion also plays important role in
# The acronym “ TEMA” stands for? Ans. human comfort
Ans. Tubular Exchangers Manufacturing Association # During simple heating and cooling process has a_____ humidity ratio.
# The acronym “API” stands for? Ans. constant
Ans. American Petroleum Institute # The ________ follows a line of constant wet-bulb temperature on the psychrometric chart.
# What occurs when the exit temperature of the cold fluid is above the exit temperature of Ans. evaporative cooling process
the hot fluid? # A vapor which is not about to condense is called a
Ans. Temperature cross Ans. superheated vapor
# Are heat exchangers whose purpose is to heat with condensing stream. # Passing from the solid phase directly into vapor phase is called
Ans. Closed feedwater heater Ans. sublimation
# Refers to the corrosion, precipitation of compound in solution, setting of particular solid # Robert Boyle observed during his experiments with a vacuum chamber that the pressure of
and biological activity that adhere to a heat transfer gases is inversely proportional to their
Ans. Fouling Ans. volume
# Fouling in a heat exchanger industries is commonly known as: # _____ is energy in transition.
Ans. Silent thief Ans. Heat
# An operation with 1 fluid flow rate substantially known as: # Is the mode of energy transfer between a solid surface and the adjacent liquid or gas which is
Ans. Down turn in motion, and it involves combine effects of conduction and fluid motion.
# Indicate the false statement? Ans. convection
Ans. Duo to viscosity, liquid cannot resist instantaneously change velocity # Radiation is usually considered as
# Ideal liquid are assumed to be? Ans. surface phenomenon
Ans. Newtonian fluids # Work is _____ between the system and the surroundings.
# Which of the following is a Newtonian fluids? Ans. energy
Ans. All of the above # Is a process during which the system remains in equilibrium at all times
# Thermal radiation heating body can be absorb, reflected or transmitted, this is known as Ans. Quasi-equilibrium
Ans. Radiation Conservation law # In the absence of any work interactions between a system and its surrounding, the amount of
# The rate of thermal emitted per unit area of a body is known as: net heat transfer is equal
Ans. Emissive power Ans. to the change in the total energy of a closed system
# States that for any two bodies in thermal equilibrium, the ratio of emissive power to # The constant volume and constant pressure specific heats are identical for
absorptivity are Ans. incompressible substance
Equal. # The velocity of fluid is zero at the wall and maximum at the center because of the
Ans. Kirchoff’s Radiation law Ans. viscous effect
# For steady flow devices, the volume of the control volume is Ans. outlet of condenser
Ans. constant # A coil in series with evaporator that is use to prevent the liquid refrigerant entering the
# The work done in a turbine is _____ since it is done by the fluid. compressor.
Ans. positive Ans. Drier loop
# Reheating process in Brayton cycle, the turbine work will # A type of valve connected from discharge of compressor directly to suction that is normally
Ans. increase closed
# As the air passes through a nozzle, which of the following will increase? And will open automatically only if there is high discharge pressure.
Ans. mach number Ans. solenoid valve
# As the air passes through a diffuser, which of the following will decrease? # Use to increase the capacity of condenser.
Ans. mach number Ans. desuperheating coils
# As the air passes through a nozzle, which of the following will decrease? # Is use to subcooled the refrigerant from the condenser.
Ans. internal energy Ans. liquid subcooler
# As the air passes through a diffuser, which of the following will increase? # Which of the following is NOT a part of low pressure side in refrigeration system?
Ans. density Ans. liquid line
# As the air passes through a diffuser, which of the following will NOT be affected? # Which of the following is NOT a part of high pressure side in refrigeration system?
Ans. entropy Ans. suction line
# After passing through a convergent-divergent nozzle, the temperature of air will: # Which of the following is NOT a part of condensing unit?
Ans. decrease Ans. liquid line
# After passing through a convergent-divergent nozzle, the density of air will: # By subcooling the refrigerant in refrigeration system, the compressor power per unit mass will
Ans. decrease Ans. remains the same
# After passing through a convergent-divergent nozzle, the mach number of air will: # Superheating the refrigerant in refrigeration system without useful cooling, the refrigeration
Ans. decrease effect
# By increasing the temperature source of Carnot cycle, which of the following will not be Per unit mass will
affected? Ans. remains the same
Ans. heat rejected # By subcooling the refrigerant in refrigeration system, the specific volume at compressor suction
# By decreasing the temperature sink of Carnot cycle, which of the following will not be affected? will
Ans. heat added Ans. remains the same
Ans. Exothermic reaction # Pressure loss due to friction at the condenser, the compressor power per unit mass will
# By reheating the steam in an ideal Rankine cycle the heat rejected will: Ans. remains the same
Ans. increase # Which of the following is NOT a type of air-cooled condenser?
# By increasing the boiler pressure in Rankine cycle the moisture content at boiler exit will: Ans. shell and tube
Ans. increase # A type of refrigerant control typically used in household refrigeration.
# Presently the highest steam temperature allowed at the turbine inlet is about ______. Ans. Capilliary tube
Ans. 620°C # Type of condenser that operates like a cooling tower.
# Two most common gases employed in Stirling and Ericson cycles are: Ans. evaporative condenser
Ans. Hydrogen and helium # The major problem of heat pump is
# In most common designs of Gas turbine, the pressure ratio ranges from: Ans. frosting
Ans. 11 to 16 # Dominant refrigerant used in commercial refrigeration system
# In Brayton cycle, the heat is transformed during what process? # The Carnot refrigeration cycle is:
Ans. isobaric process Ans. All of the above
# The fuel injection process in diesel engine starts when the piston_______. # The vapour compression cycle is essentially a reverse of:
Ans. approaches TDC Ans. Rankine vapor cycle
# If the cut-off ratio of diesel cycle increases, the cycle efficiency will: # What can be considered to be theoretically exact and can be used to determine local thermal film
Ans. decrease coefficient at a distance X from the leading edge of an isothermal plate in laminar flow?
# The fuel used in a power plant that is used during peak periods. Ans. Pohlhausen Solution
Ans. liquid # Which one is exact and can be derived from the Pohlhausen correction by setting cube root of
# Typical compression ratio of Otto cycle is Prandtl number equal to 108 and is useful in gases and water?
Ans. 8 Ans. Blausius Solution
# If Joule Thompson coefficient is equal to zero, then the process will become # What another term for skin friction coefficient?
Ans. isothermal Ans. Fanning friction factor
# If the fluid passed through a nozzle its entropy will: # The Darcy friction factor is ________.
Ans. remains the same Ans. 4 times
# Which of the following is NOT a possible location of service valve? # The ______ for submerged bodies in the sum of the skin friction coefficient is?
Ans. Drag coefficient # The tubing is never used when the fluid temperature goes beyond 100 to 175 mm?
# What can predict the average film coefficient along the entire length of laminar flow? Ans. Plastic
Ans. Labarsky-Kauffman Correlation # The tubing to be swaged is damped in a
# When can we say that the laminar flow is “Fully Developed”? Ans. Flaring block
Ans. When the difference between the surface (wall) and the mean fluid temperature is # The hand tools used to lightened and loosen the fitting are
Constant Ans. Wrenches
# Where is the accumulator usually located? # In soldering process, What substance is used to prevent oxidation?
Ans. Before the suction line Ans. Flux
# A device used to remove moisture and foreign particles inside the refrigeration system. # The process of applying molten metal to heated metals is called:
Ans. Filter drier Ans. Soldering
# What is the standard value of the atmospheric pressure at the atmospheric sea level? # What is the physical state of the refrigerant?
Ans. 14.7 psi Ans. Gas
# Requirements are classified according to: # Which of the following appliances uses a finned-coil evaporator?
Ans. Manner of absorbing heat Ans. Window-type airconditioner
# A push or pull on any thing is called: # The operation of condenser is the opposite of that of
Ans. Force Ans. Evaporator
# Anything that takes up space has mass # Which of the following devices maintains the pressure difference between the evaporator and
Ans. Matter the condenser?
# Factor use in calculating the overall heat transfer through the tube walls of the condenser tube of Ans. Flow control
Another heat transfer surface. # What device is ideal for measuring the flow of liquid metal?
Ans. Fouling factor Ans. Magnetic flow meter
# What could be the possible cause of the abnormal discharge temperature? # The frictional force that acts parallel but opposite to the direction of motion.
Ans. All of the above Ans. Drag
# Where is the relief valve located? # A pump that transform kinetic energy into fluid static pressure.
Ans. At the discharge side of the compressor Ans. Kinetic pump
# What will you do before starting a refrigeration system? # Due to friction losses between the fluid and the pump and mechanical losses in the pump itself.
Ans. Vent the condenser Ans. The brake pump power will be greater than hydraulic power
# What is the purpose of the expansion valve bypass? # The net energy actually transferred to fluid per unit time is:
Ans. To control the refrigerant to the evaporator in case the automatic valve fails. Ans. Hydraulic power
# How many feed water lines are connected to the boiler? # The input power delivered by motor to pump.
Ans. 2 Ans. Brake pump power
# Exhaust gases from the engine possesses what type of energy? # Boiler tubes are specified by
Ans. Kinetic energy Ans. Outside diameter
# What is the instrument used to measure density? # The maximum size of the boiler down lines valves and fittings shall not exceed
Ans. Hydrometer Ans. 2.5 inches
# Engine overall efficiency is ________. # The purpose of an expansion tank in a hot water heating system is to provide for the expansion
Ans. Brake thermal efficiency of
# What is the ratio of the fuel usage rate to the power generated? Ans. Water
Ans. Specific fuel consumption # Heavy accumulation of a soot in the boiler will result in:
# What is the ratio of the air mass that enters the engine to each mass of fuel? Ans. Loss of boiler efficiency
Ans. Air-fuel ratio # How are tubes secured in a fired tube boiler will result in:
# The ohmmeter is an instrument used to measure Ans. Rolled and beaded over
Ans. Resistance # A tube in a fire tube boiler is surrounded by:
# The ammeter is an electrical instrument used to measure Ans. Water
Ans. Current # What is the formula for the factor of evaporation?
# The electrical pressure that pushes the electrical current or objections. Ans. H-b/970.3
Ans. Voltage # What can cause a super heater tube to overheat?
# The voltmeter is an instrument used to measure: Ans. Dirty tube
Ans. Voltage # How many square feet of heating surface does a 3-in fire tube 20 ft long with a 3/16 in . wall
# When the winding is shorted, what is the resistance reading? have?
Ans. Zero ohm Ans. 13.7
# Which tools are used to enlarge the end of the tubing to connect a fitting? # In a high temperature hot water boiler; what could cause a tube to over heat?
Ans. Flaring tools Ans. Poor circulation
# A down corner is: # The ideal cycle based on the concept that the combination process in both diesel and
Ans. Found in a water tube boiler gasoline in
# Extreme firebox temperature changes cause: the combination of heat transfer process that is constant pressure and constant volume.
Ans. Spalling Ans. Dual cycle
# Low water cu. off: # The ratio of the volume at the end of heat addition to the volume at the start of heat
Ans. Shuts the burner when the boiler is low on water addition is called:
# Steam coming from the bottom by cock would indicate: Ans. Cut-off ratio
Ans. Low water level # A theoretical body which when heated to incandescence would emit continuous light
# Why is the method to cool water called condenser not practical? ray spectrum.
Ans. Water is expensive Ans. Blackbody
# A device use to collect liquid refrigerant entering the compressor to prevent trouble in # Which of the following is the reason for insulating the pipes?
compressor is Ans. Heat loss from the surface is minimize
Ans. Accumulator # Heat transfer due to density differential
# A disiccant is a material that has high affinity to water. Which is the common used Ans. Convection
dissicant # A process involving a gas that cannot, even in principle be reserved with being done.
Ans. Silica gel Ans. An isobaric compression
# The only means of preserving food in its original fresh state is _____________. # The maximum amount of heat that can be converted into mechanical energy
Ans. Refrigeration Ans. Depends on the intake and exhaust temperature
# PVC means # The maximum amount of mechanical energy converted into heat
Ans. Poly Vinyl Chloride Ans. 100%
# What does ABS means? # The work output of energy heat engine.
Ans. Acrylonitrite Butadiene Styrene Ans. Equals the difference between its heat intake and heat exhaust
# What measures the average roughness of imperfection inside the pipe? # To increase the output of a centrifugal pump, you must
Ans. Specific roughness Ans. Speed of rotation
# Darcy factor, friction factor is not constant but decreases as the Reynolds number increases # By superheating the refrigerant in vapor compression cycle with useful cooling, which
up to a certain point known as: of the following will increase? (Use per unit mass analysis)
Ans. Full turbulent flow Ans. Heat rejected from condenser
# What presents the friction factor graphically as 6 functions of Reynolds number and # By superheating the refrigerant in vapor compression cycle with useful cooling, which
relative roughness. of the following will decrease? (Use per unit mass analysis)
Ans. Moody diagram Ans. Mass flow rate
# If the flow in truly laminar, and fluid flowing in a circular pipe then which of the equation # By superheating the refrigerant in vapor compression cycle with useful cooling, which
is appropriate to use? of the following will decrease? (Use per unit mass analysis)
Ans. Hagen Poisenille Equation Ans. COP
# Which is true about Hagen Williams equation? # By superheating the refrigerant in vapor compression cycle with useful cooling, which
Ans. It is primarily use for water of the following will increase? (Use per unit mass analysis)
# For highly turbulent flow, what causes shear stress? Ans. Compressor power
Ans. Momentum Effect # By superheating the refrigerant in vapor compression cycle with useful cooling, which
# In a circular pipe laminar flow momentum flux is: of the following will not be affected? (Use per unit mass analysis)
Ans. Maximum at the pipe wall Ans. Compressor power
# If the head pressure is too high # By sub-cooling the refrigerant in 50apour compression cycle at condenser exit, which of the
Ans. The high pressure cut-out switch should operate following will increase? (Use per unit mass analysis)
# In a combustion engine cycle, what is the ratio of the net output power to the input? Ans. Refrigerating effect
Ans. Thermal efficiency # By sub-cooling the refrigerant in 50apour compression cycle at condenser exit, which of the
# In a reciprocating engines, what is the ratio of actual to ideal volumes of entering gases? following will decrease? (Use per unit mass analysis)
Ans. Volumetric efficiency Ans. Mass flow rate
# In a combustion engine, what is the ratio of the actual power developed to the ideal power # By increasing the vaporizing temperature in vapor compression cycle, which of the following
developed? will increase? (Use per unit mass analysis)
Ans. Mechanical efficiency Ans. COP
# The equivalent of ratio of emissive power to absorptivity for both thermal equilibrium is # By increasing the vaporizing temperature in 50apour compression cycle, which of the
described following will decrease? (Use per unit mass analysis)
by: Ans. temperature difference between evaporator and compressor
Ans. Kirchoffs law # By increasing the condenser pressure in 50apour compression cycle, which of the following
will increase? (Use per unit mass analysis)
Ans. Compressor power Ans. Actual velocity divided the theoretical velocity
# By increasing the vaporizing temperature in 51apour compression cycle, which of the # Flow measuring devices include all of the following except:
following will decrease? (Use per unit mass analysis) Ans. Magnetic dynamometers
Ans. moisture content after expansion # In the series pipe systems, all of the following parameters vary from section to section except
# If the pressure drop in the condenser increases in a vapor compression cycle, which Ans. Mass flow
of the following will increase? (Use per unit mass analysis) # The coefficient of velocity accounts for the:
Ans. heat rejected in the condenser Ans. Small effect of friction and turbulence of the orifice
# If the pressure drop in the condenser increases in a 51apour compression cycle, # Expansion factors take into account the
which of the following will decrease? (Use per unit mass analysis) Ans. Effects of compressibility
Ans. mass flow rate # The water hammer phenomenon is primarily what kind of fluid mechanics?
# If the pressure drop in the condenser increases in a 51apour compression cycle, Ans. Dynamic (a time-dependent phenomena)
which of the following will not be affected? (Use per unit mass analysis) # All of the following are forms of drug on a body moving through a fluid except:
Ans. compressor power Ans. D’Alembert’s paradox drug
# If the pressure drop in the evaporator increases in a 51apour compression cycle, # The function of a turbine is to:
which of the following will increase? (Use per unit mass analysis) Ans. Extra energy from the flow
Ans. heat rejected in the condenser # The fact that there is no bodies moving through an ideal fluids is known as:
# If the pressure drop in the condenser increases in a 51apour compression cycle, Ans. D’Alembert’s paradox
which of the following will decrease? (Use per unit mass analysis) # Liquids and gases takes the following characteristics of their contents.
Ans. COP Ans. Shapes
# By lowering the condenser pressure in Rankine cycle, which of the following will # All of the following dimensionless parameters are applicable to fluid flow problem excepts
decrease? (Use per unit mass analysis) Ans. Bolt number
Ans. heat rejected # All of the following can be characteristics of fluids except:
# By increasing the boiler pressure in Rankine cycle, which of the following will Ans. Hysteresis
decrease? (Use per unit mass analysis) # The most common method for calculating frictional energy loss for laminar flowing fluid is
Ans. heat rejected non- circular pipes is:
# By superheating the steam to a higher temperature in Rankine cycle, which of the Ans. The Darcy equation
following will decrease? (Use per unit mass analysis) # For computation convenience, fluids are usually classed as
Ans. moisture content at the turbine exhaust Ans. Real and ideal
# By superheating the steam to a higher temperature in Rankine cycle, which of the # Which of the following is not a characteristics of real fluids?
following will increase? (Use per unit mass analysis) Ans. Experience of eddy currents and turbulence
Ans. cycle efficiency # Property of a fluid whereby its own molecules are attracted is known as:
# By superheating the steam to a higher temperature in Rankine cycle, which of the Ans. Cohesion
following will decrease? (Use per unit mass analysis) # The term subsonic flow refers to a flowing gas with a speed.
Ans. pump work Ans. Less than the local speed of sound
# What type of boiler incorporates furnace water cooling in the circulatory system? # The difference between stagnation pressure and total pressure
Ans. Integral-furnace boiler Ans. None of the terms are interchangeable
#The main components of a combined cycle power plant are: # The presence of friction in the hydraulic grade line will always cause the line to slope.
Ans. Gas turbine and waste heat boiler Ans. Down in the direction of the flow
# Which of the following indicators is used to determine the anti-knocking characteristics of #The presence of minor loss in the energy grade line will cause the line to slope
gasoline? Ans. Down in the direction of the flow
Ans. Octane number # If the Mach number is greater than 1 but less than 5
# Indicate the false statement Ans. Supersonic
Ans. The heat transfer cannot exceed the work done. # The flow is called sonic when mach number is
# In fluid flow, linear momentum is: Ans. Equal to 1
Ans. A vector quantity equal to the product of mass and velocity # The flow is sub-sonic when mach number is
# A fact that a fluid’s velocity increases as the cross-sectional area of the pipe through which it Ans. Less than 1
flow decreases due to: # To check water level in the gauge glass of a steam boiler
Ans. The continuity equation Ans. Use the
# The coefficient of contraction is the ratio of: # The flame failure control is tested by?
Ans. Area of vena contracta to the orifice area Ans. Shutting off the fuel supply to the burner
# The coefficient of discharge is the ratio of: # During purge cycle what is the minimum amount air volume changes required?
Ans. Actual discharge to the theoretical discharge Ans. Four (4)
# The coefficient of velocity is the ratio of: # The purge cycle time for gas compared to oil is:
Ans . The same Ans. 90 degrees
#A high (CO) Carbon monoxide reading, indicates: # Most pressure in the Freon system have two dials or graduation on 1 gage what does the two
Ans. None of these dial represent:
# The induced draft fan is located: Ans. Temperature and pressure
Ans. In the breaching # The flame detector is directed:
# A balanced draft boiler required: Ans. Into the furnace area
Ans. Both forced and induced draft #Which of the following statement is true?
# What percent would expect in a well maintain boiler burning number 6 oil? Ans. All of the above
Ans. 15% #Which of the following is a pseudo plastic fluid?
# Laminar friction factor of fluid is flowing through a pipe is a function of all the following Ans. All of the choices
except: # A type of fluids which are capable of indefinitely resisting a small shear stress but move
Ans. Pipe roughness easily when the stress becomes large:
# The stream function is a useful parameter in describing: Ans. Bingham fluids
Ans. Conservation of mass # Which of the following is an example of Bingham fluids?
# The study of the practical law of fluid and the resistance of open pipes and channels is the Ans. All of the above
_____ # The Bingham fluids will become pseudo plastic when:
Ans. Hydraulics Ans. The stress increases
# The most common methods of calculating frictional energy loss for laminar flow fluids in # A type of fluids which exhibits viscosities that increases with an increasing velocity
non circular pipes: gradient.
Ans. Equation Ans. Dilatant fluid
# The parameter in the expression for head loss is: # The viscosity decreases with time the fluids is said to be:
Ans. Darcy friction factor Ans. Thixotropic fluid
# The characteristic length of Reynolds number use to calculate the friction in non-circular full # Viscosity increases with time the fluids is said to be:
running pipes is based on the: Ans. Rneopectic fluids
Ans. Hydraulic diameter # What is the dominating cause of viscosity in fluids:
# The hydraulic radius of a non-circular pipe is: Ans. Molecular cohesion
Ans. The ratio of flow area to wetted parameter: # In a compressor, the piston is said to be at _____ when it is at its most petracted position in
# An electromagnetic radiation wavelength is in the 0.1 to 10.000 mm range: the stroke,
Ans. Thermal radiation Ans. Bottom dead center
# What characteristics makes the difference between a Newtonian and non-newtonian fluids? # Reciprocating compressors are characterized by their
Ans. Their viscous behaviour Ans. Clearance volume
# What instrument use to measure salt solution: # The gases remaining in the clearance volume after the discharge in the dead center are
Ans. Salimeter known as the:
# A measure of fluid resistance to flow? Ans. Residual gases
Ans. Viscosity # The ratio of the clearance volume to the swept volume is known as:
# What is another for absolute viscosity? Ans. Percent clearance or just clearance
Ans. Dynamic # The work done in an adiabatic (isentropic, process is _____ the work done by the isothermal
# What is the reciprocal of viscosity? process;
Ans. Fluidity Ans. Less than
# The density of fresh water is a ship will float. # The work done in an isentropic compressor is ____ the work done by the isothermal
Ans. Lower in fresh water than in sea warer compressor;
# Bernoulli’s equation is based on? Ans. Greater than
Ans. Conservation of energy # The term used to describe a partially compressed gases withdrawn cooled and compressed
# An express train goes past a station flatform at high speed a person standing in the edge of further;
the flatforms tend to be: Ans. Intercooling
Ans. Attracted to the train # A perfect intercooling refers to the case where the gases cooled to:
# The volume of fluid flowing per second out of an orifice at the bottom. A tank does not Ans. The original inlet temperature
depend on: # Multi stage blowers can reach pressure up to ____ of water;
Ans. Density of fluid Ans. 100 inches
# Addition of detergent to water? # What represents the ratio of the coefficient of the friction at the expanded in pumping a
Ans. Decreases its surface tension fluid;
# Water neither rise or falls in silver capillary. This suggest that the contact angle between the Ans. Fried heat transfer efficiency factor
water and silver is: # What is the proper oil storage temp for No. 6 oil?
Ans. 20 ºF above pour point # The most widely used absorption system is the ammonia-water system, where ammonia
# To prime a pump serves as refrigerant and H2O as the
Ans. Fill casing with water Ans. Transport medium
# Balance draft means; # Known as drum less boiler.
Ans. Almost atmosphere pressure in the furnace Ans. Once-through boiler
# A gallon of No. 6 oil contains: # Reduces the steam temperature by spraying low temperature water from boiler drum.
Ans. 152,000 Btu Ans. Desuperheater
# What is the efficiency of a steam plant with a heat rate of 12,000 Btu/KW? # Carbon dioxide can be removed by;
Ans. 28.4% Ans. Aeration
# What type of fuel must be preheated to burn properly? # Is often used to absorb silica from water.
Ans. Heavy oil Ans. Magnesium hydroxide
# The fuel oil for mechanical atomizing burners is usually heated # Presence of excess hydrogen ions makes the water
Ans. 150 ºF Ans. Acidic
# Fuel suppliers specify a minimum flash point for their oil because a low flash point oil is; # PH of water varies with
Ans. Dangerous Ans. Temperature
# How many cubic feet of furnace volumes are needed to burn one gallon of No. 6 oil? # PH value of ____ is usually maintained for boiler water to minimize corrosion.
Ans. 10 Ans. 10.5
# A fuel oil is heated its viscosity # What type of turbine that has a degree of reaction of 1/2?
Ans. Decreases Ans. Rarsons turbine
# Only ____ of the turbine work output is required to operate the pump. # The cooling water is made to fall in series of baffles to expose large surface area for steam
Ans. 0.04% fed from below to come in direct contact.
# Superheating the steam to higher temperature decreases the moisture content of the steam at Ans. Barometric condenser
the __ # Show the variatiom of river flow (discharge) with time.
Ans. Turbine exit Ans. Hydrograph
# Regeneration also provides a convenient means of dearating the feedwater to prevent. # The pressure at the bottom of a vessel filled with liquids does not depend on the;
Ans. Boiler corrosion Ans. Area of the liquid surface
# Can be apply in Steam turbine cycle (Rankine), Gas turbine cycle (Brayton) and Combined # A person stands on a very sensitive scale, inhales deeply the reading on the scale:
cycle. Ans. Depends on the expansion of the person’s chest, relative to the volume inhaled.
Ans. Cogeneration plant # Buoyancy occurs because, with increasing depth in a fluid:
# In a Rankine cycle with fixed turbine inlet conditions. What is the effect of lowering the Ans. The pressure increases
condenser pressure, the heat rejected will; # In order for an object to sink when placed in water its average specific gravity must be;
Ans. Decrease Ans. More than 1
# In an ideal Rankine cycle with fixed boiler and condenser pressures. What is the effect of # The average solar energy heating the outer edge atmosphere is approximately 442 Btu per
superheating the steam to a higher temperature, the pump work input will; hour-foot squared of 1.41KW per meter squared is known as:
Ans. Remains the same Ans. Solar constant
# How do the following quantities change when the simple ideal Rankine cycle is modified # The ratio of total dissolve solids in the recirculating water to the total solids in the make-up
with regeneration? The heat rejected will; water.
Ans. Decreases Ans. Cycles of concentration
# During a combustion process, the components which exist before the reaction are called # What is the another term for Bryton cycle:
Ans. Reactants Ans. Joule cycle
# Is an obvious reason for incomplete combustion. # The process of transferring heat from low temperature area to high temperature area;
Ans. Insufficient oxygen Ans. Refrigeration
# Higher heating value when H2O in the product of combustion is in #The device to remove heat from water:
Ans. Liquid form Ans. Chiller
# Device which transfer heat from low temperature medium to a high temperature one is a # What device which transfer heat from low temperature area to operate on refrigeration
Ans. Heat pump system:
# A rule of thumb is that the COP improves by _____ for each °C the evaporating temperature Ans. Heat pump
is raised or the condensing temperature is lowered. # Refers to the process of one substance mixing with another substance:
Ans. 2 to 4% Ans. Diffusion
# Are generally more expensive to purchase and install than other heating systems, but they # Which of the following statement about Newtonian fluid is most accurate:
save money in the long run, Ans. Shea stress is proportional to the rate of strain
Ans. Heat pumps # Which is not characteristic of fluids?
Ans. Hysteresis
# The relationship between pressure and altitude in the atmosphere: # Pitot tube used to measure
Ans. Barometric height relationship Ans. Velocity
# Flash point means: # Liquid gases take the following characteristics of the containers:
Ans. Ignition temperature Ans. Shape
# Fire point means: # For computational convenience usually classed as:
Ans. None of the above Ans. Real and ideal
# Bernoulli’s equation is a # Which of the following statement about Newtonian fluid is more accurate?
Ans. Conservation of energy equation Ans. Shear stress is proportional to the rate of strain
# The pressure at given depth due to several immiscible liquid is: # Which of the following is not a characteristic of a real of fluid?
Ans. The sum of individual pressure Ans. Experience of eddy current and turbulence
# The relationship between pressure and altitude in the atmosphere is given by the # One could expect the possibility of Reynold’s number similarity in all of the following cases
Ans. Barometric height relationship except
# The fact that the buoyant force on a floating object is equal to the weight of the water Ans. Closed pipe turbulent flow
displaced # One could expect the possibility of Froude number similarity in all of the following cases
Ans. Archimedes principle except
# Which of the following terms does not appear in the steady flow energy equation in the Ans. Subsonic airflows
extended Bernoulli’s equation # The absolute viscosity of a fluid varies in pressure and temperature as a function of;
Ans. Hysteresis losses Ans. Shear and angular deformation rate
# The pitot tube can be use to measure fluid velocity as described by the Bernouilli equation # Turbulent flow of a fluid in a pipe. All of the following except;
and the relationship between; Ans. Reynold’s number will be less than 2300
Ans. Kinetic energy and static pressure # Flow measuring devices include all of the following except;
# The difference between stagnation pressure and total pressure is; Ans. Magnetic dynamometer
Ans. None. The terms are interchangeable # Flow measuring devices include all of the following except;
# Fully turbulent flow in a pipe is characterized by all the following except: Ans. Mercury barometer
Ans. Parabolic velocity profile # Flow measuring devices include all of the following except;
# What is the bulk modulus of the water is; Ans. Precision tachometer
Ans. 300,000 psi # The following are all examples of indirect (secondary) measurements to measure flow rates
# Atmospheric pressure does not correspond to approximately; using obstruction meters except;
Ans. 98 N/m2 Ans. Volume tanks
# Hydraulic press is able to produce a mechanical advantage because; # The following are all examples of indirect (secondary) measurements to measure flow rates
Ans. An external pressure extended on a fluid is transmitted uniformly through its using velocity meters except;
volume Ans. Weight and mass scales
# In the operation of a hydraulic press, it is impossible for the output piston to exceed the input # The following are all examples of indirect (secondary) measurements to measure flow rates
piston using flow meters except;
Ans. Work Ans. Positive displacement meters
# If one of the management legs is inclined it is known as: # In series pipe systems which of the following parameters vary from section to section
Ans. Draft gauge except;
# Why are manometer tube generally large in diameter? Ans. Mass flow
Ans. To avoid significant capillary effect # Venturimeter, pitot static gauges, flow nozzles, manometer vary with;
# Bernoulli’s equation is an energy conservation based on several reasonable assumptions: Ans. Flow velocity and pressure
Ans. All of the above # Other name for R 729?
# Body that emits a constant emissivity, regardless of the wavelength; Ans. Air
Ans. Gray body # In an indirect refrigeration system, which of the following is used to measure the density of
# What gives the total emissive power from a black body brine?
Ans. Stefan-Boltzmann law of the fourth power Ans. Hydrometer
# What accounts for the spatial arrangement of the body and their emissivity; # Which of the following is the result of opening a hand expansion valve too much in a
Ans. Emissivity factor refrigeration system?
# The gray body shape factor is the product of the Ans. Evaporator will “freeze back” to compressor
Ans. Black body shape factor and emissivity factor If one of the manometer legs is inclined, it is known as:
# The product of the area and the shape factor is known as; Ans. Draft gauge
Ans. Geometric flux Why are monometer tubes generally large in diameter?
# Surfaces that re-radiates absorb thermal radiation are known as; Ans. To avoid significant capillary effect
Ans. Refractor materials of refractories
Bernoulli’s equation is an energy conservation based on several reasonable assumptions: In series pipe systems which of the following parameters vary from section to section except:
Ans. All of the above Ans. Mass flow
Body that emits a constant emissivity, regardless of the wavelength: Venturimeter, pitot static gauges, flow nozzles, manometers vary with:
Ans. Gray body Ans. Flow velocity and pressure
What gives the total emissive from a black body: Other name for R 729?
Ans. Stefan-boltzman law of the fourth power Ans. Air
What accounts for the spatial arrangements of the body and their emissivity? In an indirect refrigeration system, which of the following is used to measure the density of
Ans. Emissivity factor the brine?
The gray body shape factor is the product of the Ans. Hydrometer
Ans. Black body shape factor and emissivity factor Which of the following is the result of opening a hand expansion valve too much in a
The product of the area and the shape factor is known as: refrigeration system?
Ans. Geometric flux Ans. Evaporator will “freeze back” to compressor
Surfaces that re-radiates absorb thermal radiation are known as: Types of turbine used up to 300m head.
Ans. Refractory materials of refractories Ans. Deriaz turbine
Pito- tube used to measure A turbine that has a diagonal flow.
Ans. Velocity Ans. Deriaz turbine
Liquid gases take the following characteristic of the containers: Oil is atomized either by air blast or pressure jet at about
Ans. Shape Ans. 70 bar
For computation convenience usually classed as: Type of solid injection that use single pump supplies fuel under high pressure to a fuel
Ans. Real and ideal header.
Which of the following statement about Newtonian fluid is more accurate? Ans. Common rail injection
Ans. Shear stress is proportional to the rate of strain Water flow in diesel engine that is caused by density differential.
Which of the following is not a characteristic of a real fluid? Ans. Thermosiphon cooling
Ans. Experience of eddy current and turbulence Type of lubrication system in diesel engine in which oil from pump is carried to a separate
One could expect the possibility of Reynold’s number similarity in all of the following cases storage tank outside the engine cylinder and used for high capacity engine.
except Ans. Dry sump lubricating system
Ans. Closed pipe turbulent flow Produces extreme pressure differentials and violent gas vibration
One could expect the possibility of Froude number similarity in all of the following cases Ans. Detonation
except In a spark ignition engine, the detonation occurs near the______________.
Ans. Subsonic airflows Ans. End of combustion
Difference between energy grade line (friction) and the energy line friction loss is: In a compression ignition engine, the detonation occurs near the _____________.
Ans. Friction and minor losses Ans. Beginning of combustion
The absolute viscosity of a fluid varies in pressure and temperature as a function of: Morse test is use to measure the _________ of multi-cylinder engine.
Ans. Shear and angular deformation rate Ans. Indicated power
Turbulent flow of a fluid in a pipe. All of the following are true except: Ignition delay can be minimized by adding ______ to decrease engine knocking.
Ans. Reynold’s number will be less than 2300 Ans. Ethyl nitrate
Flow measuring devices include all of the following except: The work done in throttling valve is
Ans. Mercury barometer Ans. Zero
Flow measuring devices include all of the following except: Stagnation enthalpy represents the enthalpy of a fluid when it is brought to rest ______.
Ans. Precision tachometer Ans. Adiabatically
The following are all examples of indirect (secondary) measurements to measure flow rates Represents the temperature an ideal gas attains when it is brought to rest adiabatically.
using obstruction meters except: Ans. Stagnation temperature
Ans. Volume tank After passing through a nozzle the density of fluid decreases as the fluid velocity.
The following are all examples of indirect (secondary) measurements to measure flow rates Ans. Increases
using velocity meters except: The volume flow passes through a venture meter will
Ans. Weight and mass scales Ans. Constant
The following are all examples of indirect (secondary) measurements to measure flow rates A converging-diverging nozzle is the standard equipment in
using flow meters except: Ans. Supersonic aircraft
Ans. Positive displacement meter Nozzles efficiencies range from
Ans. 90%-99% Ans. Varies with compression ratio
By reheating the steam before entering the second stage in Rankine cycle, which of the The useful energy transfer in Btu/hr divided by input power in Watts;
following will increase? Ans. Energy efficiency ratio
Ans. Heat rejected If air is humidified by injecting steam of by pressuring the air through a hot water spray, the dry
Contains 90% gasoline and 10% ethanol bulb temperature and enthalpy of air
Ans. Gasohol Ans. Increase
In an evaporator section in refrigerating unit, which of the following is the function of Air passing through a solid or absorbed bed, such as silica gel or activated alumina will:
evaporator? Ans. Decrease in humidity
Ans. Absorption of latent heat of vaporization A device that passes air through dense spray of recirculating water;
Which of the following may not be the type of bulb in R-12 refrigeration system? Ans. Air washer
Ans. Duplex The mass of water sprayed to the mass of air passing through the washer per unit time.
Which of the following shuts down the compressor motor when discharge pressure rises to a Ans. Spray ratio
predetermined point, it cuts in motor when pressure drops to predetermined pressure: When the cooling tower is used to provide cold water for the condense of a refrigerator system
Ans. High pressure cut-off switch the water circulation system will be approximately:
Which of the following shuts down the compressor motor when discharge pressure rises to a Ans. 3 gal/min-ton
predetermined pressure and shuts down the compressor motor when there is decrease in Water lost in small droplets and carried away by the air flow:
pressure in the suction line. Ans. Drift
Ans. Low pressure cut-off switch The ratio of total dissolved solids in the circulating water to the dissolved solids in the make-up
The P.O. service pump must have: water
Ans. A means of shutting down from outside the machinery space Ans. Ratio of concentration
The relief valve on the P.O. transfer pump discharge Through windage removes some solids, most must be removed by bleeding some of the water.
Ans. Back to the transfer pump discharge Ans. Bleed-off
When heating hot water, feed pump should at least be how many mm to prevent vapor lock? A dry cooling tower where stream travels through large diameter trunks to a cross-flow heat
Ans.1220 mm exchanger where it is condensed and cooled by air;
The water column should be provided with a valve drain at least: Ans. Direct condensing tower
Ans. 15 mm pipe size An economizer is used to:
Lubricating oils have flash points from Ans. Heat the feedwater before it enters the boiler
Ans. 375-800 degree Fahrenheit A manometer measures:
A device receives information in one form of instrument signal and transmits an output in another Ans. Air pressure
form; A pyrometer measures:
Ans. converter Ans. Stack pressure
Refinement petroleum Naphtha which by its composition is suitable for use in internal An attemperator is another name for a
combustion engine; Ans. Desuperheater
Ans. Gasoline Waterwalls are used to:
It is possible for water to get into the fuel oil by: Ans. Carry of excess heat from the furnace walls
Ans. All of the above The studs on waterwalls tubes are to:
Air chambers are attached to the: Ans. Give added surface area to tubes
Ans. Discharge side of the reciprocating pumps The heats remove from the cold area
On automatic combustion control: Ans. Use for energy transfer
Ans. Reduce the number of burners when the oil pressure approaches minimum On the upstroke of the piston, the low pressure vapor is first compressed and then discharged
specified as high pressure vapor through the discharged valves into the:
The volumetric efficiency of the compressor increases as the suction pressure: Ans. Head of the compressor
Ans. Increase The vapor that remains the clearance space at the end discharge stroke is called
As the discharge pressure increases, the volumetric efficiency: Ans. Clearance vapor
Ans. Increases Which of the following steps would you take if you found an accumulation of oil on the furnace
When the suction And discharge pressure are varied in such a direction that the compression floor?
ratio is increased, the volumetric efficiency of the compressor: Ans. Open all registers
Ans. Decrease Which of the following method can be used to clean up the inside of the boiler?
For a compressor of any given clearance, the volumetric efficiency Ans. All of the above
Another term for the oil discharge strainer is the: What would you do before giving a boiler bottom blow off?
Ans. Hot strainer Ans. Raise the water level
In a controlled feed water system the power water level maintained by: When installing a new gauge glass in water gauge, you should secure the bolts from?
Ans. The fluctuation water level in the boiler Ans. Center, alternately, toward each end
The difference between the enthalpy of the product at a specific state and the enthalpy of One of the main purpose of refractories in the boiler furnace is to:
reactants at the same state for a complete reaction: Ans. Prevent excessive furnace heat losses
Ans. Enthalpy of reaction A fuel oil service pump steam valves are fitted with:
The amount of heat released during a steady flow combustion process when I kg of fuel burned Ans. Each rod leading to a location outside the fire room
completely at a specific temperature and pressure. Water tube boilers have
Ans. Enthalpy of combustion Ans. No fusible plugs
The enthalpy of a substance at a specific state due to its chemical composition The valve that prevents water from backing out of the boiler in the feed water line:
Ans. Enthalpy of formation Ans. Feed check valve
The inlet side of condenser tube are rolled and flared to: The fuel oil heater is located
Ans. Allows a smoother entrance flow of circulating water Ans. On the discharge side of the service pump
The purpose of inspection plates on the main condenser is to: In the forced draft system:
Ans. Check tubes without removing condenser heads Ans. One fan supplies air to all furnaces
A pump that has one steam cylinder and two liquid cylinders is called a: In the closed fire room system
Ans. Tandem pump Ans. The fire room is supplied with air from one fan
A mixture of dry air and superheated water vapor The air cock on a boiler is located at the:
Ans. Unsaturated air Ans. Highest point of the steam and water drum
When the air is unsaturated, what is the relation between the dry bulb temperature and wet Soot blowers should be used in proper sequence so that
bulb temperature? Ans. The soot will be upright toward the uptake
Ans. Less than What is the first thing you would check on taking over a watch?
The difference between the dry bulb temperature and wet bulb temperature Ans. The water level
Ans. Wet bulb depression A boiler with a fan/blower located in the uptake is operating on
The relative ratio of water vapor to dry air is: Ans. Induced draft
Ans. Humidity ratio A fire room that is isolated(closed) operates on:
Also known as humidity ratio: Ans. Forced draft
Ans. Specific humidity How many feed water lines are connected to the boiler?
The ratio of the actual humidity ratio to the saturated humidity ratio: Ans. Two
Ans. Degree of saturation If the water in the gauge glass has not been blown for a period of time, the level of water in the
Also known as saturation ration: glass will be:
Ans. Percentage humidity Ans. less than that in the steam and water drum
The partial pressure of the water vapor divided by the saturation pressure: The enthalpy of fluid when it is brought to rest adiabatically
Ans. Relative humidity Ans. Stagnation enthalpy
Rapidly increasing the temperature through the air of a chord will cause the water in the gauge In the absence of any heat and work and any changes in potential energy, the stagnation
to enthalpy of a fluid during a steady flow process:
Ans. Evaporate Ans. remains constant
Since the increase in the water vapor a latent heat contents equals the decrease in the air’s During the stagnation process, which of the following is converted to enthalpy?
sensible heat, the total enthalpies before and after adiabatic saturation is: Ans. Kinetic energy
Ans. Equal The cross section of a nozzle at the smallest flow area is called:
An adiabatic saturation process follows a line of constant Ans. Throat
Ans. Dew point temperature The properties of a fluid at a location where the Mach number is unity are called:
Which of the following used with adiabatic saturation process? Ans. Critical properties
Ans. Saturation efficiency What is the average fuel oil temperature range of the oil in the discharge line?
The saturation efficiency of a large commercial air washer is typically Ans. 180-200 degree F
Ans. 80%-90% Which of the following is a common type of oil burner?
The presence of oil or scale on the boiler tubes causes: Ans. All of the above
Ans. Overheating The boiler gauge glass should be blown down.
Ans. At the beginning of every watch The cross section of a nozzle at the smallest flow area is called:
Gauge pressure of 200 pounds is equivalent to what absolute pressure? Ans. Throat
Ans. 215 The properties of a fluid at a location where a Mach number is unity are called:
In a fuel oil service system, the quick closing fuel oil shut-off valve is located between the: Ans. Critical properties
Ans. Master fuel oil shutoff valve and the boiler What is the average fuel oil temperature range of the oil in the discharge line?
A fuel oil meter placed between the fuel oil service pumps and the fuel oil heaters. Ans. 180-200 degree Fahrenheit
Ans. Cold-type meter Which of the following is a common type of burner?
The presence of oil or scale on boiler tubes causes: Ans. All of the above
Ans. Overheating The boiler gauge glass should be blown down.
What would you do before giving a boiler bottom blow off: Ans. At the beginning of every watch
Ans. raises the water level Gauge pressure of 200 pounds is equivalent to what absolute pressure?
When installing a new gauge glass in a water gauge you should secure the bolts from: Ans. 215
Ans. Center, alternately towards each ends In a fuel oil service system, the quick closing fuel oil shut-off valve is located between the:
One of the main purpose of refractories in the boiler furnace is to: Ans. Master fuel oil shutoff
Ans. Prevent excessive furnace heat losses A fuel oil meter placed between the fuel oil service pump and fuel oil heater
A fuel oil service pump steam valves are fitted with: Ans. Cold-type meter
Ans. Each rod leading to a location outside the fire room When securing a scotch boiler
Water tube boilers have Ans. The belly plug must be removed
Ans. No fusible plugs If the cooling coils temperature is between the airs dew point, the moisture will
The valve that prevents water from backing out of the boiler in the feed water line: Ans. Condense
Ans. Feed check valve What occurs when the air passes through a water spray in an air washer?
The fuel oil heater is located Ans. Evaporative cooling process
Ans. On the discharge side of the service pump To prevent ice buildup, the cooled air temperature should be kept from dropping
In the forced draft system: Ans. Below the freezing point of water
Ans. One fan supplies air to all furnaces What precaution must be taken before using steam soot blowers?
In the closed fire room system Ans. Drain thoroughly
Ans. The fire room is supplied with air from one fan Which of the following would you do before blowing the tubes with steam soot blower?
The air cock on a boiler is located at the: Ans. All of the above
Ans. Highest point of the steam and water drum Which of the following would cause a flareback?
Soot blowers should be used in proper sequence so that Ans. Trying to relight from hot
Ans. The soot will be upright toward the uptake Sputtering oil burners might indicate
What is the first thing you would check on taking over a watch? Ans. Water in the fuel oil
Ans. The water level The boiler feed water in the feed water heater is heated by:
A boiler with a fan blower located in the uptake is operating on Ans. Auxiliary exhaust steam
Ans. Induced draft
A fire room that is isolated (closed) operates on: If the boiler is smoking white smoke, a possible cause would be:
Ans. Forced draft Ans. Too much air
How many feed water lines are connected to the boiler? The Edwards-type air pump has:
Ans. Two Ans. One set of valve
If the water in the gauge glass has not been blown down for a period, the level of water in the A scoop condenser is a:
glass will be: Ans. Single pass condenser
Ans. Less than that in the steam and water The purpose of the steam to baffle in a condenser
The enthalpy of fluid when it is brought to rest adiabatically. Ans. Prevent the steam from hitting directly on the cooler tubes
Ans. Stagnation enthalpy The condenser vacuum feed valve is open and the water in the reserve tank is dry. The result is:
In the absence of any heat and work and any changes in potential energy, the stagnation Ans. A loss of vacuum
enthalpy of a fluid during a steady flow process; The division plate in a two-pass-condenser
Ans. Remains constant Ans. Prevents the circulating water from passing directly to overload discharge
During the stagnation process, which of the following is converted to enthalpy? The holes in Kinghorn-valve disks
Ans. Kinetic energy Ans. B
The equilibrium temperature that a regular thermometer measure if exposed to atmospheric -A scoop condenser is a:
air: Ans. Single pass condenser
Ans. Dry bulb temperature - The purpose of the steam baffle in a condenser is to:
The temperature of air that has gone through an adiabatic saturation process Ans. Prevent the steam from hitting directly on the cooler tubes
Ans. Wet bulb temperature -The condenser vacuum feed valve is open and the water in the reserve tank is dry.
If the vapor pressure equals the saturation process, the air is said to be: Ans. A loss of vacuum
Ans. Saturated -The division plate in a two-pass-condenser
Ans. Prevents the circulating water from passing directly to overload discharge
When the air is saturated, the dry bulb, wet bulb and the dew point temperature are
-The holes in kinghorn-valve discs
Ans. Equal
Ans. B
On taking over a watch, one should check -The equilibrium temperature that a regular thermometer measure if exposed to atmospheric
Ans. All of the above air;
When you are cleaning fuel oil burner tips, use a Ans. Dry bulb temperature
Ans. Brass knife -The temperature of air that has gone through an adiabatic saturation process
The amount of steam generated by a boiler is dependent upon Ans. Wet bulb temperature
Ans. All of the above -if the vapor pressure equals the saturation process the air is said to be
A boiler operated at a pressure not exceeding 1.055 kg/cm2 gauge steam of temperature not Ans. Saturated
exceeding 120oC. -When the air is saturated, the dry bulb, wet bulb and dew point temperature are
Ans. Low pressure heating boiler Ans. Equal
No part of the steam generator should be closer than how many from the wall? -On taking over a watch, one should check
Ans. 1 m Ans. All of the above
Steam generator should be mounted over a suitable foundation or concrete not less than 305 -When you are cleaning fuel oil burner tips, use a
mm thick and with sufficient area at base to be supported the bearing capacity of the soil with a Ans. Brass knife
factor of safety of -The amount of steam generated by a boiler is dependent upon
Ans. All of the above
Ans. Not less than 4
No smoke stacks should be closer than how many millimeters from the exposed woodwork or
framing -A boiler operated at pressure not exceeding 1.055 gauge steam of steam temperature not
exceeding 120oC.
Ans. 305 mm
Ans. Low pressure heating boiler
The air on an extinguisher fire should be
-No part of the steam generator should be closer than how many from the wall?
Ans. Kept closed Ans. 1m
Too low F.O. temperature will cause -Steam generator should be mounted over a suitable foundation or concrete of not less than
Ans. Poor combustion and smoky fires 305 mm thick and with sufficient area at base to be supported the bearing capacity of the soil a
-when securing a scotch boiler factor of safety of
Ans. The belly plug must be remove Ans. Not less than 4
-if the cooling coils temperature is between the airs dew point, the moisture will -No smoke stacks should be closer than how many millimeters from exposed wood work or
Ans. Condense framing?
-what occurs when the air passes through a water spray in an air washer? Ans. 305 mm
Ans. Evaporative cooling process -The air register on an extinguisher fire should be
-to prevent ice build up, the cooled air temperature should be kept from dropping Ans. Kept closed
Ans. Below freezing point of water -Too low F.O. temperature will cause
-What precaution must be taken before using steam soot blowers? Ans. Poor combustion and smoky fires
Ans. All of the above -When the fuel-oil temperature is too high it causes
-Which of the following would cause a flareback? Ans. Carbon deposits on the fuel-oil heater coils
Ans. Trying to relight from hot brickwork -A dry cooling tower where steam is condensed by cold water jets
-Sputtering oil burners might indicate Ans. Indirect condensing dry cooling tower
Ans. Water in the fuel oil -The hot condensate is pumped to cross heat exchangers whose it is cooled by.
-The boiler feed water in the feed water heater is heated by: Ans. Air
Ans. Auxiliary exhaust steam -which of the following is the refrigerant “of choice” in entering air conditioning
-If a boiler is smoking white smoke, a possible cause could be: Ans. R-22
Ans. Too much air -In new equipment, which of the following replaces R-11
-The Edwards-type air pump has; Ans. R-12
Ans. One set of valve
-The super heater is used to Ans. Steam and water drum
Ans. Remove moisture from steam -The purpose of the blow valve is
-Water tube boiler are Ans. Removes oil and scum from surface of water
Ans. Rolled in -The purpose of the salinometer cock is to
-The recirculating valve is used to Ans. Obtain a sample of boiler water for testing
Ans. Recirculate and heat the fuel oil prior to lighting off -Salt in boiler water usually contains from
-The fusible plug in a scotch boiler are found in the Ans. Leaky condenser tubes
Ans. Fire tubes -Before blowing down a boiler always
-Which of the following valves are arrange in a Y-branch fitting? Ans. Open skin valves
Ans. Boiler-drum safety valves -The mass flow rate produced by the compressor is
-The coils in the feedwater heater are secured by. Ans. Equal to the mass of the suction vapor that the compressor takes in the entire suction inlet
Ans. Union fitting unit time
-Natural draft towers can cool the water to within. -In order to increase back pressure, one must
Ans. 10 to 12oF Ans. Close in on the back pressure valve
Force draft towers can cool the water to within -The actual steam that condenses in the feedwater heater
Ans. 5 to 8oF Ans. Discharge to the hotwell
-When you light a fire in the burner, you must always -The excess steam pressure in the back pressure system exhausts to the
Ans. Use a torch Ans. Atmosphere through a relief valve
-The recirculating line on the burner valves returns the oil to the -The air pressure supplied to the boiler is measure in
Ans. Suction side of service pump Ans. Inches of water
-The valve which shuts off liquid line with the least amount of resistance to flow. -The condensate from the coils in the fuel oil heater to the fuel oil heaters goes to the
Ans. Gate valve Ans. Observation tank
-The process applied to iron pipe which retards corrosion is called The valve between the fuel oil heaters and the burner valve is called the
Ans. Galvanizing Ans. Root valve
-Listed in sequence, iron pipe series are; -The safety device located in the crown sheet of a scotch boiler is called a
Ans. ¼ ,3/8, ½, ¾ Ans. Fusible plug
-The two main purpose of the main condenser are; -Tube retarders in scotch boilers are used to
Ans. Convert exhaust steam to water Ans. Slow down the combustion gases
-Pyrometer is a -In Order for oil to burn properly it must
Ans. High temperature thermometer Ans. Al of the above
-The temperature and humidity of the air to be used for comfort cooling shall be maintained at -The steam that is discharge from safety valves goes to the
effective temperature at air movement within the range from Ans. Atmospheric line
Ans. 0.0762 to 0.127m/s -The polytrophic exponent approaches one with the change in the process____approaches
-As the air enters the living zone the air motion in such occupied space and which the only zero
source of contaminants shall have a velocity of not more than Ans. Temperature
Ans. 0.294m/s -The change in kinetic energy of a process ___ as the change in temperature of the process
-In air conditioning and ventilation standards, as the air enters into living zone, the distance path increases
above the floor is Ans. Increases
Ans. 1.603mm -The final temperature of an isobaric process if the system work in a _____Quantity and the
-Carbon dioxide concentration in air when measured 910 mm above the floor shall be Surrounding heat transfer to the system
estimated Ans. Positive
Ans. 1000ppm -As the volume of an isothermal expansion process approaches its first value the rate of
-The useful refrigerating effect equals to 211kj/mm is change in the system pressure.
Ans. Tons of refrigeration Ans. Decreases
-Another name for the fuel oil suction strainer; -As a fluid flow through a pipe , their potential energy
Ans. Cold strainer Ans. Decreases
-In a manually controlled feed water heater system the proper water level is maintained by -The heat of compression___ as suction vapor becomes super heated
Ans. Operating the feed check valve Ans. Increases
-The actual refrigerating capacity of the compressor -The heat content of the refrigeration cycle____ when a liquid- suction line heat exchanger is
Ans. Always less than the theoretical capacity installed
-The ratio of the actual displacement of the compressor to its piston displacement Ans. Remains the same
Ans. Total or real volumetric efficiency -The COP of refrigeration cycle___ with subcooling
-Cyclone super heater are located in the Ans. Increases
Ans. All of the above
-Which defrost method is commonly used small commercial application where the forms on -Frozen storage chamber temperature set points are usually.
the evaporator surfaces? Ans. -20.5 degree C
Ans. Hot gas -Which of the following is not a unit of density?
-Which of the following liquid-chilling evaporator types incorporates overflow heat Ans. Lb/ft3
exchangers? -Wrapping and refrigerating food products extends the storage life of
Ans. Baudelot Ans. Meat
-Which of the following is not a unit of energy? -The approximate life of strawberries is.
Ans. Watt Ans. None of the above
-The unit of special heat are: -Which of the following is not category of refrigeration application?
Ans. Btu/lbm.oF Ans. Transportation air conditioning
-Phase change process are constant________process. -Which of the following is not an industrial application
Ans. None of the above Ans. Meat display cases
-Which of the following sequences accurately indicates the responses that occur when heat is -Which of the following variable is not most often maintain by a commercial air conditioning
transferred from a gas? unit?
Ans. Sensible heat of vapors, latent heat of vaporization, sensible heat of liquids, latent heat of Ans. Temperature
fusion, sensible heat of fluids -Providing clean, filtered air for trouble free operation of equipment and instrumentation is a
-Btu/lbm are units of function of?
Ans. Specific enthalpy Ans. Industrial air conditioning process
-As a liquid changes phase to vapor, its enthalpy -______ is one of the most common application of mechanical refrigeration.
Ans. Increases Ans. Preservation of perishable commodities.
- As a liquid changes phase to a vapor , its entropy -The air conditioning systems for cabins on a luxury ocean liner belongs to the ____ category.
Ans. Decreases Ans. Commercial air conditioning
-The vaporization process that occurs at temperatures below the triple point of a substance Mechanical refrigeration system makes it possible to;
is_____ Ans. All of the above
Ans. Sublimation -As the liquid changes to a vapor. The enthalpy
-The Vaporization process that occurs when the vapor pressure of a substance is equal to the Ans. Increases
atmospheric pressure is: -Which of the following systems has the most complicated oil return system?
Ans. Boiling Ans. Direct staged
-As the pressure of a vapor increases, the amount of work increases and its enthalpy__ -Which of the following process does not after the kinetic energy level of a substance?
Ans. Increases Ans. Fusion
-The entropy of R-134a in a saturated liquid phase at 40 psi is approximately -200 degrees F is equal to
Ans.21 BTU/lb-oF Ans. None of the above
-The latent heat of vaporization of R-134a in its saturated vapor phase at 0 degree C is -An evaporator in a refrigeration unit makes use of which heat transfer mode?
approximately. Ans. All of the above
Ans.196.7Kj/Kg -When a service technician places his/her hand on a suction line to check the operation of a
-The condenser of a commercial display cause that is located within the unit is called system, he/she is using which heat transfer modes?
Ans. B and C Ans. Conduction
-Which of the following characteristics of early refrigeration systems which applies to today’s -Energy added to a vapor is known as Ans. B and C
units. -The rate of changes velocity is called:
Ans. Relative expensive Ans. Acceleration
-Mechanical refrigeration system make it possible in. -Which of the following devices should be used to measure a pressure of 90kpa?
Ans. All of the above Ans. All of the above
-_______is method used to reduce the dehydration of sensitive fruits and vegetable during the -The height of the mercury column in a barometer placed on a mountain with a local pressure
chilling process of 12.5 psi will be?
Ans. Water chilling Ans. All of the above.
-Which of the following is not a step in preparing vegetables for frozen storage? -A compound gauge measuring a pressure of 22.44 inches of mercury is equivalent to an
Ans. Coating with syrup absolute pressure of
-Loss of food juices by osmosis is a consequence of the ______ freezing process. Ans. 11.0 Psia
Ans. Immersion -The electric meter on a home or building measure the amount of that was consumed over the
-Heat transfer that occurs primarily by conduction is used for ____ freezing process. billing period.
Ans. All of the above Ans. Energy
-A disadvantage of sharp freezing is the -Which of the following is not a method of food preservation?
Ans. Grilling -The society that sponsors research on refrigerants is called:
-Which of the following is not plant, animal or fungi? Ans. ASHRAE
Ans. Enzyme -The scale factor for hard water used in a condenser is:
-Which of the following refrigerant group denote a more toxic vapor? Ans. 0.002
Ans. B3 -As the altitude of a forced air-condenser increases, the surface are of the coil or volume flow
-What is the molecular weight of helium in 3600 Btu lb K? rateof the fan must:
Ans. 4.0 lb mol Ans. Increase
-In an isobaric process changes in pressure can be caused by changes in. -As the water temperature entering a water-cooled condenser decrease, the power drawn by
Ans. None of the above the compressor.
-Boyles law states that pressure and volume changes in gas process is ___related. Ans. Decrease
Ans. Inversely -Which of the does not increase the volumetric efficiency of a compressor?
-As a gas is heated in an isothermal gases the volume Ans. Decreasing the discharge pressure
Ans. Increases -What is the compressor operating with a 30 psig suction pressure and a discharge pressure?
-As a gas is heated in an isobaric gases the volume Ans. 3.13
Ans. Decreases -As the length of the tubes in a chiller barrel increases, the pressure drop across the inlet and
-As a gas is heated in an isometric gases the volume outlet of the barrel:
Ans. Remains constant Ans. Increases
-As the volume of a gas decreases, its specific gas constant -_____ decreases the volumetric efficiency of a compressor
Ans. Remains constant Ans. Decreasing the suction pressure
-Vapor do not behave as ideal gases because they experience -The mechanical efficiency of a compressor_____ as the compressor load increases.
Ans. Friction Ans. Remain the same
-The unit of latent heat of vaporization are: -The saturated suction temperature used in the selection of a compressor corresponds to the
Ans. None of the above pressure of the vapor at the
-The unit of latent heat of fusion are: Ans. Inlet of the compressor
Ans. Btu-lbm -A smaller temperature difference between the refrigerant in the liquid line and that in the
-A smaller temperature difference between the refrigerant in the liquid line and that in the evaporator ____ the mass flow rate of the refrigerant needed per ton of refrigerating effect.
evaporator___ the mass flow rate of the refrigerant needed per ton of refrigeration effect. Ans. Decreases
Ans. Decreases -The difference between the saturation pressure in the evaporator and that in the condenser___
-Oil separator are used in system where: as the suction temperature decreases.
Ans. All of the above Ans. Increases
-Which evaporator design incorporates secondary surface heat transfer? -The COP and the efficiency of a refrigeration cycle____ as the vaporizing temperature
Ans. Finned-tube increases
-Eutectic filled plate evaporator have a higher_____ than evacuated plate evaporators: Ans. Increases
Ans. All of the above -The volume of vapor that the compressor must handle per minute per ton____ as the
-Which evaporator circuit configuration is less effective when used in expansion load vaporizing temperature increases.
applications. Ans. Decreases
Ans. Cross-flow -The quantity of heat rejected at the condenser per unit capacity per minute____ as the
-Which of the following evaporators feed methods produces the greatest surface without the vaporizing temperature decreases.
used of a liquid pump. Ans. Decreases
Ans. Flooded -As the amount of scale on the refrigerant tubes of an evaporative condenser increases, the
-What type of force convection unit cooler is used in vegetable storage application that require scale formation of hot surfaces
a velocity of approximately 120m/min? Ans. Decreases
Ans. Medium velocity -As the cycles of concentration in a tower or evaporative increases, the scale formation of hot
-Which of the following is not a hydrocarbon refrigerants surfaces:
Ans. inane Ans. Increases
-Which of the following refrigerants is most hazardous? -The____ can only be effectively employed in industrial process that has constant refrigeration
Ans. R-717 load.
-A mixture of two or more refrigerants is called a___ Ans. Hand expansion valve
Ans. Zoetrope -The device used to transfer forces across a sealed boundary is called:
-Dessicants are used to remove___ from refrigerants Ans. Diaphragm
Ans. Moisture -The device used to measure the effects of a large pressure drop across the evaporator is called
-As an oil-miscible refrigerant mixes with oil, the viscosity of the oil___ a
Ans. Decreases Ans. External equalizer
-The mass flow rate of refrigerant per unit capacity____ as the condensing pressure Cavitation occurs when the pressure at any point inside a pump drops below the vapor
decreases. pressure corresponding to the temperature of liquid. Its effect include: Ans. All of the above
Ans. Decreases
-It consists of a tapered glass tube set vertically in the fluid or gaseous piping system with its Positive displacement pump consisting of a fixed casing containing gears, cams, screws,
large end at the top and a metering float free to move vertically in the tube vanes, plungers or similar elements actuated by rotation of the drive shaft. Ans. Rotary pumps
Ans. Rotameter
-Also termed as monitor light, it indicates which among a number of normal conditions of a Branch of which is larger than the run. Ans. Bull head tee
system or device exist.
Ans. Pilot light A larger pipe or drum into which each group of boiler is connected: Ans. Header
-Which of the following is not a caliper?
Ans. Feeler gage In pipe identification, the color fro pipe used for electricity: Ans. Light orange
-An underground formation that contains sufficient saturated permeable material to yield
significant quantities of water. Has the same equipment as the refrigerating circuits each with a condenser, evaporator, and a
Ans. Aquifer pressure imposing elements, where the evaporator of one circuit cools the condenser of the
-Chemical used to speed up sewage sedimentation: other circuit. Ans. Cascade
Ans. Lime
-Power expenditure when a current of the one ampere flow across two points having a voltage The temperature of air to be used for comfort cooling shall be in the range of: Ans. 20 deg to
drop of one volt. 23.3 deg
Ans. Watt
-Prandtl number for air is generally in the order of A valve held closed by a spring or other means and designed automatically relieve pressure in
Ans. 0.70 excess of its settling. Ans. Pressure relieve device.
Relative humidity of air to be used in standard air cooling: Ans. 55%-60%
A refrigerating system in which the pressure-imposing element is mechanically operated: Ans. A refrigerants which should not be used in public assembly occupancies. Ans. Group 3
Compression refrigerants
A cushioning device all the end of a trolley, bridge or other moving parts of a crane operating
on rails to minimize shock Fire involving flammable liquids and gases. Ans. Class B fires
in the event of collision: Ans. Buffer
Mechanics of water or other liquids whether at rest or motion. Ans. Hydraulics
A boom type mobile crane mounted on endless track or treated belts: Ans. Crawler crane
Measure the pressure of water discharging from a nozzle by having its open end on the water
An apparatus for raising or lowering a load by the application of a pulling force but does not and the end connected to a manometer. Ans. Pitot Tube
include a car or platform riding in guides: Ans. Hoist
A vertical turbine pump with the pump and the motor closed coupled and designed to be
Most widely used industrial pressure gage applied to both pressure and vacuum: Ans. Bourdon installed underground. Ans. Submersible pump
tube gage
Which of the following is a type of deep well pump. Ans. All of the above.
Hydrocarbons found in liquefied petroleum gas: Ans. All of the above
Steel pipe coated with zinc to resist corrosion. Ans. Galvanized pipe
Which of the following is not a solid fuel: Ans. Tar
A fitting with a number of branches in the connecting the smaller pipes. Ans. Manifold
Amount of cooling produced by 2000 lbs of ice in melting over a period of 24 hrs.: Ans. B
and C The ratio of peak load to the Average load is termed as _____ in variable load nomenclature.
Ans. Load factor
It smoothens the flow due to the nature of flow of the liquid from a reciprocating pump: Ans.
Air Chamber A heat exchange device used to provide heat transfer between exhaust gases and the air prior
to its entrance to the combustor. Ans. Regenerator
Science of force exerted by water in motion: Ans. Hydrodynamics
In a sensible heating process, the moisture content: Ans. Remains constant
Lowest permissible water level of a boiler without internal furnace: Ans. 1/3 height of shell
Flow on both sides on a normal shockwave: Ans. Supersonic on one side, Subsonic on the
In pumps, it transmits power from the driver to the impeller: Ans. Shaft other
A change of phase directly from vapor to solid, without passing through the liquid state: Ans. The Westphal balance is a laboratory instrument used to: Ans. Specific gravity
Deposition
Generally, permissible velocity of water flowing through concrete tunnel is: Ans. 4-5 m/s
What reaction occurs when the enthalpy of the product is less than the enthalpy of the
reactants? Ans. Exothermic Refrigerant used in passenger aircraft bins. Ans. Air

A well designed engine has a volumetric efficiency within the ranged: Ans. 75%-90% It refers to atoms of the same atomic number but differ in atomic masses and molecular
weights. Ans. Isotopes
It is referred to as the maximum continuous power available from a hydroelectric power plant
even under the most adverse hydraulic condition: Ans. Firm power Piston rings are normally made of: Ans. Cast Iron

There are how many feed water lines connected to the boiler? Ans. Two Two isothermal and two reversible adiabatic process comprise a: Ans. Carnot cycle

It is the difference in pressure as measured above or below the atmospheric pressure. Ans. In terms of viscosity, density can be expressed as: Ans. Dynamic viscosity/ kinematic
Draft viscosity

The compression ratio of a gas turbine is in the range of 5-8. Ans. 5 to 8 Compressor often used in supercharging Diesel engine: Ans. Root blower type

The average pH of a normal rainfall is generally: Ans. Slightly less than 7 In treating a person whose eyes accidentally got in contact with Freon use: Ans. Sterile
mineral oil
Another term used for liquid valve. Ans. King valve
The coefficient of performance of a domestic (local) refrigerator is always: Ans. Greater than A device in vapor compression refrigeration system whose primary function is to meter the
one flow of
Refrigerant to the evaporator: Ans. Thermostatic expansion valve
A belt-Coleman cycle is a reversed : Ans. Joule cycle
From the mathematical perspective, a thermodynamic property is: Ans. A point function
In an air compressor system, the function of a receiver is to: Ans. Collects water and grease
suspended in air In the production of beer, a raw materials called yeast is added in the: Ans. Starting tubs

General layout plan for each floor drawn to scale should not be less than scale of: Ans. 1:200 Purpose is to keep the moisture from entering the system. Ans. Dehydrator

Highest vapor drop in a vapor compression refrigeration cycle occurs in the: Ans. Expansion Medium pressure as applied to valves and fittings implies that a working pressure of _____ is
valve suitable enough. Ans. 862-1200psi

Which of the following pairs represent the two broad classification of lubricating oils? Ans. A liquid-vapor mixture with a dryness factor is allowed to absorb heat. Which of the following
Straight and Additives is likely to occur? Ans. Increases

A form of misalignment between the pump and the driver shaft wherein the shafts are Work rooms referring to the maintenance shop and machine rooms shall be _____ in the
concentric but not parallel. Ans. Angular Misalignment height from floor to the ceiling? Ans. 3,000mm

In the field of metal corrosion, it is the process wherein it exhibits in the quality deterioration Defined as a passageway made of sheet metal or other suitable material not necessarily leak
of metals: Ans. Passivation tight, used for conveying air or other gases at low temperature. Ans. Duct

It is the ratio of the density of liquid substance to the density of water at standard conditions: As a good practical rule, the foundation depth may be taken as _____ times the engine stroke,
Ans. Specific gravity the lower factor for well-balanced multi-cylinder engine and the higher factor for engines with
fewer cylinder s or on less firm soil: Ans. 3.2 to 4.2
Term associated with an increase on pressure on a pipe caused by sudden velocity decrease.
Ans. Water hammer What is the suggested maximum permissible dos (MDP) of a gamma ray exposed for
individuals now working in a nuclear environment in rem year? Ans. ½
A process which takes place without change in volume. Ans. Isochoric
Most commercially petroleum lubricating oil deteriorates starting from temperature of: Ans.
Weir refers to an opening: Ans. Having partial full flow 200 F
States that the external pressure applied to a confined liquid increases the pressure of every Instrument used to analyze gases. Ans. Orsat Apparatus
point in the fluid by an amount equal to the external pressure: Ans. Pascal’s Law
The chemical formula of R-12 or dichlorofluoromethane. Ans. CCL₂F₂
The average fuel-oil temperature range of the oil in the discharge line to the boilers. Ans. 180-
200 F Which of the following is not desirable property of a refrigerant? Ans. Low thermal
conductivity
Boiler gage glasses should be blown down: Ans. At the start of every watch
Refrigerant widely used in room air conditions. Ans. R-22
The lowest portion of the storage basin where the water is not drawn: Ans. Dead storage
What occurs when pumps are connected in parallel? Ans. Increase discharge, same head
Which among the following do not measure relative humidity? Ans. Piezometer
Which of the following is not a cause of cavitation? Ans. Low velocity
Water behind the dam of a hydro electric power plant? Ans. Pondage
Pipe attached to the penstock to be able to let the water be at atmospheric pressure. Ans. Surge
Converts energy of water to mechanical energy: Ans. Turbine chamber

Ratio of average load to the rating of the equipment supplying the load. Ans. Plant factor Which of the following is not considered as gaseous fuel? Ans. Bunker

Ratio of actual maximum demand load to the connected load. Ans. Demand factor Ratio of the density of a liquid to the density of water. Ans. Specific Gravity
Measurement of randomness or disorder. Ans. Entropy
A device which preheats feed water by utilizing the heat of the flue gases. Ans. Economizer
Study of the proportion of water vapor content of air. Ans. Air conditioning Energy cannot be created nor destroyed but can only be transformed from one form to another.
Ans. First law of thermodynamics
Actual temperature of air. Ans. Dry bulb temperature
In standard Otto cycle when the compression ratio is increased then the thermal efficiency
For a machine foundation with class A mixture what is the proportion of cement, sand and will: Ans. Increase
gravel? Ans. 1:2:4
Heat energy produced by the movement of molecule within a substance caused by its
Component included in the proximate analysis of fuel. Ans. Ash, moisture, Volatile matter, temperature. Ans. Internal energy
fixed carbon
The law which states that one cannot operate a 100% efficient machine. Ans. 2nd law of
The prime mover must have its main steam line: Ans. In loops thermodynamics

Intercooler are primarily used with: Ans. Gas compressors In relation to brake power the instrument used to measure torque: Ans. Dynamometer

In the psychrometric chart, the diagonal lines represents: Ans. Wet bulb temperature Ratio of the radiation of an actual body to the radiation of the black body. Ans. Emittance

Moderator in certain types of nuclear reactors. Ans. Heavy water Refrigerant used in ice plant. Ans. Ammonia

The main advantage of turbulous boilers. Ans. Steam pressure can be raised in short time Measures the pressure of water discharging from the nozzle by having its open end in the
water and the other end connected to a manometer. Ans. Pitot tube
The law which states that the entropy of all perfect crystalline solids is zero at absolute zero
temperature. Ans. 3rd law of thermodynamics Defined as a wall designed to prevent the spread of fire having a fir resistance for four hours.
Ans. Fire wall
Operates between two constant temperature reservoir. Ans. Carnot engine
The work done per unit charge when the charge is moved from one point to another? Ans.
Specific measurements of moisture content of air. Ans. Degree of saturation Potential at a point

Simultaneous on site generation of electric energy and steam from the same plant output. Ans. In the relation of PVⁿ is constant, what value makes the process isobaric? Ans. Zero
Co-generation
In actual gas behavior, molecular collisions are: Ans. Inelastic
Instrument used for pressure readings. Ans. Manometer
An ideal refrigeration should have: Ans. Low freezing point
Difference between the indicated power and the brake power. Ans. Friction power
The immediate undesirable products from the petroleum based lubricating oil subjected to
high pressure and temperature is referred as: Ans. Carbon Dioxide In Diesel power plant, its purpose is to reduce the weight-to-power ratio: Ans. Supercharging

A rotary dynamic pump, fluid is at no time confined by moving boundaries in its passage Change of phase from solid to gas. Ans. Sublimation
through the pump. Ans. Non-positive displacement pump
The memory lost when operating power is removed. Ans. Volatile memory
Also known as tangential or Pelton wheel. Ans. Impulse turbine
A refrigeration system using direct method in which refrigerant is delivered to two or more
Term given to a chimney made of steel. Ans. Stack evaporators in space room or in refrigerators. Ans. Multi-pressure

An integral part of reaction turbine used to recover energy head. Ans. Draft tube The carbon dioxide concentration of air in all rooms when measured 910mm above the floor
space not exceed: Ans. 100ppm
Based on good practice, the vertical distance from the floor soil level to the top edge of the
foundation must be around _____ as minimum distance in machiner foundation. Ans. 6 inches Consists of helmets shall be removed immediately after having been used or the seal broken,
must be removed atleast every: Ans. 2 years
The geometric view factor for a black body is always. Ans. One
For every 1000ft ascent, there is a corresponding pressure decrease approximately. Ans. 1 in Pump whose purpose is to increase the effective water pressure by sucking water from public
of Hg. service main or private use water system: Ans. Booster pump

Used for impounding water storage and for creating head for the power plant. Ans. Dam A small fitting with a double offset, or shaped like the letter C with the ends turned out. Ans.
Cross-cover
The temperature at which the phase change takes place at a given pressure. Ans. Saturation
pressure Material interposed between two relatively moving machine elements to reduce wear and tear:
Ans. Lubricant
Distinguishes the system from its surroundings may be at rest or in motion. Ans. Boundary
Measure temperature by electromotive force: Ans. Thermocouple
Quality is a measure of: Ans. Dryness
According to SAE which of the following is a type of lubricating oil? Ans. All of the above
The highest pressure under which distinguished liquid vapor phase can exist in equilibrium:
Ans. Critical pressure A device that information in one form of an instrument signal and transmits an output signal
on another form: Ans. Digital
In a thermodynamic gaseous substance, the relationship between P-V-T given by the: Ans.
Equation of state A device whose function to pass on information in an unchanged form or in some modified
term. Ans. Switch
A correction factor for gas approaching real gas behavior: Ans. Compressibility factor
A device that receives information on one form of one or more physical quantities modifies
Furl production process wherein heavy oil is changed into gasoline by means of a high the information and/or its form and produces a resultant output signal. Ans. Transducer
pressure, high temperature and longer exposure time. Ans. Thermal cracking
Which of the following is not a requirement in designing pipe installations? Ans. Galvanized
A shut off valve for controlling the flow of refrigerant. Ans. Stop valve pipe should be used for steam

Measured by the amount of its pressure below the prevailing atmospheric pressure. Ans. Part of a valve used to guide and support valve stem: Ans. Bonnet
Partial vaccum
In piping installations, the color of pipe tube used for water is: Ans. Green
A relief valve that can positively lift the disc from its seat at least 1.5 mm when there is no
pressure in the boiler. Ans. Water relief valve The color code for pipes used in communication. Ans. White

Vapor produced in the vaporization of a small amount of liquid refrigerant downstream of the Which of the method is used for classifying coal. Ans. All of the above
expansion valve. Ans. Flash gas
Alcohol frequently considered as fuel for internal combustion engine. Ans. Ethyl alcohol
Product of the rate of evaporation and the factor of evaporation. Ans. Equivalent evaporation
Measure of resistance to flow. Ans. Viscosity PV = mRT
280(2) = m(8.314/44)(40+273)
Converts Solar energy to electrical energy. Ans. Photovoltaic cell m = ____________ ANS

Which of the following is not an instrument used to measure flow rates? Ans. Velometer # A thermal power plant has a heat rate of a 11,363 Btu/kw-hr. Find the thermal efficiency of
the plant.
Refrigeration follows what cycle? Ans. Reversed Carnot cycle Solutionj:
eth = 3412/Heat rate = 3412/11,363 = _________ANS.
An evaporator constructed on pipe tubing. Ans. Expansion coil
# What is the hydraulic gradient of a 1 mile, 17 inches inside diameter pipe when 3300
Reinstalled or second hand boilers shall have a minimum factor of safety of? Ans. 6 gal/min of water flow with f = 0.03.
A line that shows the rotation of the consumption and the load a steam turbine generator. Ans. Solution:
Willans Line V = (3300/7.481)/(π/4)(17/12)2(60) = 4.66 ft/s
L = 1 mile = 5280 ft
Instrument that indicate the percentage of carbon dioxide in flue gases in a power plant. Ans. hL = fLv2/2gD = 0.03(5280)(4.66)2/2(32.2)(17/12) = 37.7 ft
Ranarex indicator Hydraulic gradient = 37.7/5280 = ___________ANS.

For real process, the net entropy change in the universe is: Ans. Positive # Find the loss of head in the pipe entrance if speed of flow is 10 m/s.
Solution:
The sum of the internal energy and the product of the pressure and specific volume. Ans. Loss at entrance = 0.5(v2/2g) = 0.5[102 / 2(9.81)] = ___________ANS.
Enthalpy
Wet material, containing 220% moisture (dry basis) is to be dried at the rate of 1.5 kg/s in a
Ratio of the maximum demand of the system to the rated capacity of the system. Ans. continuous dryer to give a product containing 10% (dry basis). Find the moisture removed,
Utilization factor kg/hr.
Solution:
Source of thermal energy. Ans. All of the above Solid in wet feed = solid in dried product
[1/(1 + 2.2)](1.5) = [1/(1 + 0.1)](x)
Cycle that has two isentropic and two isometric process. Ans. Otto cycle x = 0,5156 kg/s (total dried product)
Moisture removed = 1.5 – 0.5156 = 0.984 kg/s = ___________ANS.
Cycle that has two isentropic and two isobaric process. Ans. Brayton cycle
# Copra enters a dryer contacting 70% moisture and leaves at 7% moisture. Find the moisture
# Two kilogram of gas is confined in a 1 m3 tank at 200 kpa and 88°C. What type of gas is in removed on each pound of solid in final product.
the tank? Solution:
Solution: Solid in wet feed = solid in dried product
PV=mRT 0.3x = 1
200(1) = 2(8.314/M) (88+273) x = 3.333 lbs
M = 30 1= 0.93y
Therefore: gas is Ethane Y = 1.07527 lb
Moisture removed = x – y = 3.333 – 1.07527 = _________ ANS.
# Find the enthalpy of helium if its internal energy is 200 KJ/kg.
Solution: # A 1 m x 1.5 m cylindrical tank is full of oil with SG = 0.92. Find the force acting at the
R = 8.314/4 = 2.0785 bottom of the tank in dynes.
K = 1.667 for helium Solution:
Cp = k R/(k-1) = 2.0785/(1.667 – 1) = 3.116 KJ/kg-K P = w h = (0.92 x 9.81) (1.5) = 13.5378 kpa
F = PA = 13.5378(π/4 x 12) = 10,632 KN = 10,632.56 N x 10,000 dynes/N
h/ = Cp/Cv F = ____________ANS.
/200 = 5.195/3.116
# Find the pressure at the 100 fathom depth of water in kpag
= ______________ANS Solution:
H = 100 fathom x 6 = 600 ft
# Compute the mass of a 2 m3 propane at 280 kpa and 40°C. P = w h = (600/3,281)(9.81) = ______________ANS.
Solution:
Propane is C3 H8 --------- M = 12(3) + 8(1) = 44
# Find the depth in furlong of the ocean (SG = 1.03) if the pressure at the sea bed is 2,032.56 Wc = 36(1-0.20) = 28.80 kw
kpag. COP = RE/Wc
Solution: 6 = RE/28.80
P=wh RE = 172.8/3.516 = ______________ANS.
2,032.56 = (1.03 x 9.81) h
H = 201.158 m x 3.281ft/m x 1 yd/3ft x 1 furlong/220 yd = 1 furlong # A 23 tons refrigeration system has a heat rejected of 100 kw. Find the energy efficiency ratio
of the system.
# Find the mass of 10 quartz of water. Solution:
Solution: QR = RE + Wc
V = 10 quartz x 1gal/4quartz x 3.785li/gal x 1m3/1000li 100 = 23(3.516) + Wc
V= 0.0094625 x 10-3 m3 Wc = 19.132 kw
W = m/V COP = RE/Wc = (23 x 3.516) / 19.132 = 4.23
1000 = m/0/0094625 x 10-3 EER = 3.412 COP = 3.412(4.23) = ____________ANS.
m = __________ ANS.
# A 200 mm x 250 mm, 8-cylinder, 4-stroke diesel engine has a break power of 150 kw. The
# Find the mass of carbon dioxide having a pressure of 20 psia at 200°F with 10 ft3 volume. mechanical efficiency is 80%. If two of the cylinders were accidentally cut off, what will be
Solution: the new friction power?
PV = m R T Solution:
(20 x 144)(10) = m (1545/44)(200 + 460) em = BP/IP
m = ___________ANS. 0.8 = 150/IP
IP = 187.5 kw
# Find the heat needed to raise the temperature of water from 30°C to 100°C with 60% FP1 = IP – BP = 187.5 – 150 = 37.50 kw
quality. Consider and atmospheric pressure of 101.325 kpa. Use the approximate enthalpy FP1 = FP2 = ____________ANS.
formula of liquid.
Solution: # If the energy efficiency ratio of the refrigeration system is 12.6, what is the COP of the
At 100°C system?
hf = Cp t = 4.187(100) 418.7 KJ/kg Solution:
hfg = 2257 KJ.kg EER = 3.412 COP
h2 = hf + xhfg = 418.7 + 0.60(2257) = 1,772.9 KJ/kg 12.6 = 3.412 COP
Q = 1(4.187)(100-30) + 1(1772.9 – 418.7) = _________ANS. COP = ____________ANS.

# Find the enthalpy of water at 212°F and 14.7 psi if the dryness factor is 30%. Use the # An air compressor has a power of 40 kw at 4% clearance. If clearance will increase to 7%,
approximate enthalpy formula of liquid. what is the new power?
Solution: Solution:
hf = (°F – 32) = (212 – 32) = 180 Btu/lb The power of compressor will not be affected with the changes in clearance.
hfg = 970 Btu/lb Therefore the power will still be 40 kw.
h = hf + xhfg
h = 180 + 0.3(970) = ________ANS. # What is the approximate value of temperature of water having enthalpy of 208 Btu/lb?
Solution:
# An air compressor consumed 1200 kw-hr per day of energy. The electric motor driving the h = °F – 32
compressor has an efficiency of 80%. If indicated power of the compressor is 34 kw, find the 208 = °F -32
mechanical efficiency of the compressor. °F = _________ANS.
Solution:
Pim = 1200kw-hr/24 hrs = 50 kw # An Otto cycle has a compression ratio of 8. Find the pressure ration during compression.
BP = 50(0.80) = 40 kw Solution:
em = 34/40 = _______ANS. P1V1k = P2V2k
(V1/V2)k = (P2/P1)
# A refrigeration system consumed 28,800 kw-hr per month of energy. There are 20% of rK = rp
energy is lost due to cooling system of compressor and motor efficiency is 90%. If COP of the rp = (8)1.4 = ___________ANS.
sustem is 6, find the tons of refrigeration of the system.
Solution: # A diesel cycle has a cut off ratio of 2.5 and expansion ratio of 4. Find the clearance of the
Pim = 28,800/(24 x 30) = 40 kw cycle.
BP = 40 (0.90) = 36 kw
# Determine the atmospheric pressure at a location where barometric reading is 740 mm Hg
and gravitational acceleration is g = 9.7 m/s2. Assume the temperature of mercury to be 10°C, cv = R / (k -1) = = 0.29086 Btu/lb-F
at which the density is 13,570 kg/m3. 20 = 10(0.29086)(t2 – 90)
Solution: t2 = ___________ANS.
P = (wg)h = (13,570 x 9.7)(9.74) (1Kpa/1000 N/m2) = ___________ANS.
# Ammonia weighing 22kgs is confirmed inside a cylinder equipped with a piston has
# The barometer of a mountain hiker reads 930 mbars at the beginning of a hiking trip and 780
mbars at the end. Neglecting the effect of altitude on local gravitational acceleration, an initial pressure of 413KPa at 38 C. If 3200KJ of heat is added to the ammonia until its final
determine the vertical distance climb. Assume g = 9.7 m/s2. pressure and temperature are 413KPa and 100oC respectively, what is the amount of work
Solution: done by the fluid in KJ?
P1 – P2 = w h SOLUTION
(0.93 – 0.78)(100 kpa/bar) = (1.2 x 0.00981) h Since the molecular weight of ammonia is 17. Then
h = 1274.21 m R = 8.3143/M = 8.3143/17 = 0.489 KJ/kgoK
h = 1274.21 (9.81/9.7) = ___________ANS. P1V1 = mRT1
413V1 = 22(0.489)(38 + 273)
# The lower half of a 10 m high cylindrical container is filled with water and the upper half V1 = 8.101 m3
with oil that has SG = 8.85. Determine the pressure difference between the top and bottom of P2V2 = mRT2
the cylinder. 413V2 = 22(0.489)(100 + 273)
Solution: V2 = 9.716 m3
W = P(V1-V2) = 413(9.716 – 8.101) = ANS.
P = Pw + Po = 9.81(5) + (0.85 x 9.81)(15) = ________ANS. # A tank contains 90ft3 of air at a pressure of 350 psig; if the air is cooled until its
pressure and temperature decreases to 200 psig and 70oF respectively. What is the decrease in
# An ideal gas at 0.80 atmospheres and 87°C occupies 0.450 liter. How many moles are in the internal energy?
sample? (R = 0.0821 liter-atm/mole – K) SOLUTION
Solution: m = PV/RT = (200 + 14.7)(90)(144)/(53.342)(70 + 460) = 98.50 lbs
PV = nRT for constant volume process:
P1/T1 = P2/T2
(0.80 atm)(0.450 li) = n (0.0821 )(87+273)K T2 = 70 + 460 = 530oR
n = ___________ANS.
=
# A certain gas at 101.325 Kpa and 10°C whose volume is 2.83 m3 are compressed into a
T1 = 900oR
storage vessel of 0.31 m3 capacity. Before admission, the storage vessel contained the gas at a ΔU = mcv(T2 – T1) = 98.50(0.171)(530 – 900) = ANS.
pressure and temperature of 173.8 Kpa and 26°C, after admission the pressure has increased to # A large mining company was provided with a 3m3 of compressed air tank. Air
1171.8 Kpa. What should be the final temperature of the gas in the vessel in Kelvin?
pressure in the tank drops from 700KPa while the temperature remains constant at 28oC. What
Solution:
percentage has the mass of air in the tank been reduced?
Solving for the mass of gas which is to be compressed: SOLUTION
PV = mRT Percent mass reduced = (700 – 150)/700 = ANS.
101.325(2.83) = m1R(10 + 273)
# A 4m3/hr pump delivers water to a pressure tank. At the start, the gauge reads 138KPa
m1 = 1.01325/R
until it reads 276KPa and then the pump was shut off. The volume of the tank is 180liters. At
Solving for the mass of gas initially contained in the vessel: 276KPa the water occupied 2/3 of the tank volume. Determine the volume of the water that
PV = mRT can be taken out until the gauge reads 138KPa.
137.8(0.31) = m2R(26 + 273)
SOLUTION
m2 = 0.14286/R
Consider the air pressure:
Solving for the final temperature: V2 = 1/3 (180) = 60liters
m3 = m1 + m2 P1V1 = P2V2
m3 = 1.01325/R + 0.14286/R = 1.156/R
(138 + 101.325)(V1) = (276 + 101.325)(60)
1171.8(0.31) = (1.156/R)RT3
V1 = 94.59 liters
T3 = ____________ANS. Amount of water to be removed = 2/3 (180) – (180 – 94.59) = ANS.
# A refrigeration plant is rated at 15tons capacity. How many pounds of air per hour will
# A perfect gas has a value of R = 58.8 ft-lb/lb-R and k = 1.23. if 20 Btu are added to 10 lbs of
it cool from 70 to 90oF at constant pressure.
this gas at constant volume when initial temperature is 90°F, find the final temperature.
SOLUTION
Solution Tons of refrigeration = mcp(t2 – t1)/12000
Q = m cv (t2 – t1)
At 300KPa and 151.86oC s = 7.088 KJ/kg
SOLUTION
s1 = sf + xsfg = 1.8607 + 0.70(4.9606) = 5.333 KJ/kg
m= ANS. s2 = 7.088 KJ/kg
# An air standard engine has a compression ratio of 18 and a cut – off ratio of 4. If the Q = T(s2 - s1) = (151.86 + 273)(7.088 – 5.333) = ANS.
intake pressure and temperature are 100KPa and 27oC, find the work in KJ per kg. # A tank contains exactly one kilogram of water consisting of liquid and vapor
SOLUTION equilibrium at 1MPa. If the liquid contains one – third and the remaining is vapor of the
volume of the tank, what is the enthalpy of the contents of the tank?
At 1MPa: vf = 0.0011273 vfg = 0.19444
hf = 762.81 hfg = 2015.3
T1 = 27 + 273 = 300oK SOLUTION
Process 1 to 2 is isentropic procees: Let V = total volume of the tank
T2 = T1(V1/V2)k-1 = 300(18)1.4-1 = 953.301oK T2 = T1(V1/V2)k-1 = 300(18)1.4 – 1 = 953.301oK
Process 2 to 3 is constant pressure process: Process 2 to 3 is constant process:
T3/T2 = V2/V3 = rc
T3 = 953.301 (4) = 3813.205oK
QA = mcp(T3 – T2) = 1(3813.205 -953.301) = 2860 KJ/kg
W = (.5531)(2860) = ANS. T3 = 953.301(4) = 3813.205oK
# Determine the air – standard efficiency of an engine operating on the diesel cycle with QA = mcp(T3 – T2) = 1(3813.205 - 953.301) = 2850 KJ/kg
clearance of 6% when the suction pressure is 99.97KPa and the fuel injected for 7% of the W = (0.5531)( 2850) = ANS.
stroke. Assume k = 1.4. # Determine the air standard efficiency of an engine operating on the diesel cycle with
SOLUTION clearance of 6% when the suction pressure is 99.97KPa and the fuel is injected for 7% of the
V3 – V2 = 0.07VD stroke. Assume k = 1.4.
V2 = 0.06VD SOLUTION
V3 - 0.06VD = 0.07VD V3 - V2 = 0.07VD
V3 = 0.13VD V2 = 0.06VD
rc = V3/ V2 = 0.13VD/0.06VD = 2.167 V3 = 0.07VD + 0.06VD
rk = (1 + 0.06)/0.06 = 17.667 V3 = 0.13VD
rc = V3/ V2 = 0.13VD /0.06VD = 2.167
rk = (1 + 0.06)/(0.060) = 17.667
= ANS.
# Steam at 2MPa and 250oC in a rigid cylinder is cooled until the quality is 30%. Find
the heat rejected from the cylinder. e= = ANS.
At 2MPa and 250oC: v = 0.11144m3/kg u = 2679.6 KJ/kg # A steam at 2MPa and 250oC in a right rigid cylinder is cooled until the quality is 30%.
At 2MPa, (saturated): vf = 0.0011767m3/kg, vg = 0.09963m3/kg uf = 906.44 Find the heat rejected from the cylinder.
KJ/kg, At 2MPa and 250oC: v = 0.11144 m3/kg u = 2679.6 KJ/kg
ufg = 1693.8KJ/kg At 2MPa, (saturated): vf = 0.0011767 m3/kg, vg = 0.09963 m3/kg
SOLUTION uf = 906.44 KJ/kg ufg = 1693.8 KJ/kg
Q = U2 – U1 SOLUTION
U1 = 2679.6 KJ/kg Q = (u2 – u1)
U2 = Uf + xUfg = 906.44 + 0.30(1693.8) = 1414.58 KJ/kg u1 = 2679.6KJ/kg
Q = 1414.58 – 2679.6 = ANS. u2 = uf + x ufg = 906.44 + 0.3(1693.8) = 1414.58KJ/kg
# At 1.3MPa, mixture steam and water has an entropy of 3KJ/kg - oK. Find the enthalpy Q = (1414.58 – 2679.6) = ANS
of the mixture. # At 1.3MPa, mixture steam and water has an entropy of 3KJ/kg-oK. Find the enthalpy
At 1.3MPa: sf = 2.2515, sg = 6.4953, hf = 814.93, hfg = 1972.7 of the mixture.
SOLUTION At 1.3MPa: sf = 2.2515, sg = 6.4953, hf = 814.93, hfg = 1972.7
s = sf + xsfg SOLUTION
3 = 2.2515 + x(6.4953) s = sf + x(sg – sf)
x = 0.17614 3 = 2.2515 + x(6.4953 – 2.2515)
h = hf + xhfg = 814.93 + 0.17614(1972.7) = ANS. x = 0.17637
# Mixture with 70% quality at 500KPa is heated isothermally until its pressure is h = hf + xhfg = 814.93 + 0.17637(1972.7) = ANS.
300KPa. Find the heat added during the process. # Mixture with 70% quality at 500KPa is heated isothermally until its pressure is
At 500KPa: sf = 1.8607, sfg = 4.9606 300KPa. Find the heat added during the process.
At 500KPa: sf = 1.8607, sfg = 4.9606 For throttling process: (h1 = h2)
At 300KPa and 151.86oC: s = 7.0888 h1 = h2 = hf + x hfg
SOLUTION 2775.6 = 962.11 + x(1841)
For isothermal process, t1 = t2 x = 98.5%
s1 = sf + xsfg = 1.8607 + 0.7(4.9606) = 5.333 s1 = sf + x sfg = 2.5447 + 0.985(3.7028) = ANS.
s2 = 7.0888 # Atmospheric pressure boils at 212oF. At the vacuum pressure at 24 in Hg, the
Q = T(s2 – s1) = (151.86 + 273)(7.0888 – 5.333) = ANS. temperature is 142oF. Find the boiling temperature when the pressure is increased by 40psia
# A tank contains exactly one kilogram of water consisting of liquid and vapor in from the atmospheric.
equilibrium at 1MPa. If the liquid contains one third and the remaining is vapor of the volume SOLUTION
of the tank, what is the enthalpy of the contents of the tank? P2 = 14.7 + 40 = 54.7 psia
At 1MPa: vf = 0.0011273 vfg = 0.19444 P1 = -24(14.7/29.92) + 14.7 = 2.908 psia
hf = 762.81 hfg = 2015.3 By interpolation:
SOLUTION
Let V = total volume of tank
mL = VL/vL = (V/3) / 0.0011273 = 295.69 V
mv = Vv / vv = (2V/3) / 0.1944 = 3.429V t2 = ANS.
# A certain coal has the following ultimate analysis:
C = 69% N2 = 5% H2 = 2.5% S = 7%
x= = ANS.
Determine the amount of oxygen of the heating value of fuel is 26 961.45 KJ/kg.
# Water substance at 70 bar and 65oC enters a boiler tube of constant inside diameter of
SOLUTION
35mm. The water leaves the boiler tube at 50 bar and 700oK at velocity of 100m/s. Calculate
the inlet volume flow(li/sec).
SOLUTION Qh = 33 820C + 144 212 (H - ) + 9304S
From steam tables: 26961.45 = 33820(.69) + 144212 (0.025 – O/8) + 9304(0.07)
At 70bar(7MPa) and 65oC O= ANS.
v1 = 0.001017m3/kg # A diesel engine consumed 945 liters of fuel per day at 35oC. If the fuel was purchased
at 50bar(5MPa) and 700oK(427oC) at 15.5oC and 30oAPI at P29.00/li, determine the cost of fuel to operate the engine per day.
v2 = 0.06081m3/kg SOLUTION
m1 = m2
Q1/ v1 = Q2/ v2 16

V1 = 1.672m/sec
Q1 = A x v = (π/4)(0.035)2(1.672) = ANS.
# Steam leaves an industrial boiler at 827.4KPa and 171.6oC. A portion of the steam is
passed through a throttling calorimeter and is exhausted to the atmosphere when the
calorimeter pressure is 101.4KPa. How much moisture does the steam leaving the boiler
contain if the temperature of the steam at the calorimeter is 115.6oC?
At 827KPa(171.6oC): hf = 727.25 KJ/kg, hfg = 2043.2 KJ/kg Cost = P29.00/li(935.44li) = ANS.
From table 3: At 101.4KPa and 115.6oC: h2 = 2707.6 KJ/kg # A cylindrical tank 4m long and 3m diameter is used for oil storage. How many days
SOLUTION can the tank supply the engine having 27oAPI with fuel consumption of 60kg.hr?
Let x = quality of steam entering the throttling calorimeter SOLUTION
h1 = h2 V = π/4 D2 h = π/4 (3)2(4) = 28.274 m3
hfg + xhfg = h2
727.25 + x(2043.2) = 2707.6
x = 0.9692; y = 1 - 0.9692 = ANS. Density of fuel = 0.89274(1000kg/m3) = 892.74kg/m3
# A throttling calorimeter is connected to the desuperheated steam line supplying steam w = m/V
to the auxiliary feed pump on a ship. The line pressure measures 2.5MPa. The calorimeter V = 60/892.74 = 0.0672 m3/hr
pressure is 110KPa and 150oC. Determine the entropy of the steam line. Number of days = 28.274/0.0672 = 420.75hrs = ANS.
At 110KPa and 150.C: h2 = 2775.6KJ.kg # A logging firm in Isabella operates a Diesel Electric Plant to supply its electric energy
At 2.5MPa: hf = 962.11KJ/kg, hfg = 1841 KJ/kg, sf = 2.5547, sfg = 3.7028 requirements. During a 24 period, the plant consumed 250 gallons of fuel at 80oF and
SOLUTION
produced 2900KW-hrs. Industrial fuel used is 30oAPI and was purchased at P30.00/li at 60oF. Molal A/F = 0.259 + 0.259(3.76) = ANS.
Determine the overall thermal efficiency of the plant. # A volumetric analysis of a gas mixture is as follows:
SOLUTION CO2: 12% N2: 80%
Qh = 41130 + 139.6xoAPI = 41130 + 139.6(30) = 45318 KJ/kg O2: 4% CO: 4%
60oF = 15.6oC What is the percentage of CO2 on a mass basis?
80oF = 26.6oC SOLUTION
Converting to mass basis:
CO2 = 0.12 x 44 = 5.28
O2 = 0.014x32 = 1.28
N2 = 0.82x28 = 22.96
At 26.6oC CO = 0.02x28 = 0.56
mf = 250gal/24hrs x 3.785li/gal x 0.869kg/li x 1hr/3600sec Total mass of product = 5.28 + 1.28 + 22.96 + 0.56 = 30.08 kg
mf = 0.00952 kg/sec % mass of CO2 = 5.28/30.08 = ANS.
load = 2900/24 = 120.833KW # The following coal has the following ultimate analysis by weight:
C = 70.5% H2 = 4.5% O2 = 6.0% N2 = 1.0%
overall efficiency = = ANS. S = 3.0% ash = 11% moisture = 4%
# The dry exhaust gas from oil engine has the following gravimetric analysis: A stocker fired boiler of 195000kg/hr steaming capacity uses this coal as fuel. Calculate
CO2 = 21.6% O2 = 4.2% N2 = 74.2% volume of air in m3/kg with air at 60oF and 14.7 psia pressure of boiler efficiency is 70% and
Specific heats at constant pressure for each component of the exhaust gas in Kcal/kgoC are: FE = 1.10.
CO2 = 0.203 O2 = 0.219 N2 = 0.248 SOLUTION
Calculate the specific gravity if the molecular weight of air is 28.97 kg/kg-mol.
SOULTION Theo. A/F = 11.5C + 34.5(H – O/8) + 4.3S
Converting the gravimetric analysis to volumetric:
CO2 = 0.219/44 = 0.004909 = 11.5(0.705) + 34.5(0.045 – 0.06/8) + 4.3(0.03)
O2 of nitrogen in lb/= 0.042/32 = 0.001312
N2 = 0.742/28 = 0.026500 = 9.53
0.032721 mols/kg-mol
Molecular weight = 1/0.032721 = 30.56kg/kg-mol Actual A/F = 9.53(1.3) = 12.389 kg air/kg fuel
SG = 30.56/28.97 = ANS.
# A bituminous coal has the following composition:
C = 71.5% H = 5.0% O = 7.0% N = 1.3% S = 3% Ash =
7.6%
W = 3.4%
Determine the theoretical weight of nitrogen in lb/lb of coal.
SOLUTION
Theo. A/F = 11.5C + 34.5(H – O/8) + 4.3S
= 11.5(0.715) + 34.5 (0.05 – 0.07/8) + 4.3(0.03)
= 9.8 lb air / lb coal
N2 in air by weight = 76.8%
therefore:
theoretical weight of N2 = 0.768(9.8) = ANS.
# A gaseous fuel mixture has a molal analysis:
H2 = 14% CH4 = 3% CO = 27%
O2 = 0.6% CO2 = 4.5% N2 = 50.9%
Determine the air fuel ratio for complete combustion of molal basis.
SOLUTION
Chemical reaction with oxygen:
0.14H2 + 0.070O2 = 0.14H2O
0.03CH4 + 0.060O2 = 0.03CO2 + 0.06H2O
0.27Co + 0.135O2 = 0.27CO2
0.265O2 ; mf = 23107.56 (12.389) = 286279.57 kg/hr
Actual O2 in product = 0.265 O2 – 0.006 O2 = 0.259 O2
101.325(V) = 286279.57(0.287)(15.6 + 273)
Efficiency = ANS.
V= ANS.
# A two-stage air compressor air at 100 Kpa and 22⁰C discharges to 750Kpa. If intercooler
# 23.5 kg of steam per second at 5MPa and 400oC is produced by a steam generator. intake is 105⁰C, determine the value of n.
The feedwater enters the economizer at 145oC and leaves at 205oC. The steam leaves the Solution:
boiler drum with a quality of 98%. The unit consumes 3kg of coal per second as received
having a heating value of 25102 KJ/kg. What would be the overall efficiency of the unit in Px = = 273.86 Kpa
percent?

Steam properties:

At 5MPa and 400oC: h = 3195.7KJ/kg At 5MPa: hf = 1154.23, hg =1640.1

At 205oC: hf = 875.04 At 145oC: hf = 610.63

SOLUTION 1.281 =
n = _____________ ANS.
# A single acting air compressor has a volumetric efficiency of 89%, operates at 500 rpm. It
takes in air at 100kpa and 30⁰C and discharges it at 600 Kpa. The air handled is 8 m3/min
= ANS. measured at discharge condition. If compression is isentropic, find mean effective pressure in
Kpa
# In a Rankine cycle steam enters the turbine at 2.5MPa (enthalpies and entropies given) SOLUTION:
and condenser of 50KPa (properties given), what is the thermal efficiency of the cycle? P1V1k = P2V2k
100(V11.4) = 600(8)1.4
At 2.5MPa: hg = 2803.1 sg = 6.2575 V1 = 28.37687 m3/min
VD = 28.768/0.89 = 32.32 m3/min
At 50KPa: sf = 1.0910 sfg = 6.5029 hf = 340.49 hfg = 2305.4 vf =
0.0010300
W= =
SOLUTION

h1 = 2803.1 KJ/kg W = Pm x VD
7562.19 = Pm x 32.32
solving for h2: Pm = ________ ANS.
# A water-jacketed air compressor handles 0.343 m3/s of air at 96.5 kpa and 21⁰C and leaving
s = sf + xsfg at 480 kpa and 132⁰C; 10.9 kg/h of cooling water enters the jacket at 15⁰C and leaves at 21⁰C.
Determine the compressor break power.
6.2575 = 1.0910 + x(6.5029) SOLUTION:

x = 0.7945

h2 = hf + xhfg = 340.49 + 0.7945(2305.4) = 2172.13 KJ/kg

h3 = 340.49 KJ/kg
n = 1.249
h4 = hf + vf (P2 – P1) = 340.49 + 0.00103(2500 - 50) = 342.98 KJ/kg
W= = 62.57 KW
Q = heat loss = mCp(t2 – t1) = (10.9/3600)(4.187)(21 – 15) = 0.076 KW P1 = ___________ ANS.
Brake power = W + Q = 62.57 + 0.076 = _______ ANS. # A submersible, multi-stage, centrifugal deep well pump 260 gpm capacity is installed in a
# A double suction centrifugal pump delivers 20 ft3/sec of water at a head of 12 m and well 27 feet below the static water level and running at 3000 rpm. Drawdown when pumping
running at 650 rpm. What is the specific speed of the pump? at rated capacity is 10 ft. the pump delivers the water into a 25,000 gallons capacity overhead
SOLUTION: storage tank. Total discharge head developed by pump, including friction in piping is 243 ft.
Calculate the diameter of the impeller of this pump in inches if each impeller diameter
Ns = developed a head of 38 ft.
SOLUTION:
V=πDN
Q=
h = 12 x 3.281 = 39.37 ft V=

π D (3000/60) =
Ns = __________ANS. D = 0.315 ft = _________ ANS.
# Determine the number of stages needed for a centrifugal pump if it is used to deliver 400 # A fan draws 1.42 m3/sec of air at a static pressure of 2.54 cm of water through a duct 300
gal/min of water and pump power of 15 Hp. Each impeller develops a head of 30 ft.
mm diameter and discharges it through a duct of 275 mm diameter. Determine the static fan
SOLUTION:
efficiency if total fan mechanical is 75% and air is measured at 25⁰C and 760 mmHg.
Wp = w Q h
SOLUTION:
15 x 0.746 = 9.81(400 gal/min x 0.003785 m3/gal x 1/60) h
h =45.20 m x 3.281 ft/m = 148.317 ft
Number of stages = 148.317/30 = 4.94 stages wa =
# The suction pressure of a pump reads 3 in. of mercury vacuum and discharge pressure reads # A total head of fan is 187 m and has a static pressure of 210 mm of water gage, what is the
140psi is use to deliver 120 gpm of water with specific volume of 0.0163 ft3/lb. Determine the velocity of air flowing if density of air is 1.15 kg/m3?
pump work. SOLUTION:
SOLUTION: hs = 0.21(1000-1.15) = 182.61 m
P1 = -3 in Hg x 101.325/29.92 = -10.15 Kpa h = hs + hv
P2 = 140 psi x 101.325/14.7 = 965 Kpa 187 = 182.61 + hv
W = 1/v = 1/.0163 = 61.35 lb/ft3 x 9.81/62.4 = 9.645 KN/m3 hv = 4.39 m
4.39 = v2 /2(9.81)
h= = = 101.105 m v = _________ANS.
Q = 120 gal/min x 3.785 li/1gal x 1m3/1000li x 1/60 = 0.00757 m3/sec # A fan delivers 5.7 m3/sec at a static pressure of 5.08 cm of water when operating at a speed
P = w Q h = 9.645(0.00757)(101.105) = ____________ ANS. of 400 rpm. The power input required is 2.963 KW. If 7.05 m3/sec are desired in the same fan
# A submersible pump delivers 350 gpm of water to a height of 5 ft from the ground. The and installation, find the pressure in cm of water.
pump was installed 150 ft below the ground level and a draw down of 8 ft during the SOLUTION:
operation. If water level is 25 ft above the pump, determine the pump power.
SOLUTION:
h = 5+ 150 – (25 – 8) = 138/3.281 = 42.06 m
Q = 350 gal/min x 0.003785 m3/gal x 1min/60sec = 0.02246 m3/sec
Wp = w Q h = 9.819.02246)(42.06) = __________ ANS.
# A vacuum pump is used to drain a flooded mine shaft of 20⁰C water. The pump pressure of N2 = 494.74 rpm
water at this temperature is 2.34 Kpa. The pump is incapable of lifting the water higher than
16 m. What is the atmospheric pressure?
SOLUTION:

h2 = __________ANS.
# A rigid container is closed at one end and measures 8 in diameter by 12 in long. The
container is held vertically and is slowly moved downward until the pressure in the container
is 17 psia. What will be the depth of the top of the container from the free water surface?
SOLUTION: SOLUTION:
Patm = Pgage + Patm Re =dV/ v
17 = Pgage + 14.7
Pgage = 2.30 psi Re =
Pgage = w h Since it is greater than 2000, then it is _________ANS
2.30(144) = 62.4 (h) # An insulated rigid tank initially contains 1.5 lb of helium at 80⁰F and 50 psia. Apaddle
h = 5.3077ft x 12 =__________ans. wheel with power rating of 0.02 hp is operated within the tank for 30 min. Determine the final
# An empty, open can is 30 cm high with a 15 cm diameter. The can, with the open end temperature.
down, is pushed under water with a density of 1000 kg/m3. Find the water level in the can SOLUTION:
when the top of the can is 50 cm below the surface. W= ΔU = mcv(T2 – T1)
SOLUTION: 0.02hp (0.50 hr)(2545 Btu/hr/hp) = 1.5 (0.171)(T2 – 80)
Consider the water pressure T2 = __________ANS.
Pw = wh + 101.325 = (0.8 - x)(9.81) + 101.325 = 109.173 – 9.81x # A 4 m2 asphalt pavement with emissivity of 0.85 has a surface temperature of 50⁰C. Find
Consider the air pressure: the maximum rate of radiation that can be emmited from the surface.
P1V1 = P2V2 SOLUTION:
101.325(A x 0.3) = P2[A(0.3 – x)]
Qr = e ksvA
Ksv = 5.67 x 10-8 (Stefan Boltzman constant)
P2 =
Qr = 0.85(5.67 x 108)(4)(50 +273)4 = ____________ANS.
Pw = P2
# Air at 10⁰C and 80 kpa enters a diffuser of a jet engine steadily with a velocity of 200 m/s.
The inlet area of diffuser is 0.40 m2. Determine the mass flow rate of air.
109.173 – 9.81x = SOLUTION:
9.81x2 – 112.116x + 2.3705 = 0 w = P/RT = 80/0.287(10+273) = 0.985 kg/m3
By quadratic formula: m = wvA = 0.985(200)(0.40) = ___________ANS.
X = 0.02118 m = _____________ANS # Consider a refrigeration whose 40watts light bulb remains on continuously as a result of a
# A cylindrical pipe with water flowing downward at 0.03 m3/s having top diameter of 0.08, malfunction of the switch. If the refrigerator has a COP of 1.3 and the cost of electricity is 8
bottom diameter of 0.04 m and height of 1.5 m. Find the pressure between the pipe. cents per kw-hr, determine the increase in the energy consumption of the refrigeration and its
SOLUTION: cost per year if the switch is not fixed.
SOLUTION:
COP = Re / Wref
Z1 – Z2 = 1.5 m 1.3 = 40 / Wref
Z2 – Z1 = -1.5 m Wref = Wb + Wref = 40 + 30.769 = 70.77 watts
W = 0.07077 KW
V1 = Cost = 0.07077(8760)(P0.08) = ___________ANS.
# A 75 hp motor that has an efficiency of 91% is worn out and is replaced by a high-
V2 =
efficiency motor that has an efficiency motor that has an efficiency of 95.4%. Determine the
reduction in heat gain of the room due to higher efficiency under full-load conditions.
+ (-1.5) SOLUTION:
P1 – P2 = __________ANS. P01 = (75x0.746)(0.91) = 50.91 KW
# Determine the size of pipe which will deliver 8 liters of medium oil (v = 6.10 x 10-6 m2/s) P02 = (75x0.746)(0.954) = 53.376 KW
assuming laminar flow conditions. Qreduced = 53.376 – 50.91 = _________ANS.
# A household refrigerator that has a power input of 450 watts and a COP of 2.5 is to cool
five large watermelons, 10 kg each, to 8⁰C. If the watermelons are initially at 20⁰C, determine
V=
how long will it take for the refrigerator to cool them. The watermelons can be treated as water
whose specific heat is 4.2 KJ/kg-⁰K.
Re = SOLUTION:
For laminar flow, Re = 2000 COP = Re/Wc
2.5 = Re/450
2000 = Re = 1,125 watts
d = 0.835 m = ________ANS. Re = mcp(t2 – t1)
# The type of flow occupying in a 1 cm diameter pipe which water flows at a velocity of 2.50 450t = (10 x 5)(4.2)(20 – 8)
m/s. Use v=1.13x10-6 m2/s for water. t = ____________ANS.
# When a man returns to his wall-sealed house on a summer day, he finds that the house is at A heat engine receives hat from a source at 1200°K at a rate of 500KJ/s and rejects the waste
32⁰C. He returns on the air conditioner which cools the entire house to 20⁰C in 15 minutes. If heat to a medium at 300°K. The power output of the heat engine is 180 KW. Determine the
COP is 2.5, determine the power drawn by the air conditioner. Assume the entire mass within irreversibility rate for this process.
the house is 800 kg of ait which cv= 0.72 KJ/kg-K, cp=1.0 KJ/kg-K. Solution:
SOLUTION: e = (1200-300) / 1200 = 0.75
Re = mcv(t2 – t1) = (800/15x60)(0.72)(32 – 20) W= 0.75(500) = 375 KW
Re = 7.68 KW Irreversibilities = 375-180 = _____________ANS
Wc = 7.68 / 2.5 = ___________ANS. A dealer advertises that he has just received a shipment of electric resistance heaters for
residential buildings that have an efficiency of 100 percent. Assuming an indoor temperature
A heat source at 800°K losses 2000 KJ of heat to a sink at 500°K. Determine the entropy of 21°C and outdoor temperature of 10°C, determine the second law efficiency of these
generated during the process. heaters.
Solution: Solution:
∆Ssource = -2000/800 = -2.5 COP1 = 100 efficient = 1
∆Ssink = 2000/500 = 4 COP2 = (21+273) / (21-10) = 26.72
∆Sgen = -2.5/4 = _____________ANS esl = COP1 / COP2 = 1 / 26.72 = _____________ANS
Helium gas in compressed in an adiabatic compressor from an initial state of 14 psia and 50°F A thermal power plant has a heat rate of 11,363 Btu/KW-hr. Find the thermal efficiency of the
to a final temperature of 320°F in a reversible manner. Determine the exit pressure of Helium. plant.
Solution: Solution:
T2/T1 = (P2/P1)n-1/n e = 3412 / Heat rate = 3412 / 11363 = _____________ANS
(320+460)(50+460) = (P2/14)1.667-1/1.667 A rigid tank contains 2 kmol of N2 and 6 kmol of CO2 gases at 300°K and 115 Mpa. Find the
P2 = _____________ANS tank volume using ideal gas equation
Air pass thru a nozzle with efficiency of 90%. The velocity of air at the exit is 600 m/s. Find Solution:
the actual Velocity at the exit. Pm Vm = Nm Ru Tm
Solution: 15,000 Vm = (6+2)(8.314)(300)
e = (Va/V3)2 Vm = _____________ANS
0.9 = (Va/600)2 A spherical balloon with a diameter of 6 m is filled with helium at 20°C and 200 kpa.
Va = _____________ANS Determine the mole number.
A 50 kg block of iron casting at 500°K is thrown into a large lake that is at a temperature of Solution:
285°K. The iron block eventually reaches the thermal equilibrium with the lake water. PV=NRT
Assuming average specific heat of 0.45 KJ/kg-K for the iron, determine the entropy generated
during the process. (200) = N (8.314) (20+273)
Solution: N = _____________ANS
∆Siron = m c ln(T2/T1) = 50(0.45)ln (285/500) = -12.65 KJ/K The air is an automobile tire with a volume of 0.53 ft3 is at 90°Fand 20 psig. Determine the
∆Slake = Q/T = [50(0.45)(500-285)] / 285 = 16.97 KJ/K amount of air that must be added to raise the pressure to the recommended value of 30 psig.
∆Sgen = -12.65 + 16.97 = _____________ANS Assume the atmospheric pressure to be 14.7 psia and the temperature and the volume to
A windmill with a 12 m diameter rotor is to be installed at a location where the wind is remain constant.
blowing at an average velocity of 10 m/s. Using standard condition of air (1 atm, 25°C), Solution:
determine the maximum power that can be generated by the windmill. PV=mRT
Solution: (20+14.7)(144)(0.53) = m1 (53.3)(90+460)
w = P/RT = 101.325 / (0.287)(25+273) = 1.1847 kg/m3 m1 = 0.09034 lb
m = w A v = 1.1847(π/4 x 122)(10) = 1,339.895 kg/s (30+14.7)(144)(0.53) = m2 (53.3)(90+460)
KE = v2/2000 = 102/2000= 0.05 KJ/kg m2 = 0.11634 lb
Power = m KE = 1,339.895(0.05) = _____________ANS madded = m2 – m1 = 0.11634 - 0.09034 = _____________ANS
Consider a large furnace that can supply heat at a temperature of 2000°R at a steady rate of A rigid tank contains 20 lbm of air at 20 psia and 70°F. More air is added to the tank until the
3000 Btu/s. Determine the exergy of this energy. Assume an environment temperature of pressure and temperature rise to 35 psia and 90°F, respectively. Determine the amount of air
77°F. added to the tank.
Solution: Solution:
P1 V1 = m1 R1 T1
(20 x 144) (V1) = 20(53.3)(70+460)
W = e Q = 0.7315 (3000) = 2194.5 Btu/s = _____________ANS V = 196.17 ft3
P2 V2 = m2 R2 T2
(35 x 144)(196.17) = m2(53.3)(90+460)
m2 = 33.73 lbs Solution:
madded = m2 – m1 = 33.73 – 20 = _____________ANS Qc = k A (t2-t1) = 15(π x 0.30 x 1.7) (34-20) = _____________ANS
A rigid tank contains 5 kg of an ideal gas at 4 atm and 40°C. Now a valve is opened and half A 5 cm diameter spherical ball whose surface is maintained at a temperature of 70°C is
of mass of the gas is allowed to escape. If the final pressure in the tank is 1.5 atm, the final suspended in the middle of a room at 20°C. If the convection heat transfer coefficient is 15
temperature in the tank is: W/m2-°C and the emissivity of the surface is 0.8, determine the total heat transfer from the
Solution: ball.
PV=mRT Solution:
(4 x 9.81)(V) = 5(0.287)(40+273) A = 4 π r2 = 4 π (0.05)2 = 0.0314 m2
V = 11.446 m3 Qc = h A (t2-t1) = 15(0.0314) (70-20) = 23.56 watts
PV=mRT Qr = (0.80) (5.67 x 10-8) (0.0314) [(70+273)4-(50+273)4] = 9.22 watts
(1.5 x 9.81)(11.446) = (5/2)(0.287)(T) Q = Qc + Qr = 23.56 + 9.22 = _____________ANS
T = _____________ANS A frictionless piston-cylinder device and a rigid tank contain 1.2 kmol of an ideal gas at the
The pressure of an automobile tire is measured to be 200 kpa(gage) before the trip and 220 same temperature, pressure, and volume. Now heat is transferred, and the temperature of both
kpa(gage) after the tip at a location where the atmospheric pressure is 90 kpa. If the system is raised by 15°C. The amount of extra heat that must be supplied to the gas in the
temperature of the air in the tire before the trip is 25°C, the air temperature after the trip is: cylinder that is maintained at constant pressure.
Solution: Solution:
T2/T1 = P2/P1 Q = m Cp (t2-t1) = (1.2 x 8.314)(1)(15) = _____________ANS
T2 / (25+273) = (220+90) / (200+90) A supply of 50 kg of chicken at 6°C contained in a box to be frozen to -18°C in a freezer.
T2 = 318.55°K Determine the amount of heat that needs to be removed. The latent heat of the chicken is 247
t2 = _____________ANS KJ/kg, and its specific heat is 3.32 KJ/kg-C above freezing and 1.77 KJ/kg-C below freezing.
Water is boiling at 1 atm pressure in a stainless steel pan on an electric range. It is observed The container box is 1.5 kg, and the specific heat of the box material is 1.4 KJ/kg-C. Also the
that 2 kg of liquid water evaporates in 30 min. The rate of heat transfer to the water is: freezing temperature of chicken is -2.8°C.
Solution: Solution:
Qchicken = 50[3.32(6+2.8) + 247 + 1.77(-2.8+18)] = 15,156 KJ
Q = mL = = _____________ANS Qbox = 1.5(1.4)(6+18) = 50.4 KJ
Consider a person standing in a breezy room at 20°C. Determine the total rate of heat transfer Q = 15,156 + 50.4 = _____________ANS
from this person if the exposed surface area and the average outer surface temperature of the Water is being heated in a closed pan on top of a range while being stirred by a paddle wheel.
person are 1.6 m2 and 29°C, respectively, and the convection heat transfer coefficient is 6W/ During the process, 30 KJ of heat is transferred to the water and 5 KJ of heat is lost to the
m2 with emissivity factor of 0.95. surrounding air. The paddle-wheel work amounts to 500 N-m. Determine the final energy of
Solution: the system if its initial energy is 10 KJ.
Qc = h A (t2-t1) = (6)(1.6)(29.20) = 86.40 watts Solution:
Qf = (0.95)(5.67 x 10-8)(1.6)[(29+273)4-(20+273)4] = 81.7 watts Final energy = QA + ∆U – Qlose + W = 30 + 10 – 5 + 0.50 = _____________ANS
Q = Qc + Qf = 86.40 + 81.7 = _____________ANS A classroom that normally contains 40 people is to be air-conditioned with window air-
Water is boiled in a pan on a stove at sea level. During 10 minutes of boiling, it is observed conditioning units of 5 KW cooling capacity. A person at rest may be assumed to dissipate
that 200 grams of water has evaporated. Then the rate of heat transfer to the water is: heat at rate of about 360 KJ/hr. There are 10 light bulbs in the room, each with a rating of 100
Solution: watts. The rate of heat transfer to the classroom through the walls and the windows is
Q = mL = (0.2/10) (2257) = _____________ANS estimated to be 15,000 KJ/hr. If the room to be maintained at a constant temperature of 21°C,
An aluminum pan whose thermal conductivity is 237 W/m-C has a flat bottom whose diameter determine the number of window air-conditioning units required.
is 20 cm and thickness 0.4 cm. Heat is transferred steadily to boiling water in the pan through Solution:
its bottom at a rate of 500 watts. If the inner surface of the bottom of the pan is 105°C, Q = total heat load = 40(360/3600) + 10(0.100) +15,000/3600 = 9.167 KW
determine the temperature of the surface of the bottom of the pan. No. of air conditioning = 9.167/5 = 1.833 = _____________ANS
Solution: A 4m x 5m x 6m room is to be heated by a baseboard resistance heater. It is desired that the
A = π/4 (0.20)2 = 0.0314m2 resistance heater be able to raise the air temperature in the room from 7 to 23°C within 15
minutes. Assuming no heat losses from the room and an atmospheric pressure of 100 kpa,
determine the required power of the resistance heater. Assume constant specific heats at room
temperature.
Solution:
t2 = _____________ANS w = P/RT = 100 / (0.287)(7+273) = 1.244 kg/m3
For a heat transfer purposes, a standing man can be modeled as a 30 cm diameter, 170 cm long m = 1.244(4 x 5 x 6) = 149.28 kg
vertical cylinder with both the top and bottom surfaces insulated and with the side surface at Q= m Cv (t2-t1) = 149.28(0.7186)(23-7) = 1,716.36 KJ
an average temperature of 34°C. For a convection heat transfer coefficient of 15 W/m2-°C, Power = 1,716.36 / (15 x 60) = _____________ANS
determine the rate of heat loss from this man by convection in an environment at 20°C. A student living in a 4m x 6m x 6m dormitory room turns on her 150 watts fan before she
leaves the room on a summer day, hoping that the room will be cooler when she comes back
in the evening. Assuming all the doors and windows are tightly closed and disregarding any h = P/w = 12/9.81 = 1.223 m
heat transfer through walls and the windows, determine the temperature in the room when she Wc = w Q h = 9.81(20.843)(1.223) = _____________ANS
comes back 10 hours later. Use specific heat values at room temperature, and assume the room A plane-type of solar energy collector with an absorbing surface covered by a glass plate is to
to be at 100 kpa and 15°C in the morning when she leaves. receive an incident radiation of 800 W/m2. The glass plate has a reflectivity of 0.12 and a
Solution: transmissivity of 0.85. The absorbing surface has an absorptivity of 0.90. The area of the
w = P/RT = 100 / (0.287)(15+273) = 1.2098 kg/m3 collector is 5 m2. How much solar energy in watts is absorbed by the collector?
m = 1.2098(4 x 6 x 6) = 174.216 kg Solution:
Q= m Cv (t2-t1) Q = heat absorbed from the sun
0.15(10 x 3600) = 174.216(0.7186)(t2 - 15) Q = 16.3(π/4 x 0.008)[15-(-190)] = _____________ANS
t2 = _____________ANS An elastic sphere containing gas at 120 kPa has a diameter of 1.0 m. Heating the sphere causes
A piston-cylinder device whose piston is resting on top of a set of stops initially contains 0.50 it to expand to a diameter of 1.3 m. During the process the pressure is proportional to the
kg of helium gas at 100 kpa and 25°C. The mass of the piston is such that 500 kpa of pressure sphere diameter. Calculate the work done by the gas in KJ.
is required to raise it. How much heat must be transferred to the helium before the piston starts Solution:
rising? PαD
Solution: P = kD
For helium: Cv = R/(k-1) = (8.314/4)(1.667-1) = 3.116 KJ/kg-K 120 = k(1)
T2 = (25+273)(500/100) = 1,490°K k = 120
T1 = 25+273 = 298°K P = 120 D
Q= m Cv (T2-T1) = 0.50(3.116)(1490-298) = _____________ANS V = 4/3 π (10/2)3
In order to cool 1 ton (1000 kg) of water at 20°C in an insulated tank, a person pours 80 kg of dV = 12/24 π D2 dD
ice at -5°C into the water. Determine the final equilibrium temperature in the tank. The
melting temperature and the heat of fusion of ice at atmospheric pressure are 0°C and 333.7
KJ/kg, respectively. W = _____________ANS
Solution:
Qwater = Qice # An ideal gas with a molecular weight of 7.1 kg/kg mol is compressed from 600 kPa and 280
1000(4.187)(20-tB) = 80(2.09)(0+5) + 80(333.7) + 80(4.187)(tB-0) K to a final specific volume of 0.5 m3/kg. During the process the pressure varies according to
tB = _____________ANS p = 620 + 150v + 95v2 where p is in kPa and v in m3/kg. Calculate the work of compression
A fan is powerd by a 0.5 hp motor delivers air at a rate of of 85 m3/min. Determine the highest in KJ/kg.
value for the average velocity of air mobilized by the fan. Take the density of air to be 1.18 Solution:
kg/m3. V1 = RT/P = (8.314/7.1)(280)/(600) = 0.546 m3/kg
Solution: W=
P=wQh
0.50(0.746) = (1.18 x 0.00981)(85/80)(h) __________________ANS.

= _____________ANS # A one cubic meter container contains a mixture of gases composed of 0.02 kg-mol of
An Ocean-Thermal Energy Conversion power plant generates 10,000 KW using a warm oxygen and 0.04 kg-mol of helium gas at a pressure of 220 kPa. What is the temperature of
surface water inlet temperature of 28°C and a cold deep-water temperature of 15°C. On the this ideal gas mixture in degrees Kelvin?
basis of a 3°C drop in the temperature of the warm water and a 3°C rise in the temperature of Solution:
the cold water due to removal and addition of heat, calculate the power required in KW to V = V1 + V2
pump the cold-deep water to the surface and through the system heat exchanger if the required VT = m1R1T1/P1 + m2R2T2/P2
pumping pressure increase is 12 kpa.Assume a Carnot cycle efficiency and density of cold 1 = (0.02x32)(8.314/32)(T)/220 + (0.04x32)(8.314/4)(T)/220
water to be 1000 kg/m3. T = __________ANS.
Solution:
e = (TH-TL)/TH = [(28+273)-(15+273)(28+273)] = 0.03679 # Methyl alcohol (CH3OH) is burned with 25% excess air. How much unburned oxygen in
e = W/QA kg-mol-oxygen/kg-mol-fuel will there be in the products if the combustion is complete?
0.03679 = 10,000/QA Solution:
QA = 271,812.99 KW CH3OH + O2 +3.76N2 = CO2 + H2O + 3.76N2
QR = QA – W = 271,812.99 -10,000 = 261,813 KW CH3OH + 1.5O2 + (1.5) (3.76) N2 = 1CO2 + H2O + 1.5(3.76) N2
QR = m Cp (∆t) Consider 25% excess air:
261,813 = m (3)(4.187) CH3OH + 1.5O2 + 1.25(1.5) (3.76) N2 = 1CO2 + H2O + 1.25(1.5) (3.76) N2 + 0.25(1.5) O2
m = 20,843.32 kg/s Unburned O2 = 0.25(1.5) = _________ANS.
20,843.32 kg/s or 20,843.32 lit/s = 20.843 m3/s
# A 12 DC electrical motor draws a current of 15 amps. How much work in KJ does this a reservoir at 300K. If thermal efficiencies of both engines are the same, determine the
motor produce over a 10-minute period of operation? temperature T.
Solution: Solution:
W=E=QV e1 = e2
W = (15 x 10 x 60)(12) = 108,000 J = _____________ANS.

# A 4liter (2-liter per revolution at standard pressure and temperature) spark ignition engine T = ___________ANS.
has a compression ratio of 8 and 2200 KJ/kg heat addition by the fluid combustion.
Considering a cold air-standard Otto cycle model, how much power will the engine produce # An ideal gas mixtrure consists of 2 kmol of N2 and 6 mol of CO2. The mass fraction of CO2
when operating at 2500 rpm? is:
Solution: Solution:
w= 1.2 kg/m3 (standard density of air) M = (2/8)(28) + (6/8)(44) = 40
m = 2 li/rev x 2500 rev/min / 1.2 kg/m3 x 1 m3/1000 li x 1 min/60 sec = 0.10 kg/s R = 8.314/M = 8.314/40 = __________ANS.
e = W/QA
e = 1 – 1/81.4-1 = 0.5647 # A Carnot cycle operates between the temperature limits of 300K and 1500K, and produces
0.5647 = W/2200 600 KW of net power. The rate of entropy change of the working fluid during the head
W = 1,242.34 KJ/kg (0.10 kg/s) = 124.23 KW = _____________ANS. addition process is:
Solution:
# A simple Rankine cycle produces a 40 MW of power, 50 MW of process heated and rejects
W=
50MW of heat to the surroundings. What is the utilization factor of this cogeneration cycle
neglecting the pump work? 600 =
Solution:
QA = WT + Wprocess WP = 40 + 50 + 50 = 140 KW = ___________ANS.
UF = (Qprocess + WT)/QA = (50 +40)/140 = _______________ANS.
# Air in an ideal Diesel cycle is compressed from 3 L to 0.15 L and then it expands during the
# The rate of heat transfer to the surroundings from a person at rest is about 400 KJ/hr. constant pressure heat addition process to 0.3 L. Under cold air standard conditions, the
Suppose that the ventilation system fails in an auditorium containing 120 people and assuming thermal efficiency of this cycle is:
that the energy goes into the air of volume 1500 m3 initially at 300K and 101 kPa, calculate Solution:
the rate in °C/min of air temperature change. rK = 3/0.15 = 20
Solution: rC = 0.3 / 0.15 = 2
e = 1 – (1/rKk-1)[(rCK – 1)/k(rC – 1)] = 0.6467 = ________ANS.
Q = m CV
PV = mRT # Helium gas in an ideal Otto cycle is compressed from 20°C and 2 L to 0.25 L and its
101(1500) = m(0.287)(300) temperature increases by an additional 800°C during the head addition process. The
m = 1,759.58 kg temperature of helium before the expansion process is:
Solution:
Q = m CV
rk = 2/0.25 = 8
120(400/60) = 1759.58(0.7186) T2 = (20 + 273)(8)1.667-1 = 1,172 K
T3 = T2 + 800 = 1172 + 800 = 1972K = ______________ANS.
= __________ANS.
# In an ideal Otto cycle, air is compressed from 1.20 kg/m3 and 2.2 L to 0.26 L and the net
# An insulated box containing helium gas falls from a balloon 4.5 km above the earth’s work output of the cycle is 440 KJ/kg. The mean effective pressure for the cycle is:
surface. Calculate the temperature rise in °C of the helium when box hits the ground. Solution:
Solution: VD = V2 – V1 = 2.2 x 10-3 m3 – 0.26 x 10-3 m3 = 1.94 x 10-3 m3
Cv of helium = 3118.9 J/kg-C W = 440 KJ/kg (1.2 kg/m3 x 2.2 x 10-3 m3) = 1.1616 KJ
m g h = m Cv Pm = W/VD = 1.1616 / 1.94 x 10-3 = _____________ANS.
m (9.81) (4500) = m (3118.9) # An ideal Brayton cycle has a net work output of 150 KJ/kg and backwork ratio of 0.4. Of
= ___________ANS. both the turbine and the compressor had an isentropic efficiency of 80%, the net work output
of the cycle would be?
# Consider two Carnot heat engines operating in series. The first engine receives heat from the Solution:
reservoir at 2400 K and rejects the waste heat to another reservoir at temperature T. The Backwork Ratio = WC / WT
second engine receives heat by the first one, convert some of it to work, and rejects the rest to WC = 0.40 WT
Wnet = WT - WC A. 64.67 B. 46.67 C. 84.67 D. 74.84
Wnet = 250 KJ/kg A # It is specified that the angular deformation in a shaft should not exceed 1” in length of
Wnet’ = 250(0.80) – 0.40(200)/0.80 = ___________ANS. 1.8m. The allowable shearing stress is 53 Mpa. Determine the diameter of the shaft. The shaft
material has G = 77 x 106 Mpa.
# Air enters a turbojet engine at 200 m/s at a rate of 20 kg/s, and exits at 800 m/s relative to A. 222.34 mm B. 234.22 mm C. 23.42 cm D. 24.22 cm
the aircraft. The thrust developed by the engine is: B # What modulus of elasticity in tension is required to obtain a unit deformation of 0.00105
Solution: from a load producing a unit stress of 45,000 psi?
Thrust developed = m (v2 – v1) = 20(800-200) = ____________ANS. A. 40 x 106 psi B. 43 x 106 psi C. 45 x 106 psi D. 46 x 106
A # A thrust washer has an inside diameter of 0.5 inch and an outside diameter of 3 inches.
# A thermal power plant has a net power 10 MW. The backwork ratio of the plant is 0.005. For an allowable bearing pressure of 90 psi, how much axial load can it sustain?
Determine the compressor work. A. 618.5 lb B. 537.2 lb C. 702.2 lb D. 871.2 lb
Solution: D # An air cylinder has a bore of 25 mm and is operated with shop air at a pressure of 90 ps.
Wnet = WT + WP Find the push force extended by the piston rod in N.
BW = WP / WT A. 127 B. 70 C. 402 D. 305
WP = 0.005WT B # A line shaft is to transmit 200 Hp at 900 rpm. Find the diameter of the shaft.
10,000 = WT – 0.005WT A. 2.18 inches B. 2.26 inches C. 3.18 inches D. 3.26 inches
WT = 10,050.25 KW B # A main transmitting shaft transmits 350 KW to drive a generator at 2500 rpm, what is the
WC = 0.005(10,050.25) = ____________ANS. required diameter of the shaft?
# A heat engine receives heat from a source at 1200K at a rate of 500 KJ/s and rejects the A. 58.5 mm B. 82.7 mm C. 85.3 mm D. 56.2 mm
waste heat to a sink at 300K. If the power output of the engine is 200 KW, the second law D # A round steel shaft rotates at 200 rpm and is subjected to a torque of 275 N-m and a
efficiency of this heat engine is? bending moment of 415 N-m. Determine the equivalent twisting moment.
Solution: A. 597.84 N-m B. 456.42 N-m C. 546.43 N-m D. 497.85 N-m
ea = 200/500 = 0.40 A # A 4 inches shaft using a flat key, whose width is 1 inch, is transmitting a torque of 53,000
et = (TH - TL)/TH = (1200-300)/1200 = 0.75 in-lb. If the design shearing stress is 5000 psi, determine the safe length of key.
es = 0.40/0.75 = _________ANS. A. 6.3 inches B. 5.3 inches C. 4.3 inches D. 7.0 inches
A # a 75 mm diameter shaft is transmitting 300 KW at 600 rpm. A flange coupling is used and
# A water reservoir contains 100,000 kg of water at an average elevation of 60m. The has 6 bolts, each 18 mm in diameter. Find the required diameter if the bolt circle based on an
maximum amount of electric power that can be generated from this water is: average shearing stress of 27.5 Mpa.
Solution: A. 227.4 mm B. 477.2 mm C. 274.7 mm D. 247.7 mm
P = mh = (100,000x0.00981)(60) = 58,860 KJ C # A heavy duty shaft coupling is to be secured woth 25 mm bolts at a distance of 150 mm
P = 58,860 KJ x KWh/3600KJ = ____________ANS. from the shaft center. The shaft transmits 4330 KW of power at a speed of 1200 rp,. If the
allowable shearing stress for bolts is 100 Mpa, how many bolts are required?
# A house is maintained at 22°C in winter by electric resistance heaters. If the outdoor A. 3 B. 6 C. 5 D. 4
temperature is 5°C, the second law efficiency of the resistance heaters is: B # A 1.75-inch diameter shaft is supported by two sleeve bearings. The total load on the wo
Solution: bearings is 2900 lb. Find the friction power loss, in Hp, if the coefficient of friction between
ea = 100% resistance heaters shaft and bearing is 0.10 and the shaft rotates 200 rpm.
et = (22-15)/(22+273) = 5.8% A. 0.88 Hp B. 0.78 Hp C. 0.98 Hp D. 0.68 Hp
es = 5.8/100 = _________ANS. B # Find the horsepower lost when a collar is loaded with 1000 lb, rotates at 25 rpm, and a
coefficient of friction at 0.15. The outside diameter of the collar is 4 inches and the inside
# A thermoelectric refrigerator that resembles a small ice chest is powered by a car battery, diameter is 2 inches.
and has a COP of 0.10. If the refrigerator cools a 0.350 L canned drink from 20°C to 4°C in 30 A. 0.0629 Hp B. 0.0925 Hp C. 0.0269 Hp D. 0.0692 Hp
min, determine the average electric power consumed by the thermoelectric refrigerator. C # A sleeve bearing has an outside diameter of 38.1 mm and a length of 50.1 mm, the wall
Solution: thickness is 3/16 inch. The bearing is subjected to radial load of 450 lb, determine the bearing
Q = m cp (t2 – t1) =[(1 x 0.35)/(30 x 60)] (4.187) (20-4) = 13 watts pressure.
COP = RE / WC A. 100 psi B. 150 psi C. 200 psi D. 250 psi
0.10 = 13/0.10 = __________ANS. A # A vertical steel cylinder water tank is 30 m in diameter and 45 m high. The allowable
stress of the steel plate is 120 Mpa. Without reinforcing angle bars and rods, determine the
MACHINE DESIGN ELEMENTS thickness of the steel plate.
A. 55.2 mm B. 56.2 mm C. 65.2 mm D. 52.6 mm
C # What is the polar section modulus of a 4-inch solid shaft?
A. 25.13 in3 B. 12.57 in4 C. 12.57 in3 D. 25.13 in4 B # Determine the bursting steam pressure of a hemispherical steel shell with a diameter of
A # A hollow shaft carries a torque 3.4 KN-m at a shearing stress if 55 Mpa. The outside 100 inches and made of 0.0635 m thick steel plate. The joint efficiency is 70% and the tensile
diameter is 1.25 times that of the inside diameter. Find the inside diameter in mm. strength is 60,000 psi.
A. 4020 psi B.4200 psi C. 2500 psi D. 2040 psi A. 2.02 inch B. 6.06 inch C. 4.04 inch D. 2.06 inch
A # A cylinder having an internal diameter of 508 mm and external diameter of 814.4 mm is A # A right-handed single-thread hardened steel worm has a catalog rating of 2.25 KW at 650
subjected to an internal pressure of 69 Mpa and an external of 14 Mpa. Determine the hoop rpm when meshed with a 48-tooth cast-steel gear. The axial pitch of the worm is 25 mm,
stress at the inner surface of the cylinder normal pressure angle is 14.5o, and the pitch diameter of the worm is 100 mm. The coefficient
A. 90.11 Mpa B. 91.10 Mpa C. 911.0 Mpa D. 19.10 Mpa of friction is 0.085. Determine the shaft center distance.
A # The root diameter of a double square thread is 0.55 inch. The screw has a pitch of 0.2 A. 241 mm B. 142 mm C. 412 mm D. 124 mm
inch. Find the outside diameter and the number of threads per inch. A # A 20o straight-tooth bevel pinion having 14 teeth and a diametral pitch of 6 tooth/inch
A. 0.75 inch and 5 threads/inch C. 0.50 inch and 5 threads/inch drives a 42-tooth gear. The two shafts are at right angles and in the same plane. Find the pitch
B. 0.75 inch and 4 threads/inch D. 0.50 inch and 4 threads/inch angle of the pinion.
B # Two shafts 3.6 m between centers carry pulleys 1.2 m in diameter and 0.91 m in diameter A. 18.4o B. 20o C. 14.5o D. 20.5o
respectively, connected by a crossed belt. It is desired to put the belt on as an open belt. How A # A triple thread worm has a lead angle of 17o and a pitch diameter of 2.2802 inches. Find
long a piece must be cut of it? the center distance when the worm is mated with a wheel of 48 teeth.
A. 300 mm B. 350 mm C. 400 mm D. 250 mm A. 6.72 inches B. 7.26 inches C. 6.27 inches D. 7.62 inches
B # A flat belt is 6 inches wide and 1/3 inch thick and transmits 15 Hp. The center distance is B # A double thread worm has a pitch diameter of 3 inches. The wheel has 20 teeth and a pitch
8 ft. The driving pulley is 6 inches in diameter and rotates at 2000 rpm such that the loose side diameter of 5 inches. Find the gear helix angle.
of the belt is on top. The driven pulley is 18 inches in diameter. The belt material is 0.035 A. 4.69o B. 9.46o C. 6.49o D. 6.94o
lb/in3 and the coefficient of friction is 0.30. Determine the net belt tension. C # What is the polar section modulus of a 4-inch solid shaft?
A. 167.56 lb B. 157.56 lb C. 156.75 lb D. 175.56 lb A. 25.13 in3 B. 12.57 in4 C. 12.57in3 D. 25.13in4
A # Two pulleys, 80 cm apart, carry a belt in an open connection. If the diameter of the A # A hollow shaft carries a torque 3.4 KN-m at a shearing stress Mpa. The outside diameter
pulleys are 40 cm and 15 cm, what is the length of the belt needed? is 1.25 times that of the inside diameter. Find the inside diameter in mm.
A. 248.35 cm B. 348.35 cm C. 265.45 cm D. 305.35 cm A. 64.87 B. 46.87 C. 84.67 D. 74.64
A # A pulley 600 mm in diameter transmits 40 KW at 500 rpm. The arc of contact between the A # It is specified that the angular deformation in a shaft should not to exceed 1” in length of
belt and pulley is 144o , the coefficient of friction between belt and pulley is 0.35 and the safe 1.8 m. The allowable shearing stress is 83 Mpa. Determine the diameter of the shaft. The shaft
working stress of the belt is 2.1 Mpa. Determine the belt tension ratio, neglecting the effect of material has G = 77 x 108 Mpa.
centrifugal force. A. 222.34 mm B. 234.22 mm C. 23.42 mm D. 24.22 mm
A. 2.41 B. 2.14 C. 1.24 D. 4.12 B # What modulus of elasticity in tension is required to obtain a unit deformation of 0.00105
B # A roller chain and sprocket is to drive vertical discharge bucket elevator. The pitch of from a load producing a unit stress of 45,000 psi?
chain connecting sprockets is 1.75”. The sprocket is rotating at 120 rpm and has 11 teeth while A. 40 x 106 psi B. 43 x 106 psi C. 45 x 106 psi D. 46 x 106 psi
the driven sprocket is rotating at 38 rpm. Determine the number of teeth of driven sprocket. A # A thrust washer has an inside diameter of 0.5 inch and an outside diameter of 3 inches.
A. 33 teeth B. 35 teeth C. 30 teeth D. 37 teeth For an allowable bearing pressure of 90 psi, how much axial load can it sustain?
C # A disc clutch has 6 pairs of contacting friction surfaces with an outside diameter of 200 A. 618.5 lb B. 537.2 lb C. 702.2 lb D. 871.2 lb
mm and an inside diameter of 100 mm. The coefficient of friction of the clutch materials is 0.4 D # An air cylinder has a bore of 25 mm and is operated with shop air at a pressure of 90 psi.
and the axial force is 1500 N. The shaft speed is 1200 rpm. Find the Hp that can be transmitted Find the push force exerted by the piston rod in N.
by the clutch assuming uniform pressure. A. 127 B. 70 C. 402 D. 305
A. 35.2 Hp B. 23.5 Hp C. 47.2 Hp D. 27.4 Hp B # A line shaft is to transmit 200 Hp at 900 rpm. Find the diameter of the shaft.
A # Determine the power capacity of a cone clutch under uniform pressure and assuming the A. 2.18 inches B. 2.28 inches C. 3.18 inches D. 3.28 inches
following pressure and assuming the following conditions; major diameter = 250 mm, minor B # A main transmitting shaft transmits 350 KW to drive a generator at 2500 rpm, what is the
diameter = 200 mm, length of conical elements in contact-125 mm, rotating speed = 870 rpm, required diameter of the shaft?
coefficient of friction = 0.30, and allowable pressure = 70 kpa. A.58.5 mm B. 62.7 mm C. 65.3 mm D. 56.2 mm
A. 19.2 KW B. 21.9 KW C. 29.1 KW D. 12.9 KW D # A round steel shaft rotates at 200 rpm and is subjected to a torque of 275 N-m and a
D # A flywheel has a mean diameter of 4ft and is required to handle 2250 ft-lb of kinetic bending moment of 415 N-m. Determine the equivalent twisting moment.
energy. It has a width of 8 inches, mean operating speed is 300 rpm and the coefficient of A. 597.84 N-m B. 456.42 N-m C. 546.43 N-m D. 497.85 N-m
fluctuation is to be 0.05. Find the weight of rim, assuming that the arms and hub are equivalent A # A 4 inches shaft using a flat key, whose width is 1 inch, is transmitting a torque of 63,000
to 10% of the total rim weight. The flywheel is made up of cast iron with specific weight of in-lb. If the design shearing stress is 5000 psi, determine the safe length of key.
0.26 lb per cubic inch. A. 6.3 inches B. 5.3 inches C. 4.3 inches D. 7.0 inches
A. 334 lb B. 434 lb C. 433 lb D. 343 lb A # A 1200 mm cast iron pulley is fastened to a 112.5 mm shaft by means of a 28.13 mm
A # A 20o involute spur gear has a tooth whole depth of 16.95 mm, a tooth thickness of 13.2 square key 175 mm long. The key and the shaft have a shearing stress of 14,000 psi.
mm, and a pitch of 3. Determine the circular pitch of the gear. Determine the force acting at the pulley rim that will shear the key.
A. 26.6 mm B. 16.6 mm C. 25.6 mm D. 24.6 mm A. 10015 lb B. 11005 lb C. 11050 lb D. 10501 lb
C # A parallel helical gear-set consists of a 19-tooth pinion driving a 57-tooth gear. The pinion A # A 75 mm diameter shaft is transmitting 300 KW at 600 rpm. A flange coupling is used
has a left-hand helix of 20o, a normal pressure angle of 14 1/2 deg . and a normal diametral and has 6 bolts, each 18 mm in diameter. Find the required diameter of the bolt circle based on
pitch of 10 tooth/inch. If the pinion is to transmit 50 Hp at a speed of 1750 rpm, determine the an average shearing stress of 27.5 Mpa.
center distance of the two gears. A. 227.4 mm B. 477.2 mm C. 274.7 mm D. 247.7 mm
C # A heavy duty shaft coupling is to be secured with 25 mm bolts at a distance of 150 mm A. 33 teeth B. 35 teeth C. 30 teeth D. 37 teeth
from the shaft center. The shaft transmits 4330 KW of power at a speed of 1200 rpm. If the C # A disc clutch has 6 pairs of contacting friction surfaces with an outside diameter of 200
allowable shearing stress for bolts is 100 Mpa, how many bolts are required? mm and an inside diameter of 100 mm. The coefficient of friction of the clutch materials is 0.4
A. 3 B. 6 C. 5 D. 4 and the axial force is 1500 N. The shaft speed is 1200 rpm. Find the Hp that can be transmitted
B # A 1.75-inch diameter shaft is supported by two sleeve bearings. The total load on the two by the clutch assuming uniform pressure.
bearings is 2800 lb. Find the friction power loss, in Hp, if the coefficient of friction between A. 35.2 Hp B. 23.5 Hp C. 47.2 Hp D. 27.4 Hp
shaft and bearing is 0.10 and the shaft rotates 200rpm. A # Determine the power capacity of a cone clutch under uniform pressure and assuming the
A. 0.88 Hp B. 0.78 Hp C. 0.98 Hp D. 0.68 Hp following pressure and assuming the following conditions; major diameter = 250 mm, minor
B # Find the horsepower lost when a collar is loaded with 1000 lb, rotates at 25 rpm, and a diameter = 200 mm, length of conical elements in contact-125 mm, rotating speed = 870 rpm,
coefficient of friction at 0.15. The outside diameter of the collar is 4 inches and the inside coefficient of friction = 0.30, and allowable pressure = 70 kpa.
diameter is 2 inches. A. 19.2 KW B. 21.9 KW C. 29.1 KW D. 12.9 KW
A. 0.0629 Hp B. 0.0926 Hp C. 0.0269 Hp D. 0.0692 Hp D # A flywheel has a mean diameter of 4ft and is required to handle 2250 ft-lb of kinetic
C # A sleeve bearing has an outside diameter of 38.1 mm and a length of 50.1 mm, the wall energy. It has a width of 8 inches, mean operating speed is 300 rpm and the coefficient of
thickness is 3/16 inch. The bearing is subjected to radial load of 450 lb, determine the bearing fluctuation is to be 0.05. Find the weight of rim, assuming that the arms and hub are equivalent
pressure. to 10% of the total rim weight. The flywheel is made up of cast iron with specific weight of
A. 100 psi B. 150 psi C. 200 psi D. 250 psi 0.26 lb per cubic inch.
A # A vertical steel cylinder water tank is 30 m in diameter and 45 m high. The allowable A. 334 lb B. 434 lb C. 433 lb D. 343 lb
stress of the steel plate is 120 Mpa. Without reinforcing angle bars and rods, determine the A # A 20o involute spur gear has a tooth whole depth of 16.95 mm, a tooth thickness of 13.2
thickness of the steel plate. mm, and a pitch of 3. Determine the circular pitch of the gear.
A. 55.2 mm B. 56.2 mm C. 65.2 mm D. 52.6 mm A. 26.6 mm B. 16.6 mm C. 25.6 mm D. 24.6 mm
B # Determine the bursting steam pressure of a hemispherical steel shell with a diameter of C # A parallel helical gear-set consists of a 19-tooth pinion driving a 57-tooth gear. The pinion
100 inches and made of 0.0635 m thick steel plate. The joint efficiency is 70% and the tensile has a left-hand helix of 20o, a normal pressure angle of 14 1/2 deg . and a normal diametral
strength is 98 psi. pitch of 10 tooth/inch. If the pinion is to transmit 50 Hp at a speed of 1750 rpm, determine the
A. 4020 psi B. 4200 psi C. 2500 psi D. 2040 psi center distance of the two gears.
A # A cylinder having an internal diameter of 508 mm and external diameter of 914.4 mm is A. 2.02 inch B. 6.06 inch C. 4.04 inch D. 2.06 inch
subjected to an internal pressure of 69 Mpa and an external of 14 Mpa. Determine the hoop A # A right-handed single-thread hardened steel worm has a catalog rating of 2.25 KW at 650
stress at the side surface of the cylinder. rpm when meshed with a 48-tooth cast-steel gear. The axial pitch of the worm is 25 mm,
normal pressure angle is 14.5o, and the pitch diameter of the worm is 100 mm. The coefficient
A. 90.11 Mpa B. 91.10 Mpa C. 911.0 Mpa D. 19.10 Mpa of friction is 0.085. Determine the shaft center distance.
A # The root diameter of a double square thread is 0.55 inch. The screw has a pitch of 0.2 A. 241 mm B. 142 mm C. 412 mm D. 124 mm
inch. Find the outside diameter and the number of threads per inch. A # A 20o straight-tooth bevel pinion having 14 teeth and a diametral pitch of 6 tooth/inch
A. 0.75 inch and 5 threads/inch C. 0.50 inch and 5 threads/inch drives a 42-tooth gear. The two shafts are at right angles and in the same plane. Find the pitch
A. 0.75 inch and 4 threads/inch D. 0.50 inch and 4 threads/inch angle of the pinion.
B # Two shafts 3.6 m between centers carry pulleys 1.2 m in diameter and 0.91 m in diameter A. 18.4o B. 20o C. 14.5o D. 20.5o
respectively, connected by a crossed belt. It is desired to put the belt on as an open belt. How A # A triple thread worm has a lead angle of 17o and a pitch diameter of 2.2802 inches. Find
long a piece must be cut of it? the center distance when the worm is mated with a wheel of 48 teeth.
A. 300 mm B. 350 mm C. 400 mm D. 250 mm A. 6.72 inches B. 7.26 inches C. 6.27 inches D. 7.62 inches
B # A flat belt is 6 inches wide and 1/3 inch thick and transmits 15 Hp. The center distance is B # A double thread worm has a pitch diameter of 3 inches. The wheel has 20 teeth and a pitch
8 ft. The driving pulley is 6 inches in diameter and rotates at 2000 rpm such that the loose side diameter of 5 inches. Find the gear helix angle.
of the belt is on top. The driven pulley is 18 inches in diameter. The belt material is 0.035 A. 4.69o B. 9.46o C. 6.49o D. 6.94o
lb/in3 and the coefficient of friction is 0.30. Determine the net belt tension. C # What is the polar section modulus of a 4-inch solid shaft?
A. 167.56 lb B. 157.56 lb C. 156.75 lb D. 175.56 lb A. 25.13 in3 B. 12.57 in4 C. 12.57in3 D. 25.13in4
A # Two pulleys, 80 cm apart, carry a belt in an open connection. If the diameter of the A # A hollow shaft carries a torque 3.4 KN-m at a shearing stress Mpa. The outside diameter
pulleys are 40 cm and 15 cm, what is the length of the belt needed? is 1.25 times that of the inside diameter. Find the inside diameter in mm.
A. 248.35 cm B. 348.35 cm C. 265.45 cm D. 305.35 cm A. 64.87 B. 46.87 C. 84.67 D. 74.64
A # A pulley 600 mm in diameter transmits 40 KW at 500 rpm. The arc of contact between the A # It is specified that the angular deformation in a shaft should not to exceed 1” in length of
belt and pulley is 144o , the coefficient of friction between belt and pulley is 0.35 and the safe 1.8 m. The allowable shearing stress is 83 Mpa. Determine the diameter of the shaft. The shaft
working stress of the belt is 2.1 Mpa. Determine the belt tension ratio, neglecting the effect of material has G = 77 x 108 Mpa.
centrifugal force. A. 222.34 mm B. 234.22 mm C. 23.42 mm D. 24.22 mm
A. 2.41 B. 2.14 C. 1.24 D. 4.12 B # What modulus of elasticity in tension is required to obtain a unit deformation of 0.00105
B # A roller chain and sprocket is to drive vertical discharge bucket elevator. The pitch of from a load producing a unit stress of 45,000 psi?
chain connecting sprockets is 1.75”. The sprocket is rotating at 120 rpm and has 11 teeth while A. 40 x 106 psi B. 43 x 106 psi C. 45 x 106 psi D. 46 x 106 psi
the driven sprocket is rotating at 38 rpm. Determine the number of teeth of driven sprocket.
A # A thrust washer has an inside diameter of 0.5 inch and an outside diameter of 3 inches. D # A 35-tooth pinion turning at 300 rpm drives a 120-tooth gear of 14.5° involute full depth
For an allowable bearing pressure of 90 psi, how much axial load can it sustain? pressure angle. Determine the rpm of the driven gear.
A. 618.5 lb B. 537.2 lb C. 702.2 lb D. 871.2 lb A. 60 rpm B. 45 rpm C. 75 rpm D. 90 rpm
D # An air cylinder has a bore of 25 mm and is operated with shop air at a pressure of 90 psi. B # Two parallel shafts connected by cylinders in pure rolling contact and turning in the same
Find the push force exerted by the piston rod in N. direction having a speed ratio of 2.75. Determine the center distance of the two shafts if the
A. 127 B. 70 C. 402 D. 305 diameter of the smaller cylinder is 22 cm.
B # A line shaft is to transmit 200 Hp at 900 rpm. Find the diameter of the shaft. A.18.25 B. 19.25 cm C. 20.25 cm D. 17.25 cm
A. 2.18 inches B. 2.28 inches C. 3.18 inches D. 3.28 inches A # Three extension springs are hooked in parallel that supports a single weight of 100 kg.
B # A main transmitting shaft transmits 350 KW to drive a generator at 2500 rpm, what is the The first spring is rated at 4 KN/m and the other two springs are rated at 6 KN/m each.
required diameter of the shaft? Determine the equivalent stiffness of the three springs.
A. 58.5 mm B. 62.7 mm C. 65.3 mm D. 56.2 mm A. 1.71 KN/m B. 5 KN/m C. 2.71 KN/m D. 3.71 KN/m
D # A round steel shaft rotates at 200 rpm and is subjected to a torque of 275 N-m and a B # Three extension springs are hooked in parallel that support a single weight of 100 kg. The
bending moment of 415 N-m. Determine the equivalent twisting moment. springs are rated 4 KN/m, 5 KN/m, and 6 KN/m, respectively. Determine the equivalent spring
A. 597.84 N-m B. 456.42 N-m C. 546.43 N-m D. 497.85 N-m constant of the three springs.
A # A 4 inches shaft using a flat key, whose width is 1 inch, is transmitting a torque of 63,000 A. 10 KN/m B. 15 KN/m C. 9 KN/m D. 11 KN/m
in-lb. If the design shearing stress is 5000 psi, determine the safe length of key. A # Two extension coil springs are hooked in series that support a single weight of 100 kg.
A. 6.3 inches B. 5.3 inches C. 4.3 inches D. 7.0 inches The first spring is rated at 4 KN/m and the other spring is rated at 6 KN/m. Determine the total
A # A 1200 mm cast iron pulley is fastened to a 112.5 mm shaft by means of a 28.13 mm deflection of the springs.
square key 175 mm long. The key and the shaft have a shearing stress of 14,000 psi. A. 410 mm B. 310 mm C. 510 mm D. 210 mm
Determine the force acting at the pulley rim that will shear the key.
A. 10015 lb B. 11005 lb C. 11050 lb D. 10501 lb A # If the ultimate shear strength of a steel plate is 42,000 psi, what force is necessary to
A # A 75 mm diameter shaft is transmitting 300 KW at 600 rpm. A flange coupling is used
and has 6 bolts, each 18 mm in diameter. Find the required diameter of the bolt circle based on punch inch diameter hole in a 5/8 inch thick plate/
an average shearing stress of 27.5 Mpa. A. 61,850 lb B. 65,810 lb C. 61,580 lb D. 60,185 lb
A. 227.4 mm B. 477.2 mm C. 274.7 mm D. 247.7 mm C # In a straight bevel gear, the angle between an element of the pitch cone and an element of
C # A heavy duty shaft coupling is to be secured with 25 mm bolts at a distance of 150 mm the face cone is called:
from the shaft center. The shaft transmits 4330 KW of power at a speed of 1200 rpm. If the A. Face angle B. Pitch angle C. Addendum angle D. Dedendum
allowable shearing stress for bolts is 100 Mpa, how many bolts are required? angle
A. 3 B. 6 C. 5 D. 4 D # A four bar mechanism in which one of the links can perform a full rotation relative to the
B # A 1.75-inch diameter shaft is supported by two sleeve bearings. The total load on the two other three links.
bearings is 2800 lb. Find the friction power loss, in Hp, if the coefficient of friction between A. Geneva mechanism C. Triple rocker mechanism
shaft and bearing is 0.10 and the shaft rotates 200rpm. B. Crossover-position mechanism D. Grashof mechanism
A. 0.88 Hp B. 0.78 Hp C. 0.98 Hp D. 0.68 Hp A # A Grashof four-bar mechanism in which the shorted link is the frame or the fixed link and
B # Find the horsepower lost when a collar is loaded with 1000 lb, rotates at 25 rpm, and a the other two cranks completely rotate with their axes.
coefficient of friction at 0.15. The outside diameter of the collar is 4 inches and the inside A. Drank link mechanism C. Double-rocker mechanism
diameter is 2 inches. B. Crank-rocker mechanism D. Triple-rocker mechanism
A. 0.0629 Hp B. 0.0926 Hp C. 0.0269 Hp D. 0.0692 Hp C # “For planar four bar linkage, the sum of the shortest and the longest lengths cannot be
C # A sleeve bearing has an outside diameter of 38.1 mm and a length of 50.1 mm, the wall greater than the sum of the remaining two links” lengths if there is to be a continuous relative
thickness is 3/16 inch. The bearing is subjected to radial load of 450 lb, determine the bearing rotation between the two member” The preceding statement is known as:
pressure. A. Grubler’s law B. Coriolli’s law C. Grashof’s law D.
A. 100 psi B. 150 psi C. 200 psi D. 250 psi Freudentein’s law
A # A vertical steel cylinder water tank is 30 m in diameter and 45 m high. The allowable B # Which of the following is not true for an instant center or centro of planar linkages?
stress of the steel plate is 120 Mpa. Without reinforcing angle bars and rods, determine the A. Centro is a point common to two bodies having the same velocity in each.
thickness of the steel plate. B. Centro is a point in one body about which another body does tend to rotate.
A. 55.2 mm B. 56.2 mm C. 65.2 mm D. 52.6 mm C. Centro is a point in one body about which another body actually turns.
B # Determine the bursting steam pressure of a hemispherical steel shell with a diameter of D. Centro is a point in one body about which another body tends to turn.
100 inches and made of 0.0635 m thick steel plate. The joint efficiency is 70% and the tensile A # The most common work holding devices of a shaper machine with the base graduated in
strength is 98 psi. degrees that make it possible to swivel any angle.
A. 4020 psi B. 4200 psi C. 2500 psi D. 2040 psi A. Shaper vise B. Parallel bars and hold down bars C. Lathe holder D.
A # A cylinder having an internal diameter of 508 mm and external diameter of 914.4 mm is Swivel head
subjected to an internal pressure of 69 Mpa and an external of 14 Mpa. Determine the hoop A # Shaper operation which is shaping the given stock and having the excess material
stress at the side surface of the cylinder. remain with a tolerable allowance for finishing.
A. Roughing B. Finishing C. Angular Cutting D. Contouring
C # A cutting tool that has two or more cutting edges as in drill presses and milling machine B. Clearance journal bearing D. Full journal bearing
cutters. B # A lubrication where the load-carrying surfaces of the bearing are separated by a relatively
A. Grinder B. Single-point cutting tool C. Multi-point cutting tool D, thick film of lubricant, so as to prevent metal to metal contact, and where the stability of the
Two-point cutting tool film can be explained by the laws of fluid mechanics.
A # The trade name for a patented alloy made up chiefly of cobalt, chromium, and tungsten A. Hydrostatic lubrication C. Elastohydrodynamic lubrication
varying proportions. B. Hydrodynamic lubrication D. Boundary lubrication
A. Stellite B. Carbology C. Stainless Steel D. Copper A # A lubrication condition where non-conformal surfaces are completely separated by
A # The transformation of concepts and ideas into useful machinery is called as: lubricant film and no asperities are in contact.
A. Design B.Synthesis C. Analysis D. Theorem A. Elastohydrodynamic lubrication C. Hydrodynamic lubrication
C # It is a combination of mechanisms and other components that transform, transmits, or uses B. Boundary lubrication D. Hydrostatic lubrication
energy, load, or motion for a specific purpose. C # A speed at which rotating shaft becomes dynamically unstable.
A. Mechanism B. Engine C. Machine D. Linkage A. Normal Speed B. Variable Speed C. Critical Speed D. Average
Speed
B # It is defined as synergistic collection of machine elements; synergistic because as a design B # A ball bearing with race contacting pronounced groove for rolling elements.
it represents an idea or concept greater than the sum of the individual parts. A. Crown bearing B. Conrad bearing C. Angular-contact bearing D.
A. System of mechanism B. Mechanical system C. Design system D. Cylindrical bearing
Expert system A # A machining process for producing internal straight cylindrical surface or profiles, with
C # It may be defined the displacement per length produced in a solid as the result of stress. process characteristics and tooling similar to those for turning operations.
A. Deformation B. Elongation C. Strain D. Stress A. Boring B. Drilling C. Reaming D. Milling
D # The combination of applied normal and shear stresses that produces maximum principal B # A machining operation for all types of metallic and nonmetallic materials and is capable
normal stress, with a third principal stress between or equivalent to the extremes. of producing circular parts with straight of various profiles.
A. Principal shear stress C. Maximum shear stress A. Boring B. Turning C. Drilling D. Milling
B. Principal normal stress D, Bending and shear stress B # A set of specification for parts, materials, or processes intended to achieve uniformity,
C # It is a load applied transversely to longitudinal axis of member. efficiency, and a specified quality.
A. Combined loads B. Concentrated load C. Bending load A. Code B. Standard C. Law D. Theorem
D. Distributed load A # A set of specification for the analysis, design, manufacture, and construction of
C # It is the intensity and direction of internal force acting at a given point on particular plane. something; the purpose of which is to achieve a specified degree of safety, efficiency, and
A. Load B. Strain C. Stress D. Sustained load performance or quality.
A # It is the capacity of a material to absorb energy when it is deformed elastically and then, A. Code B. Standard C. Law D. Theorem
upon unloading, to release this energy. # It is defined as synergistic collection of machine elements; synergistic because as a design it
A. Resilience B. Toughness C. Rigidity D. Ductility represents an idea or concept greater than the sum of the individual parts.
D # It is the strain energy per unit volume required to stress a material from an unloaded state A. System of mechanism B. Mechanical system C. Design system
to the point of yielding. D. Expert system
A. Modulus of roughness C. Modulus of rigidity
B. Modulus of elasticity D. Modulus of resilience # It may be define the displacement per length produced in a solid as a result of stress.
A # The ability of the material to absorb energy up to fracture. A. Deformation B. Elongation C. Strain D. Stress
A. Toughness B. Rigidity C. Resilience D. Stiffness
B # The Maximum Shear Stress Theory, as a failure prediction theory, is also known as: # The combination of applied normal and shear stresses that produces maximum principal
A. von Mises criterion B. Tresca yield criterion C. Coulomb-Mohr normal stress, with a third principal stress between or equivalent to the extremes.
Theory A. Principal shear stress C. Maximum shear stress
D. Modified Mohr Theory B. Principal normal stress D. Bending and shear stress
B # A failure prediction theory, which states that a part subjected to any combination of loads,
will fail (by yielding or fracturing) whenever the maximum shear stress exceeds a critical # It is a load applied transversely to longitudinal axis of member.
value. A. Combined load B. Concentrated load C. Bending load
A. Distorsion-energy theory C. Internal friction theory D. Distributed load.
B. Maximim-shear-stress theory D, Modified Mohr theory
A # A theory in a cyclic and impact loading, which states that damage at any stress level is # It is the intensity and direction of internal force acting at given point on particular plane.
proportional ro the number of cycles. A. Load B. Strain C. Stress D. Sustained load
A, Miner’s rule B. Paris Power Law C. Goodman Rule D. Manson-
Coffin Relationship # It is the capacity of a material to absorb energy when it is deformed elasticity and then,
B # A journal bearing where the radius of the journal is less than the radius of the bushing or upon unloading, to release this energy.
bearing. A. Resilience B. Toughness C. Rigidity D. Ductility
A. Fitted journal bearing C. Partial journal bearing
# It is the strain energy per unit volume required to stress a material from an unloaded state to # An American nonprofit society, founded in 1921, whose objectives are to improve and
the point of yielding. advance the use of fabricated structural stress.
A. Modulus of roughness B. Modulus of elasticity C. Modulus of rigidity A. American Iron Steel Institute (AISI)
D. Modulus of resilience B. American Institute of Steel Construction (AISC)
C. American Society for Metals (ASM)
# The ability of the material to absorb energy up to the fracture. D. American Society of Testing and Materials (ASTM)
A. Toughness B. Rigidity C. Resilience D. Stiffness
# A sketch of a machine, a machine element, or part of a machine element that shows all
# The Maximum Shear Stress Theory, as a failure prediction theory, is also known as: acting forces, such as applied load and gravity forces, and all reactive forces.
A. von Mises criterion B. Tresca yield criterion C. Coulomb-Mohr theory A. Schematic diagram B. Free body diagram C. Moment diagram
D. Modified Mohr-theory D. Skeletal diagram

# A failure prediction theory, which states that a part subjected to any combination of # The size to which a limit of deviations is assigned and is the same for both members of the
loads, will fail (by yielding and fracturing) whenever the maximum shear stress exceeds a fit, it is the exact theoretical size.
critical value. A. Nominal stress B. Basic size C. Maximum size D. Minimum size
A. Distorsion-energy theory C. Internal friction theory
B. Maximum-shear-stress theory D. Modified Mohr theory # The algebraic difference between a size and the corresponding basic size.
A. Tolerance B. Allowance C. Deviation D. Limit
# A theory in cyclic and impact loading, which states that damage at any stress level is
proportional to the number of cycles. # The algebraic difference between the maximum limit and the corresponding basic.
A. Mine’s rule B. Paris Power Law C. Goodman Rule A. Fundamental deviation B. Upper Deviation C. Lower deviation D.
D. Manson-coffin Relationship Tolerance

# A journal bearing where the radius of the journal is less than the radius of the bushing or # The algebraic difference between the minimum limit and the corresponding basic.
bearing. A. Fundamental deviation B. Upper Deviation C. Lower deviation D.
A. Fitted journal bearing C. Partial journal bearing Tolerance
B. Clearance journal bearing D. Full journal bearing
# Either the upper of the lower deviation, depending on which is closer to the basic size.
# A lubrication where the load-carrying surfaces of the bearing are separated by a relatively A. Fundamental deviation B. Upper Deviation C. Lower deviation D.
thick film of lubricant, so as to prevent metal to metal contact, and where the stability of the Tolerance
film can be explained by the laws of fluid mechanics.
A. Hydrostatic lubrication C. Elastohydrodynamic lubrication # The difference between the maximum and minimum size limits of a part.
B. Hydrodynamic lubrication D. Boundary lubrication A. Allowance B. Tolerance C. Deviation D. Basic size

# A lubrication condition where non-conformal surfaces are completely separated by lubricant # The stated maximum and minimum dimensions.
film and no asperities are in contact. A. Tolerance B. Limits C. Nominal size D. Basic size
A. Elastohydrodynamic lubrication C. Hydrodynamic lubrication
B. Boundary lubrication D. Hydrostatic lubrication # A general term that refers to the mating of cylindrical parts such as bolt or a hole; it is used
only when the internal member is smaller that the external member.
# A speed at which rotating shaft becomes dynamic unstable. A. Clearance B. Interference C. Allowance D. Tolerance
A. Normal speed B. Variable speed C. Critical speed D. Average speed
# The opposite of clearance, for mating cylindrical parts in which the internal member is
# A ball bearing with race containing pronounced groove for rolling elements. larger than the external member.
A. Crown bearing B.Conrad bearing C.Angular-contact bearing A. Clearance B. Allowance C. Tolerance D. Interference
D.Cylindrical bearing
# The minimum stated clearance or the maximum stated interference for mating parts.
# A machining process for producing internal straight cylindrical surface or profiles, with A. Clearance B. Allowance C. Tolerance D. Interference
process characteristics and tooling similar to those for turning operations.
A.Boring B.Drilling C.Reaming D.Milling # The property of a material that measures the degree of plastic deformation sustained at
fracture.
# A machining operation for all types of metallic and nonmetallic materials and is capable of A. Toughness B. Stiffness C. Ductility C. Brittleness
producing circular parts with straight or various profiles.
A.Boring B.Turning C.Drilling D.Milling # Compounds of metallic elements, most frequently oxides, nitrides, and carbides.
A. Plastic B. Polymers C. Ceramics D. Alloy

# A material having different properties in all directions at point in solid.


A. Isotropic material B. Anisotropic material C. Orthropic material
D. Ceramic material

# A material having different properties in three mutually perpendicular directions at point in


solid and having three mutually perpendicular planes of material symmetry.
A. Orthotropic material B. Isotropic material C. Anistropic material
D. Thermoplastic material

# The combination of two or more materials, usually consisting of fiber and thermosetting
polymer.
A. Brittle materials B. Composite materials C. Polymers D. Ceramics

# A theorem stating that “when a body is elastically deformed by a system of loads, the
deflection at any point p in any direction a is equal to the partial derivatives of the strain energy
(with the system of loads acting) with respect to a load at p in the direction a”.
A. Poisson’s Theorem B. Newton’s Theorem C. Castigliano’s Theorem
D. Mohr’s Theorem

# A principal or method that a deflection at any point in bar is equal to sum f deflection caused
by each load acting separately.
A. Summation Method B. Method of balancing C. Method of superposition
D. Shear and Moment diagram method

# A failure prediction theory in which failure is caused by the elastic energy associated with
shear deformation.
A. Maximum-shear-stress theory B. Distorsion-energy theory C. Maximum-normal-stress
theory
D. Internal friction theory

You might also like